ML081370232

From kanterella
Jump to navigation Jump to search
Draft - RO & SRO Written Exam (Folder 2)
ML081370232
Person / Time
Site: FitzPatrick Constellation icon.png
Issue date: 02/28/2008
From:
Operations Branch I
To:
Hansell S
Shared Package
ML072850856 List:
References
ES-401-5 NUREG-1021, Rev 9
Download: ML081370232 (221)


Text

ES-401 Sample Written E:xa minat ion Form ES-401-5 Question Worksheet Examination Outline Cross-reference: Leviel RO SRO Tier # 1 Group # 1 KIA # 295001 AKI .02 lmplortance Rating 3.3 3.5 295001 Partial or Complete Loss of Forced Core Flow Circulation AK1 . Knowledge of the operational implications of the following concepts as they apply io PARTIAL OR COMPLETE LOSS OF FORCED CORE FLOW CIRCULATION : (CFR: 41.8 to 41 . I O ) AKI .02 Power/flow distribution Proposed Question: #I Plant conditions are as follows:

0 100% power.

95% core flow.

0 A spurious trip of Feedwater Pump Aoccurs.

0 A runback of both reactor recirculation plumps occurs.

0 NO operator actions are taken Which ONE of the following is the approximate reactor power and core flow value one minute later based on automatic actions and the above conditions?

A. 57% reactor power, 35% core flow B. 66% reactor power, 45% core flow C. 65% reactor power, 52% core flow D. 75% reactor power, 60% core flow Proposed Answer: B Explanation (Optional): Plant is on approximately 102% rod line initially, power comes down on same rod line in response to flow reduction. Runback to # 2 speed limiter; (44%), occurs if less than two RFPs operating and RPV level less than 196.5 inches is sensed. AOP-8 provides estimate of power at 75% for runback of 1 pump,,65% for trip of 1 pump, 52% for trip of both pumps, a runback of two pumps will be to 44% and approximately 62% power.

A. Incorrect - this value is in response to trip of both recirculation pumps B. Correct - this value is in response to runback to 44% of both recirculation pumps C. Incorrect - this value is in response to trip of one recirculation pump D. Incorrect - this value is in response to runback to 44% of one recirculation pump Technical Reference(s): RAP-7.3.16 Rev 41 (Attach if not previously provided)

AOP-8 Rev 29 NUREG-1021, Revision 9

ES-401 Sample Written Examination Form ES-401-5 Question Worksheet Proposed references to be provided to applicants during examination: RAP-7.3.16Attachment

  1. 4 Power to Flow Map Learning Objective: SDLP-021 EO-1.09.A & B (As available)

Question Source: Bank #

Modified Bank # (Note changes or attach parent)

New X Question History: Last NRC Exam Question Cognitive Level: Memory or Fundamental Knowledge Comprehension or Analysis X 10 CFR Part 55 Content: 55.41 55.43 Comments:

Note changed stem from runback to 44% to elimiinate clue to answer of question. Changed answer to B from A to allow for ordinate response on the flow value.

NUREG-1021,Revision 9

ES-401 Sample Written Examination Form ES-401-5 Question Worksheet POWER - :PLOW MAP PLANT POWEI Rev. N o . 4 NUREG-1021, Revision 9 3 of 3

ES-401 Sample Written Examination Form ES-401-5 Question Worksheet Examination Outline Cross-reference: Level RO SRO Tier # 1 Group # 1 WA # 295003 AK2.02 Importance Rating 4.1* 4.2*

295003 Partial or Complete Loss of A.C. Power AK2. Knowledge of the interrelations between PARTIAL OR COMPLETE LOSS OF A.C. POWER and the following: (CFR: 41.7 / 45.8) AK2.02 Emergency generators Proposed Question: #2 The plant is operating at 100% power with a normal electrical lineup when the following annunciators alarm:

0 09-8-3-25 Bus 10300 Reserve Supp Bkr 10312 Trip 0 09-8-3-26 Bus 10300 Norm Supp Bkr IO302 Trip Assume operator actions have been taken zind all equipment functions as designed.

What will be the status of the 4 kV emergency bus breakers after this event?

A. ACB 10502 (EDG A Load Bkr) Closed, ACB 10512 (EDG C Load Bkr) Closed, ACB 10304 (Bus 10300 - 10500 Tie Bkr) Closed, ACB 10514 (Bus 10300 - 10500 Tie Bkr) Open, ACB 10504 (EDG A & C Tie Bkr) Open.

B. ACB 10502 (EDG A Load Bkr) Closed, ACB 10512 (EDG C Load Bkr) Closed, ACB 10304 (Bus 10300 - 10500 Tie Bkr) Open, ACB 10514 (Bus 10300 - 10500 Tie Bkr:) Closed, ACB 10504 (EDG A & C Tie Bkr) Open.

C. ACB 10502 (EDG A Load Bkr) Closed, ACB 10512 (EDG C Load Bkr) Closed, ACB 10304 (Bus 10300 - 10500 Tie Bkr) Open, ACB 10514 (Bus 10300 - 10500 Tie Bkr) Open, ACB 10504 (EDG A & C Tie Bkr) Closed.

D. ACB 10502 (EDG A Load Bkr) Closed, ACB 10512 (EDG C Load Bkr) Closed, ACB 10304 (Bus 10300 - 10500 Tie Bkr) Open, ACB 10514 (Bus 10300 - 10500 Tie Bkr) Open, ACB 10504 (EDG A & C Tie Bkr) Open.

NUREG-1021, Revision 9 1of2

ES-401 Sample Written EIxamination Form ES-401-5 Question Worksheet Proposed Answer: D Explanation (Optional): Stem results in a loss of power to the 10300 Bus, the EDGs A&C auto start and force parallel , (ACB 10504 closes init,iallythen subsequently opens when a EDG >

90% and closes onto the 10500 bus) The resultant bkr alignment per OP-22 Step G.1.2 will be 10512 closed, 10502 closed, 10504 open, 10304 open, 10514 open.

A. Incorrect - ACB 10304 (Bus 10300 - 10500 Tie Bkr) should be Open vice Shut.

B. Incorrect - ACB 10514 (Bus 10300 - 10500 Tie Bkr) should be Open vice Shut.

C. Incorrect - ACB 10504 (EDG A & C Tie Bkr) should be Open vice Shut.

D. Correct - Breakers are in correct position per OP-22 Step G.1.2.

Technical Reference(s): OP-22Rev.52 Step G.1.2 (Attach if not previously provided)

AOP-16 Rev 14 Proposed references to be provided to applicants during examination: NONE Learning Objective: (As available)

Question Source: Bank #

Modified Bank # (Note changes or attach parent)

New X Question History: Last NRC Exam Question Cognitive Level: Memory or Fundamental Knowledge Comprehension or Analysis X 10 CFR Part 55 Content: 55.41 55.43 Comments:

NUREG-1021, Revision 9 2of2

ES-401 Sample Written Examination Form ES-40 1-5 Question Worksheet Examination Outline Cross-reference: Level RO SRO Tier # 1 Group # 1 KIA # 295004 AK3.03 Importance Rating 3.1 3.5 295004 Partial or Complete Loss of D.C. Power - AK3. Knowledge of the reasons for the following responses as they apply to PARTIAL OR COMPLETE LOSS OF D.C. POWER: (CFR: 41.5 / 45.6)

AK3.03 Reactor SCRAM: Plant-Specific Proposed Question: #3 Plant startup is in progress with reactor power at 39%.

All systems are in a normal lineup per procedures with the following exception:

Bus 10300 loads remain supplied from off-site power due to mechanical binding in breaker NSS TO BUS 10300 BKR 1030:z.

Subsequently, a loss of DC Power System A occurs with a resulting reactor scram.

What is the reason for the reactor scram?

A. Main turbine trip from high RPV water level, due to loss of DC power to the selected Rx Wtr Lvl column.

B. Inboard MSlV closure due to loss of DC power to the solenoids.

C. Main turbine trip due to loss of DC power to EHC trip logic.

D. Low RPV water level due to loss of DC power to Feedwater and Steam flow instruments.

Prbposed Answer: C NUREG-I 021, Revision 9 1 Of3

ES-401 Sample Written Examination Form ES-401-5 Question Warksheet Explanation (Optional): Per AOP-45 , Loss of DC Power System A- Section C- AUTOMATIC ACTIONS-Main turbine trip from loss of DC power to EHC trip logic causing the following:

- If Rx power is > 29 YO,a Rx scram occurs from main turbine stop valve closure.

- If 10100, 10200, 10300, 10400, 10500, 10600, & 10700 Buses were energized from Normal Station Service Transformer 71T-4, the following occurs:

0 10100, 10300, 10500, & 10700 E3uses are lost.

0 Inboard MSlVs close & a Rx scram occurs.

0 10400 Bus residual transfers to reserve power.

10600 Bus residual transfers to either reserve power or EDGs B & D 0 UPS Bus is lost Loss of DC Power System Aresults in a loss of control power to 10100, 10300, 10500, and 10700 Bus breakers. When the generator subsequently trips power will be loss to all loads associated with 10100, 10500, and 10700, however due to stem 10300 will still remain powered, as BUS 10300 is being supplied from RSS instead of NSS and would take a trip of BUS 10300 RESERVE SUPP BKR 10312 to deenergize it.

Loss of DC Power System A results in a downscale failure of FDWTR FLOW 06FI-89A and STM FLOW 06FI-88A and C.

A. Incorrect - Loss of DC power system1 A affects Rx Wtr Lvl 06LI-94A & C, not RX WTR LVL 06LI-94B, which is the normal selected column.

6. Incorrect -Rx would have scrammed clue to response noted in the Correct answer prior to any MSlV closure caused by loss of both DC & AC control power. To lose power to the Inboard MSlV solenoids would require 29AOV-80A-D MSIV DC Inboard Solenoids powered from .

71 DC-A2 & 29AOV-80A-D MSlV AC Inboard Solenoids powered from RPS A to be de-energized. Power has been lost to the DC solenoids however, RPS MG Set 1A is fed from MCC-251 fed via 10500 which would not lose power till the generator trips & the 10500 buss loses power as it will not transfer due to loss of DC control power to the 10500 breakers.

C. Correct Response - Main turbine trip from loss of DC power to EHC trip logic, If Rx power >.29 %, a Rx scram will occur from main turbine stop valve closure.

D. Incorrect - Loss of DC Power System A results in a downscale failure of FDWTR flow 06FI-89A & Stm flow 06FI-88A & C. Control power is lost to RFP A but RFP B is not affected. The Rx would have scrammed due to response noted in the Correct answer prior to any scrams caused by water level from Steam /FW flow mismatch & loss of RFPT A speed control.

Technical Reference(s): AOP-45 R 9 (Attach if not previously provided)

AOP-21 R 21, AOP-16 R 14, OP-46B R 26, OP-65 R 106 Proposed references to be provided to applicants during examination: NONE Learning Objective: SDLP-94A EO-1.IO.F (As available)

NUREG-1021, Revision 9 20f3

ES-401 Sample Written Eixamination Form ES-401-5 Question Worksheet Question Source: Bank #

Modified Bank # (Note changes or attach parent)

New X Question History: Last NRC Exam Question Cognitive Level: Memory or Fundamental Knowledge Comprehension or Analysis X 10 CFR Part 55 Content: 55.41 55.43 Comments: Added to the stem to support distracter A: All systems are in a normal lineup per procedures with the exception of:

NUREG-1021, Revision 9 3 of 3

ES-401 Sample Written EIxamination Form ES-401-5 Question Worksheet Examination Outline Cross-reference: Levlel RO SRO Tier # 1 Group # 1 WA # 295005 AA1.04 Importance Rating 2.7 295005 Ability to operate and/or monitor the following as they apply to MAIN TURBINE GENRATOR TRIP: (CFR:41.7 / 45.6) MI.04 Main Generator controls Proposed Question: #4 The plant is at 100% power when annunciator 09-7-1-26 MAIN GEN EXCITER FIELD BKR TRIP alarms. All systems/componentsfunction per design.

Circuit breakers 71PCB-10042 and 71PC13-10052 open Disconnect 71 MOD-10031 opens The Exciter Field Breaker opens What turbine component(s)/parameters condition caused the generator breakers response?

A. Low bearing oil pressure at 5 10 psig B. Turbine Stop and Intermediate Stop valves closed C. Low EHC pressure at 5 800 psig D. Turbine Bypass valves closed Proposed Answer: B Explanation (Optional):

Technical Reference(s): (Attach if not previously provided)

Proposed references to be provided to applicants during examination:

Learning Objective: 1.10 SDLP94D (As available)

Question Source: Bank #

Modified Bank # (Note changes or attach parent)

New X NUREG-1021, Revision 9 1 of2

ES-401 Sample Written Eixamination Form ES-401-5 Question Worksheet Question History: Last NRC Exam Question Cognitive Level: Memory or Fundamental Knowledge X Comprehension or Analysis 10 CFR Part 55 Content: 55.41 55.43 Comments:

NUREG-1021, Revision 9 2 of 2

ES-401 Sample Written Examination Form ES-401-5 Question Worksheet Examination Outline Cross-reference: Level RO SRO Tier # 1 Group # 1 WA # 295006 AA2.03 Importance Rating 4.0 4.2*

295006 SCRAM -AA2. Ability to determine and/or interpret the following as they apply to SCRAM: (CFR: 41 . I O / 43.5 / 45.13) AA2.03 Reactor water level Proposed Question: #5 A plant startup was in progress with the following plant conditions:

0 Reactor power was 10%.

0 FDWTR STARTUP VLV (34FCV-137) WiaS in AUTO controlling RPV level.

0 Reactor Feed Pump (RFP) A in-service.

0 Subsequently, RFP A tripped and RPV level approached 180 inches before RFP B could be started and placed in service.

0 The lowest RPV water level observed WEIS 150 inches.

0 A manual scram was initiated when RPV level reached 180 inches.

The following conditions now exist:

0 Reactor power at 0%, all control rods inserted.

0 Reactor Scram has been reset.

0 RPV pressure is 900 psig and stable.

0 RFP B in service.

0 FDWTR STARTUP VLV (34FCV-137) is in AUTO indicating fully shut.

0 RPV level at 206.5 inches and slowly risiing (-4 inches per minute).

Which of the following is the reason RPV water level is rising?

A. Thermal expansion of the cool Feedwater injected after the scram.

B. Level swell due to normal turbine steam bypass valve operation.

C. Level swell due to a failed open SRV D. Level rise due to HPCl automatic injection.

Proposed Answer: A Explanation (Optional):

NUREG-1021, Revision 9 1 Of3

ES-401 Sample Written Examination Form ES-401-5 Question Worksheet A. Correct response - In response to the scram, FDWTR STARTUP VLV (34FCV-137) is at its setpoint & closed, FW is no longer injecting, cold FW is now heating up

& causing swelling of RPV level.

B. Incorrect - Bypass valves are closed due to pressure setpoint is 970 psig per OP-65.

C. Incorrect - Stem provides that post scram the RPV pressure is 900 psig & stable thus a failed SRV is not responsible for RPV level swell.

D. Incorrect - At 10% power as given in stem, level will not reach the HPCl initiation setpoint on level, additionally, level would have recovered with RClC &

FW prior to the HPCl injection valve opening. See stem 150 Inches lowest level observed.

Technical Reference(s): AOP-1 R 43 (Attach if not previously provided)

OP-65 R 106 Proposed references to be provided to applicants during examination: NONE Learning Objective: LP-AOP EO-1.02 (As available)

Question Source: Bank #

Modified Bank # 24512 (Note changes or attach parent)

RBS New Question History: Last NRC Exam RBS 2/1/03 Question Cognitive Level: Memory or Fundamental Knowledge Comprehension or Analysis X 10 CFR Part 55 Content: 55.41 55.43 Comments: Note JAF does not have Feedwater level control setpoint set-down operation-changed the distracter to Level rise caused by HPCl automatic injection.

Added the following to the stem for clarification - clarifies HPCl not injecting for distracter D.

0 The lowest RPV water level observed was 150 inches.

Changed the following to reflect JAF operation:

Subsequently, RFP A tripped and RPV level approached 180 inches (instead of 190 inches- reason -closer to scram setpoint) before RFP B could be started and placed in service.

0 A manual scram was initiated when RPV level reached 180 inches (instead of 190 inches- reason -closer to scram setpoint) and eliminated opening the RFP B discharge valve as FW would be controlled on the FDWTR STARTUP VLV (34FCV-137) in auto.

Changed:

The following conditions now exist:

NUREG-1021, Revision 9 2 of 3

ES-401 Sample Written Eixamination Form ES-401-5 Question Worksheet a RFP 6 in service due to in service with its discharge valve fully open to just RFP 6 FW would be controlled on the FDWTR STARTUP VLV (34FCV-137) in auto.

RPV level at 206.5 inches (vice 180 inches due to directions are to maintain in normal band 196.5 to 206.5 inches).

Added Reactor Scram has been reset to eliminate input from CRD.

NUREG-1021, Revision 9 3 of 3

ES-40 1 Sample Written Examination Form ES-401-5 Question Woirksheet Examination Outline Cross-reference: Level RO SRO Tier # 1 Group # 1 WA # 295016 G 2.4.12 Importance Rating 3.4 295016 Control Room Abandonment G2.4.12 Knowlledge of general operating crew responsibilities 0 / 45.12 )

during emergency operations. (CFR: 41 .I Proposed Question: #6 With the plant at 100% power a large smoky fire occurs in the Control Room and the CRS announces that he has entered AOP-43 and control room evacuation is required. You are the ATC operator.

Per AOP-43, which of the following actions are you as the ATC operator required to complete before leaving the Control Room?

A. Trip the main turbine at Panel 09-5

6. Initiate SLC C. Transfer RPS MG Set to ALT D. Start at least one EDG Proposed Answer: A Explanation (Optional):

Technical Reference(s): AOP-43 (Attach if not previously provided)

Proposed references to be provided to applicants during examination:

Learning Objective: (As available)

Question Source: Bank #

Modified Bank # (Note changes or attach parent)

New _.

X Question History: Last NRC Exam NUREG-1021, Revision 9 1 of2

~ ~~

ES-40 1 Sample Written Examination Form ES-401-5 Question Woirksheet Question Cognitive Level: Memory or Fundatmental Knowledge X Comprehension or Analysis 10 CFR Part 55 Content: 55.41 55.43 Comments:

NUREG-I 021, Revision 9 2 of 2

ES-40 1 Sample Written Examination Form ES-401-5 Question Worksheet Examination Outline Cross-reference: Level RO SRO Tier # 1 Group # 1 KIA # 295018 AKI .01 Importance Rating 3.5 3.6 295018 Partial or Complete Loss of Component Cooling Water AKl . Knowledge of the operational implications of the following concepts as they apply to PARTIAL OR COMPLETE LOSS OF COMPONENT COOLING WATER : (CFR: 41.8 to 41.IO)

AKI .01 Effects on componentkystem operations Proposed Question: #7 The Plant is operating at 90% power with one Reactor Building Closed Loop Cooling (RBCLC) pump tagged out of service.

An electrical problem causes the two running RBCLC pumps to trip.

Operators have the ability to restore cooling via Emergency Service Water to EACH of the following EXCEPT:

A. Drywell Ventilation Coolers

6. RWCU Pump Coolers C. Control Rod Drive Hydraulic Pump Coolws D. Drywell Equipment Drain Sump Cooler Proposed Answer: B Explanation (Optional): RWCU Pump Coolers are NOT supplied by ESW.

A. Incorrect - Can be manually aligned for cooling with ESW.

6. Correct Response - RWCU Pump Coolers are NOT supplied by ESW C. Incorrect - Can be manually aligned for cooling with ESW D. Incorrect - Can be manually aligned for cooling with ESW Technical Reference(s): AOP-11 R15 (Attach if not previously provided)

Proposed references to be provided to applicants during examination: NONE Learning Objective: SDLP-15 EO-1.06.6 (As available)

Question Source: Bank #

Modified Bank # (Note changes or attach parent)

NUREG-1021. Revision 9 1of2

ES-401 Sample Written Examination Form ES-401-5 Question Worksheet New X Question History: Last NRC Exam Question Cognitive Level: Memory or Fundamental Knowledge X Comprehension or Analysis 10 CFR Part 55 Content: 55.41 55.43 Comments:

NUREG-I 021, Revision 9 2of2

ES-40 1 Sample Written Examination Form ES-401-5 Question Woirksheet Examination Outline Cross-reference: Level RO SRO Tier # 1 Group # 1 WA # 295019 AK2.09 Importance Rating 3.3 3.3 295019 Partial or Complete Loss of Instrument Air AK2. Knowledge of the interrelations between PARTIAL OR COMPLETE LOSS OF INSTRUMENT AIR and the following: (CFR: 41.7 / 45.8) AK2.09 Containment Proposed Question: #8 Drywell sprays are in service to support EOP actions when a complete loss of nitrogen occurs.

Due to a logic failure, the drywell spray valves (IOMOV-26A & B, IOMOV-31A & B) have been opened locally using the local handwheels.

Is the Torus AUTOMATICALLY protected from exceeding design parameter(s) under these conditions and why/why not?

A. Yes. Drywell vacuum breakers will open tlo relieve Torus pressure to the Drywell.

B. Yes. Reactor Building to Torus vacuum breakers will open at 0.5 psid, Reactor Building pressure greater than Torus pressure.

C. No. Drywell vacuum breakers fail closed on a loss of pneumatic pressure and will fail to relieve Torus pressure to the Drywell.

D. No. Reactor Building to Torus vacuum breakers are isolated by pneumatically operated butterfly valves, which fail closed on a loss; of pneumatic pressure.

Proposed Answer: D Explanation (Optional): Per DBD-OIGA, AOP-12 and SDLP-166 lnst N2 supplies 27AOV-101A(B) RB to Torus Vacuum Bkr isolations which fail closed on a loss of pneumatics. Per stem as DW pressure lowers from spray operation, it will result in torus to DW vacuum breaker (purely mechanical) opening to relieve higher torus pressure to DW. As torus pressure lowers, the RB atmosphere would normally be relieved to the torus via the 27AOV-I01A(B) path but due to a loss of pneumatics, these valves will fail closed.

NUREG-1021, Revision 9 1of2

ES-40 1 Sample Written Examination Form ES-401-5 Question Worksheet A. Incorrect - lnst N2 supplies 27AOV-l0IA(B) RB to Torus Vacuum Bkr isolations which fail closed on a loss of pneumatics thus the containment is not protected automatically from a negative pressure condition. DW vacuum breaker functions independently of IA but will not prevent containment failure under the conditions given in the stem.

6. Incorrect - lnst N2 supplies 27AOV-I01A(B) RB to Torus Vacuum Bkr isolations which fail closed on a loss of pneumatics thus the containment is not protected automatically from a negative pressure condition. DW vacuum breaker functions independently of IA but will not prevent containment failure under the conditions given in the stem.

C. Incorrect - lnst N2 supplies 27AOV-I01A(B) RB to Torus Vacuum Bkr isolations which fail closed on a loss of pneumatics thus the containment is not protected automatically from a negative pressure condition. DW vacuum breaker functions independently of IA but will not prevent containment failure under the conditions given in the stem.

D. Correct - lnst N2 supplies 27AOV-1OIA(B) RB to Torus Vacuum Bkr isolations which fail closed on a loss of pneumatics thus the containment is not protected automatically from a negative pressure condition.

Technical Reference(s): DBD-Ol6A Rev 5 (Attach if not previously provided)

AOP-12 Rev 26 Proposed references to be provided to applicants during examination:

Learning Objective: SDLP-16B EO-1.1O.B (As available)

Quest ion Source: Bank #

Modified Bank # (Note changes or attach parent)

New Question History: Last NRC Exam Question Cognitive Level: Memory or Fundamental Knowledge Comprehension or Analysis X 10 CFR Part 55 Content: 55.41 55.43 Comments: Added automatically to the stem- operators have the ability to still stop the RHR pumps thus securing spray action.

In the stem- Due to a logic failure, the drywell spray valves (1OMOV-26A & 6, 1OMOV-31A 8, 6) have been opened locally using the focal handwhsels. This is not covered by a procedure as a specific instruction.

NUREG-1021, Revision 9 2 of 2

ES-40I Sample Written Examination Form ES-401-5 Question Worksheet Examination Outline Cross-reference: Level RO SRO Tier # I Group # 1 WA # 295021 AK3.05 Importance Rating 3.6 295021 Knowledge of the reasons for the following responses as they apply to LOSS OF SHUTDOWN COOLING (CFR: 41.5 / 45.6) AK3.05 Establishing alternate heat removal flow paths.

Proposed Question: #9 The reactor is shutdown at 70 psig with the 8 RHR pump in shutdown cooling.

  • RPV level is 195 inches.
  • B reactor recirc pump is running.

Subsequently, a loss of off-site power (LOOP) occurs.

  • All EDGs start and load onto the emergency busses.

-Reactor pressure increases to 115 psig.

Given the above conditions, which of the following actions will remc re decal heat from the core?

A. Manually start RClC

6. Increase RPV water level to 230 inches C. Ensure 6 RHR pump restarts D. Start A RHR pump Proposed Answer: A Explanation (Optional): With a reactor pressure increase of 30 psig from the starting condition of 70 psig, shutdown cooling would isolate. RClC would be choice to remove heat and reduce reactor pressure Technical Reference(s): (Attach if not previously provided)

Proposed references to be provided to applicants during examination:

Learning Objective: (As available)

NUREG-1021, Revision 9 1of2

ES-40 1 Sample Written Examination Form ES-401-5 Question Worksheet Question Source: Bank #

Modified Bank # (Note changes or attach parent)

New X Question History: Last NRC Exam Question Cognitive Level: Memory or Fundamental Knowledge Comprehension or Analysis X 10 CFR Part 55 Content: 55.41 55.43 Comments:

NUREG-1021, Revision 9 2of2

ES-401 Sample Written E:xamination Form ES-40 1-5 Question Worksheet Examination Outline Cross-reference: Level RO SRO Tier # I Group # 1 WA # 295023 AA1.02 Importance Rating 2.9 3.1 295023 Refueling Accidents AA1 . Ability to operate and/or monitor the following as they apply to REFUELING ACCIDENTS: (CFR: 41.7 / 45.6) AAI .02 Fuel pool cooling and cleanup system Proposed Question: # 10 The plant is shutdown for a refueling outage with the fuel pool gates installed.

Annunciator: 09-3-1-9 Fuel Pool Cool & Cln Up Trouble alarms.

The NPO reports that the spent fuel pool level is' slowly dropping and the running fuel pool cooling pump has tripped.

Which one of the following methods is available to provide makeup to the spent fuel pool?

A. Align and inject core spray into the reactor cavity.

B. Start the second fuel pool pump to refill the pool.

C. Align condensate transfer to the skimmer surge tanks.

D. Start a second control rod drive pump to inject into the reactor cavity.

Proposed Answer: C Explanation (Optional): Per the stem the loss of lsvel has caused the FPC pump to trip on low surge tank level. With the gates installed the only viable method listed of making up level is condensate transfer. The other methods are not available due to the gates installed refer to note in AOP-53.

A. Incorrect - not available due to the gates installed refer to note in AOP-53.

6. Incorrect - will not add water, pump takes suction off the surge tank which receives water from the pool which has a lowering level.

C. Correct - guidance is given in AOP-53 to lutilize this method regardless of the gate status.

D. Incorrect - not available due to the gates installed refer to note in AOP-53.

Technical Reference(s): AOP-53 Rev 8 (Attach if not previously provided)

Proposed references to be provided to applicants during examination:

Learning Objective: SDLP-19 EO-1.15.A (As available)

NUREG-1021, Revision 9 1of2

ES740I Sample Written Examination Form ES-401-5 Question Worksheet Question Source: Bank #

Modified Bank # (Note changes or attach parent)

New X Question History: Last NRC Exam Question Cognitive Level: Memory or Fundamental Knowledge Comprehension or Analysis X 10 CFR Part 55 Content: 55.41 55.43 Comments:

NUREG-1021, Revision 9 2of2

ES-401 Sample Written Examination Form ES-401-5 Question Worksheet Examination Outline Cross-reference: LeveI RO SRO Tier # 1 Group # 1 KIA # 295024 EA2.02 Importance Rating 3.9 295024 EA 2.02 Ability to determine andlor interpret the following as they apply to HIGH DRYWELL PRESSURE (CFR: 41 .I 0 / 43.5 I45.13) Drywell Temperature Proposed Question: # 11 The reactor was at 100% power when a small break LOCA (SBLOCA) and loss of off-site power (LOOP) occurs. All automatic actions occur as designed.

HPCl flow is 4300 gpm Drywell pressure is 18 psig and stable Reactor pressure is 1000 psig and stable Reactor water level is 100 inches and increasing The crew has entered EOP-4 Primary Containment Control, Regarding containment integrity, which of the following is the most important parameter or equipment condition to track prior to initiating drywell sprays?

A. Drywell temperature

6. Drywell pressure C. Torus temperature D. Torus level Proposed Answer: A Explanation (Optional):

Technical Reference(s): EOP-4 (Attach if not previously provided)

Proposed references to be provided to applicants during examination:

Learning Objective: (As available)

NUREG-1021, Revision 9 1of2

ES-401 Sample Written Examination Form ES-401-5 Question Worrksheet Question Source: Bank #

Modified Bank # (Note changes or attach parent)

New X Question History: Last NRC Exam Question Cognitive Level: Memory or Fundam'ental Knowledge X Comprehension or Analysis 10 CFR Part 55 Content: 55.41 55.43 Comments:

NUREG-1021, Revision 9 2of2

ES-40 1 Sample Written E:xamination Form ES-401-5 Question Woirksheet Examination Outline Cross-reference: Level RO SRO Tier # 1 Group # 1 KIA # 295025 G2.1.7 Importance Rating 3.7 4.4 295025 High Reactor Pressure - 2.1.7 Ability to evaluate plant performance and make operational judgments based on operating characteristics / reactor behavior / and instrument interpretation. (CFR:

43.5 I 45.12 I 45.13)

Proposed Question: # 12 A transient has occurred at full power with the following indications:

06PI-90A, Reactor Pressure A indicates 1041 psig.

06PI-90B, Reactor Pressure B indicates 1042 psig.

Reactor vessel water level is 201 inches Reactor power is 100% on APRM recorders All equipment responded as designed.

Which of the following actions is required for this transient?

A. Insert CRAM rods.

B. Reduce both Recirc pumps to minimum speed.

C. Fully open bypass valves with the Bypass Valve Jack.

D. Lower RPV pressure with the pressure set DECREASE pushbutton.

Proposed Answer: D NUREG-1021, Revision 9 1 Of3

ES-401 Sample Written Eixamination Form ES-401-5 Question Worksheet Explanation (Optional): Per ARP 09-5-1-38 Rx Press Alarm Hi R3

1. Check other RPV pressure instruments in the control room, relay room, or reactor building to confirm possible increasing pressure.
2. Check EHC pressure setpoint; if possible, decrease RPV pressure to a value less than the alarm point by using pressure set DECREASE pushbutton.
3. Check bypass and control valves
4. Check MSlVs and steam line flow indication
5. If RPV pressure indicates normal on other multiple instruments, check the operability of recorder 06LWPR-97 (including the microswitch).

Per AOP-6 R 7 MALFUNCTION OF EHC PREISSURE REGULATOR C. AUTOMATIC ACTIONS

- Rising RPV Pressure - Rising Pressure Set Point or Equivalent

- Backup EHC pressure regulator takes control s3psi higher than initial RPV pressure.

NOTE : Normal pressure at 95% power is 1030 to 1035 psig G. FOLLOWUP ACTIONS Upon restoration of power to loo%, ensure RPV pressure is adjusted to BETWEEN 1038 and 1040 psig.

A. Incorrect - Not required for given pressure, if RPV pressure > I 080 pig this action would be required Per EOP-2. Per AOP-6, immediate actions are to scram on lowering pressure.

B. Incorrect - per AOP-3 step F.2.2 Lower Rx power level by approximately 5 % per RAP-7.3.16.

C. Incorrect - No procedural guidance exists, per ARP 09-5-1-38 R3 decrease RPV pressure to a value the alarm point by using pressure set DECREASE pushbutton.

D. Correct - per AOP-6 R 7 step F.2.1 Verify backup EHC pressure regulator stabilizes RPV pressure (approximately 3 psi > initial pressure).

Technical Reference(s): ARP 09-5-1-38 R3, AOP-6 R 7 (Attach if not previously provided)

Proposed references to be provided to applicants during examination: NONE Learning Objective: LP-AOP EO-1.03.A (As available)

Question Source: Bank #

Modified Bank # 28023 (Note changes or attach parent)

New Question History: Last NRC Exam 9/26/03 Monticello Question Cognitive Level: Memory or Fundamental Knowledge Comprehension or Analysis X 10 CFR Part 55 Content: 55.41 NUREG-1021, Revision 9 20f3

ES-401 Sample Written EIxamination Form ES-401-5 Question Worksheet 55.43 Comments: changed distracter C from Fully open bypass valves with Pressure Regulator override. To Fully open bypass valves with the Pressure Regulator Jack. To ensure correct plant terminology.

NUREG-1021, Revision 9 3Of3

ES-401 Sample Written Eixamination Form ES-401-5 Question Worksheet Examination Outline Cross-reference: Level RO SRO Tier # 1 Group # 1 WA # 295026 EKI .01 Importance Rating 3.0 3.4 295026 Suppression Pool High Water Temperature - EK1. Knowledge of the operational implications of the following concepts as they apply to SUPPRESSION POOL HIGH WATER TEMPERATURE:

(CFR:41.8 to 41 .IO) EKI .01 Pump NPSH Proposed Question: # 13 Following a loss of coolant accident (LOCA), the A Core Spray is in service per EOP-2.

Which of the following sets of conditions assures adequate NPSH for Core Spray?

A. Torus Over Pressure 1 psig Torus Water Temperature 200 OF ACore Spray Flow 3,500 gpm.

B. Torus Over Pressure 5 psig Torus Water Temperature 205 OF.

A Core Spray Flow 4,000 gpm.

C. Torus Over Pressure 7 psig Torus Water Temperature 215 OF A Core Spray Flow 4,500 gpm.

D. Torus Over Pressure 10 psig Torus Water Temperature 230 O F .

ACore Spray Flow 5,000 gpm.

Proposed Answer: B Explanation (Optional): Left Answer at 7 psig due to 2 choices are on a pressure line (5 6110 psig) and 2 are not (1 psig and 7psig).

A. Incorrect - A 5 psig overpressure would be required for the given flow/temperature combination.

B. Correct - The overpressure lines represent the maximum limits of Core Spray flow &

temperatures. If overpressure falls between the lines, the limits are the lower, most conservative line. This choice falls between the 0 psig 8,5 psig lines. The conservative choice requires a torus over pressure of !5 psig to insure NPSH.

C. Incorrect - A 10 psig overpressure would be required for the given flow/temperature combination.

D. Incorrect - The given flow/temperature combination is off the chart. Temperature or flow must be reduced to at least the 10 psig over pressure line to meet the pump NPSH limits.

NUREG-1021, Revision 9 1 Of3

ES-401 Sample Written Examination Form ES-401-5 Question Worksheet Technical Reference(s): EOP-2 & 4 & 11 MIT-301.11 B (Attach if not previously provided)

OP-I 4 Attachment # 4 Proposed references to be provided to applicants during examination: EOPs 2, & 11 OP-14 Attachment # 4 Learning Objective: MIT-301. I 16, EO 1.01 (As available)

Question Source: Bank # 5920- W 2005 W Q # 14 Bank Modified Bank # (Note changes or attach parent)

New Question History: Last NRC Exam 2005 W Question Cognitive Level: Memory or Fundamental Knowledge Comprehension or Analysis X 10 CFR Part 55 Content: 55.41 55.43 NUREG-1021, Revision 9 2 of 3

ES-401 Sample Written Examination Form ES-401-5 Question Worksheet 1 EQP Page 1 of 1 CORE SPRAY NPSW I W D VORTEX LIMITS CS PUMP NPSH LfMST VORTEX L \ m T T o r u s Water Level Greater Than or E q u a l to 9 . 0 7 feet.

NUREG-1021, Revision 9 3 of 3

ES-401 Sample Written Eixamination Form ES-401-5 Question Worksheet Examination Outline Cross-reference: Level RO SRO Tier # 1 Group # 1 WA # 295028 EK2.03 Importance Rating 3.6 3.8 295028 High Drywell Temperature EK2. Knowledge of the interrelations between HIGH D R W E L L TEMPERATURE and the following: (CFR: 41.7 / 45.8) EK2.03 Reactor water level indication Proposed Question: # 14 Given the following conditions:

The plant is at 100% power.

The RPV water level reference leg backfill system is out of service due to CRD supply pipe leak- the pipe is currently tagged out for maintenance.

Feedwater control is in MANUAL.

A loss of drywell cooling causes elevateld drywell temperature and pressure.

Which statement below describes Narrow Range Level instrument response and the reason for this response?

Assume NO operator action.

A. There will be NO change in indicated level because the reference and variable leg densities both increase.

B. There will be NO change in indicated level because the narrow range instruments are density compensated .

C. Indicated level will decrease because of increased reference leg density.

D. Indicated level will increase because of decreased reference leg density.

Proposed Answer: D Explanation (Optional): On elevated DW temperature, the Narrow Range instrument would sensehndicate a level higher than normal. The X dimension (reference leg) is greater than the Y dimension (variable leg). The pressure reduction due to density change is greater than the change in the variable leg causing the indicated level to be greater than actual level.

Additionally, the FW level instrument is density compensated due to variations in RPV water temperature by sensing RPV pressure to compensate for the normal temperature of the reference leg being more dense that the RPV water, the pressure exerted by 1 ft of water in the reference leg would be greater than that of 1 ft of RPV water, with the conditions given in the stem, higher DW temperature this would not offset the density compensation.

NUREG-1021, Revision 9 1of2

ES-40 1 Sample Written Examination Form ES-401-5 Question Worksheet A. Incorrect-The reference leg density is affected more due to length than the variable leg thus the indicated level would be higher than normal.

B. Incorrect- The reference leg density is affected more due to length than the variable leg thus the indicated level would be higher than normal. Additionally, the FW level instrument is density compensated by sensing RPV pressure to compensate for the normal temperature of the reference leg being more dense that the RPV water, the pressure exerted by 1 ft of water in the reference leg would be greater than that of 1 ft of RPV water, with the conditions given in the stem, higher DW temperature this would not offset the density compensation.

C. Incorrect- The reference leg density is affected more due to length than the variable leg thus the indicated level would be higher than normal.

D. Correct - The reference leg density is affected more due to length than the variable leg thus the indicated level would be higher than normal. Additionally, the FW level instrument is density compensated by sensing RPV pressure to compensate for the normal temperature of the reference leg being more dense that the RPV water, the pressure exerted by 1 ft of water in the reference leg would be greater than that of 1 ft of RPV water, with the conditions given in the stem, higher DW temperature this would not offset the density compensation.

Technical Reference(s): OP-27A Rev 10 (Attach if not previously provided)

Proposed references to be provided to applicants during examination: NONE Learning Objective: SDLP-02B EO-1. I 0.E (As available)

Question Source: Bank #

Modified Bank # (Note changes or attach parent)

New X Question History: Last NRC Exam Question Cognitive Level: Memory or Fundamental Knowledge Comprehension or Analysis X 10 CFR Part 55 Content: 55.41 55.43 Comments: Added: The RPV water level reference leg backfill system is out of service due to CRD supply pipe leak- the pipe is currently tagged out for maintenance. Done to eliminate argument that this would off set the reference leg density change from a minor DW temperature rise.

NUREG-1021, Revision 9 2of2

~ ~

ES-40 1 Sample Written Examination Form ES-401-5 Question Worksheet Examination Outline Cross-reference: Level RO SRO Tier # 1 Group # 1 WA # 295030 EK3.06 Importance Rating 3.6 3.8 295030 Low Suppression Pool Water Level EK3. Knowledge of the reasons for the following responses as they apply to LOW SUPPRESSION POOL WATEiR LEVEL: (CFR: 41.5 / 45.6) EK3.06 Reactor SCRAM Proposed Question: # 15 Given the following:

0 The reactor is at 100% power.

0 A Torus level lowering transient is in prolgress.

0 Torus level reaches13.00 feet and contiriues to lower.

0 A manual scram is inserted in accordance with EOP's.

What is the basis for this action?

A. SRV tailpipe vacuum breakers are uncovered, so the containment will be directly pressurized if the SRVs lift as Torus level continues to lower.

B. Tech Specs requires an immediate reactor scram.

C. Limits the energy which may subsequen.tly be discharged into the primary containment.

D. Low pressure ECCS is inoperable and uinavailable due to NPSH and vortex concerns.

Proposed Answer: C Explanation (Optional): Per SAG-2 R6, Inability to maintain torus water level above 10.75 ft represents a clearly degraded plant condition. Entry into the RPV Control guideline will ensure that a reactor Scram is initiated to place the reactor in a lower energy state and permits actions to be initiated which limit the energy which may be subsequently discharged into the primary containment.

NUREG-1021, Revision 9 1of3

ES-40 1 Sample Written Examination Form ES-401-5 Question Worksheet A. Incorrect - SRV tailpipe vacuum breakers are normally uncovered as they relieve DW atmosphere pressure to the SRV tailpipes to prevent after a SRV lift the steam condensation from causing a low pressure condition that draws water up from the Torus into the tailpipe which on a subsequent SRV lift would result in equipment damage.

B. Incorrect - TS has a two hour action to recover level to normal and then a 12 Hour period to be n Mode 3.

C. Correct - Entering EOP-2 assures that, if possible, the Rx is scrammed & shutdown is assured by control rod insertion before RPV depressurization is initiated. The inability to maintain torus water level above 10.75 ft. represents a clearly degraded plant condition.

Entering EOP-2, places the Rx in a lower energy state and permits actions to be initiated which limit the energy which may subsequently be discharged into the primary Cnmt.

Entry into EOP-2 must be explicitly stated because conditions requiring entry into EOP-4 do not necessarily require entry into EOP-2. Therefore, a scram may not yet have been initiated. Directing that EOP-2 be entered, rather than explicitly stating here Initiate a Rx scram, coordinates actions currently being executed if EOP-2 has already been entered. (Note: EOP-2 requires initiating a Rx scram only if one has not previously been initiated.) In addition, entry to EOP-2 must be made because it is through EOP-2 that the transfer to RPV ED is affected.

D. Incorrect - Low pressure ECCS is inoperable and unavailable due to NPSH and vortex concerns is addressed in Torus temperature leg and is associated with the HCTL on torus temperature and RPV pressure relationships.

Technical Reference(s): EOP BASES (Attach if not previously provided)

SAG-2 R6 Proposed references to be provided to applicants during examination: NONE Learning Objective: MIT-301. I 1E EO-4.05 (As available)

Question Source: Bank #

Modified Bank # (Note changes or attach parent)

New X Quest ion History: Last NRC Exam Question Cognitive Level: Memory or Fundamental Knowledge Comprehension or Analysis X 10 CFR Part 55 Content: 55.41 55.43 Comments: Changed from Suppression pool to Torus throughout, also changed from 13.88 feet to a level lower than the entry condition- 13.00 feet.

Proposed answer has been changed due to technically inaccurate: Torus downcomer vent openings are almost uncovered, so the containment could be directly pressurized in the event of NUREG-1021, Revision 9 2 of 3

ES-40 1 Sample Written Examination Form ES-401-5 Question Worksheet a LOCA as Torus level continues to lower Is NlOT correct, these openings are 4 feet below the water surface. EOP provides action for a scram at 10.75 feet to address this issue.

NUREG-?021, Revision 9 3 of 3

ES-401 Sample Written Examination Form ES-401-5 Question Worksheet Examination Outline Cross-reference: Level RO SRO Tier # 1 Graup # 1 W A# 295031 EA1-13 Importance Rating 4.3* 4.3*

295031 Reactor Low Water Level EAI. Ability to operate and/or monitor the following as they apply to REACTOR LOW WATER LEVEL: (CFR: 41.7 / 45.6) EA1.I 3 Reactor water level control Proposed Question: ## 16 The reactor was operating at 33% power when a main turbine trip occurred due to an electrical fault in the main generator.

0 All immediate actions have been taken per AOP-1, Reactor Scram.

Reactor Scram was successful with all rods inserted.

Reactor conditions have stabilized with pressure controlled on the bypass valves automatically.

0 Feedwater Pump A has been tripped and feedwater controls adjusted as normally expected when responding to AOP-1, Reactor Scram.

The SNO is ready to transfer feedwater level control to automatic when the SNO notes reactor level is at 200 inches and stable.

Which of the following is done first for these conditions to establish automatic control?

A. Transfer control from Low Flow Control Valve (34FCV-137) to RFP 6 MGU in manual.

B. Transfer control from Low Flow Control Valve (34FCV-137) in manual to automatic.

C. Transfer control for Feedwater Pump Bfrom manual on the MGU to manual on the MSC.

D. Transfer control for Feedwater Pump B from Single-Element to Three-Element RFP Control.

Proposed Answer: B Explanation (0ptional):The plant has scrammed, operators are trained to take manual control of FW, trip one RFP shut the discharge valves of RFPs and control level via the startup valve.

Stem sets up for this condition. Question asks the candidate what is the correct action to take next per OP-2A. At 30% power, pressure was controlled within the capacity of the Bypass Valves and HPCl & RClC would not have been initiated to complicate level control.

NUREG-1021, Revision 9 1 of2

ES-40 1 Sample Written Examination Form ES-401-5 Question Worksheet A. Incorrect - Control will be transferred to the low flow valve in automatic with the RFP in automatic - this action results in openirig the RFP Disch 34MOV-100B valve which is beyond the feedwater capacity need for the plant conditions.

B. Correct - The Low Flow Control Valve (34FCV-137) is in manual & needs to be in automatic.

C. Incorrect - The control is maintained on the MGU per current plant conditions- this will not support placing controls in automatic.

D. Incorrect - Shifting RFP from Single-Element to Three-Element control is not permitted by procedure as there is a NOTE that 3-Element control is for Rx power above 25%

power.

Technical Reference(s): OP-2A Rev 59 (Attach if not previously provided)

AOP-1 Rev43 Proposed references to be provided to applicants during examination: NONE Learning Objective: LP-AOP EO-1.03.A (As available)

Question Source: Bank #

~~

Modified Bank # 23495 (Note changes or attach parent)

New Question History: Last NRC Exam Question Cognitive Level: Memory or Fundamental Knowledge Comprehension or Analysis X 10 CFR Part 55 Content: 55.41 55.43 Comments: JAF does not have setpoint setdown. Modified the question to reflect what action is required to control water level post scram following normal operator actions of taking RFPs to manual, tripping one of them and closing the disch valves to place control on the SU valve.

Changed original conditions to lower power to avoid HPCI & RCIC starts and high pressure condition fro turbine trip.

NUREG-1021, Revision 9 2of2

ES-401 Sample Written Examination Form ES-401-5 Question Worksheet Examination Outline Cross-reference: Level RO SRO Tieir # 1 Group # 1 KIA # 295037 EA2.07 Importance Rating 4.0 4.2*

295037 SCRAM Condition Present and R :tor Power Above APRM Downscale or Unknown - EA2.

Ability to determine and/or interpret the following as they apply to SCRAM CONDITION PRESENT AND REACTOR POWER ABOVE APRM DOWNSCALE OR UNKNOWN: (CFR: 41 . I O / 43.5 / 45.1 3)

EA2.07 Containment conditions/isolations Proposed Question: # 17 EOP-3, FAILURE TO SCRAM has been entered.

Plant conditions are as follows:

0 The plant is at 14% power with two SRVs OPEN.

0 Reactor water level is 75 inches.

Reactor pressure is 800 psig.

0 Torus water temperature is 115 OF.

0 Torus water level is 16 ft.

Which of the following combined conditions require boron injection?

A. Reactor Power and Torus Temperature.

6. Reactor Power and Torus Level.

C. Torus Temperature and RPV Pressure.

D. RPV Pressure and Torus Water Level.

Proposed Answer: A Explanation (Optional):

A. Correct Response - Rx power & torus temperature are used for Boron Injection Initiation Temperature (BITT) per EOP-1It R1 B. Incorrect - These parameters are not used for boron injection criteria.

C. Incorrect - These parameters are not used for boron injection criteria.

D. Incorrect - These parameters are not used for boron injection criteria.

Technical Reference(s): EOP-11 R 1 (Attach if not previously provided)

EOP-3 R 8 Proposed references to be provided to applicants during examination: NONE NUREG-1021, Revision 9 1 of2

ES-40 1 Sample Written Examination Form ES-401-5 Question Worksheet Learning Objective: MIT 301. I1D EO-1.06 (As available)

Question Source: Bank #

Modified Bank # 26940 (Note changes or attach parent)

New Question History: Last NRC Exam Monticello 9/26/03 Question Cognitive Level: Memory or Fundamental Knowledge X Comprehension or Analysis 10 CFR Part 55 Content: 55.41 55.43 Comments:

The following Plant conditions do not require trends to support answering the question:

0 The plant is at 14% power with two SRVs OPEN. (power is 14% acceptable w/o trend) 0 Reactor water level is 75 inches. (no trend required) 0 Reactor pressure is 800 psig. (no trend required) 0 Torus water temperature is 115 OF. (no trend required, stem provides enough information to make determination that it is rising, 2 SRVs open which is beyond capacity of full torus cooling) 0 Torus water level is 16 ft. (no trend required, stem provides enough information to make determination that it is rising, 2 SRVs open adding inventory).

NUREG-1021, Revision 9 2of2

ES-40 1 Sample Written Examination Form ES-401-5 Question Worksheet Examination OutIine Cross-reference: Level RO SRO Tierr # 1 Group # 1 WA. # 295038 G2.4.18 lmplortance Rating 2.7 3.6 295038 High Off-Site Release Rate -2.4.18 Knowledge of the specific bases for EOPs. (CFR: 41.10 /

45.13)

Proposed Question: # 18 A Torus pressure or temperature condition which cannot be maintained within which ONE of the following limits would require the Torus to be vented IRRESPECTIVE of the Offsite Radioactivity Release Rate?

A. Heat Capacity Temperature Limit B. Pressure Suppression Pressure C. Primary Containment Pressure Limit D. SRV Tail Pipe Level Limit Proposed Answer: C Explanation (Optional): Per EOP-4 R7 Before Torus pressure reaches the PCPL (vent); Vent the primary containment to control pressure below the PCPL. Defeat interlocks & exceed offsite radioactivity release rate limits if necessary (EP-6).

A. Incorrect - Per EOP-4 R7 If Torus temperature & RPV pressure cannot be maintained below the HCTL, Then Emergency RPV depressurization is required.

B. Incorrect - Per EOP-4 R7 WAIT until torus pressure cannot be maintained below the PSP -Emergency RPV depressurization is required.

C. Correct Response - Per EOP-4 R7 Before Torus pressure reaches the PCPL (vent);

Vent the primary containment to control pressure below the PCPL. Defeat interlocks &

exceed offsite radioactivity release rate limits if necessary (EP-6).

D. Incorrect - Per EOP-4 R7 Torus water level & RPV pressure cannot be maintained below the SRV Tail Pipe Level Limit IF the core will still be adequately cooled, THEN terminate injection into the RPV from sources external to the primary containment, except SLC or CRD when required to shutdown the Rx. Torus water level & RPV pressure cannot be restored & maintained below the SRV Tail Pipe Level Limit- Emergency RPV depressurization is required.

Technical Reference(s): EOP-4 R7 (Attach if not previously provided)

Proposed references to be provided to applicants during examination: NONE NUREG-1021, Revision 9 1 of2

ES-401 Sample Written Examination Form ES-40 1-5 Question Worksheet Learning Objective: MIT-301.I 1E EO-41.07 (As available)

Question Source: Bank # 19697 Modified Bank # (Note changes or attach parent)

New Question History: Last NRC Exam 6/14/01 Fermi Unit #

2 Question Cognitive Level: Memory or Fundamental Knowledge Comprehension or Analysis 10 CFR Part 55 Content: 55.41 55.43 Comments:

NUREG-1021, Revision 9 2of2

ES-401 Sample Written Examination Form ES-401-5 Question Worksheet Examination Outline Cross-reference: Level RO SRO Tier # 1 Grcup # 1 WA # 600000 AKI .02 Importance Rating 2.9 3.1 600000 Plant Fire On Site - AKI Knowledge of the operation applications of the following concepts as they apply to Plant Fire On Site: AKI .02 Fire Fighting Proposed Question: # 19 The plant is operating at normal 100% power when a fire occurs in the "A" Emergency Diesel Generator room. The Fire Brigade leader has said it is a serious fire and off-site Fire Department support is required.

In addition to AOP-28, Operation During Plant Fires, what other AOP shall be entered?

A. AOP-43 Plant Shutdown From Outside The Control Room B. AOP-17 LOSSOf 10400 BUS C. AOP-19 LOSSOf 10600 BUS D. AOP-1 Reactor Scram Proposed Answer: D Explanation (Optional):

A. Incorrect - Only required in fires of specific electrical rooms (not the EDG room)

B. Incorrect - Bus is associated with the B/D EDGs C. Incorrect - Bus is associated with the B/D EDGs D. Correct - AOP 28 says the SM shall, on a serious fire, enter AOP-1 or the EOPs (no applicable EOP entry conditions in stern).

Technical Reference(s): AOP-28 R18 (Attach if not previously provided)

Proposed references to be provided to applicants during examination: NONE Learning Objective: LP-AOP EO-1.06 (As available)

Question Source: Bank # # 5728 Modified Bank # (Note changes or attach parent)

New X NUREG-1021, Revision 9 1 of2

ES-401 Sample Written Examination Form ES-401-5 Question Worksheet Question History: Last NRC Exam Question Cognitive Level: Memory or Fundamental Knowledge X Comprehension or Analysis 10 CFR Part 55 Content: 55.41 55.43 Comments: An appropriate JAF reference could not be found for the original selected question so it was replaced with the above. For the K&A, the operations application of fire fighting to a onsite plant fire is when the Fire Brigade Leader declares the fire serious.

Changed question from proposed:

An electrical fire is burning in MCC-XXM.

The fire brigade wants to use water to fight the .fire in MCC-XXXX.

In accordance with AOP-28, - the MCC-XXX .?

A. MUST be deenergized and the Fire Brigade Leaders permission is required to use water on an electrical fire.

B. SHOULD be deenergized and the Fire Blrigade Leaders permission is required to use water on an electrical fire.

C. MUST be deenergized and the Shift Managers permission is required to use water on an electrical fire.

D. SHOULD be deenergized if possible, and the Shift Managers permission is required to use water on an electrical fire.

NUREG-1021, Revision 9 2of2

ES-401 Sample Written Examination Form ES-401-5 Question Worksheet Examination Outline Cross-reference: Level RO SRO Tier # I Group # 1 KIA # 295018 AK2.01 Importance Rating 3.3 295018 Knowledge of the interrelations between PARTIAL OR COMPLETE LOSS OF COMPONENT COOLING WATER and the following: (CFR: 41.7 / 45.8) AK2.01 System loads.

Proposed Question: # 20 The reactor is operating at 100% power.

Annunciator 09-6-2-31 RBC HDR PRESS LO is, received. Low RBCLC system pressure is verified and the standby pump will NOT start.

Which ONE of the following will be AUTOMATICALLY supplied with cooling water if RBCLC header pressure continues to lower?

A. DW ventilation coolers B. PASS system cooler C. DW Equipment Drain Sump cooler D. Reactor Recirc pump and motor coolers Proposed Answer: B Explanation (Optional):

Technical Reference(s): (Attach if not previously provided)

Proposed references to be provided to applicants during examination:

Learning Objective: (As available)

Question Source: Bank #

Modified Bank # (Note changes or attach parent)

NUREG-1021, Revision 9 1of2

ES-401 Sample Written Examination Form ES-401-5 Question Worksheet Question History: Last NRC Exam Question Cognitive Level: Memory or Fundamental Knowledge X Comprehension or Analysis 10 CFR Part 55 Content: 55.41 55.43 Comments:

NUREG-1021, Revision 9 2of2

ES-401 Sample Written IExamination Form ES-401-5 Question Worksheet Examination Outline Cross-reference: Level RO SRO Tier # 1 Group # 2 WA. # 295009 AA2.03 Importance Rating 2.9 2.9 295009 Low Reactor Water Level AA2. Ability to determine and/or interpret the following as they apply to LOW REACTOR WATER LEVEL: (CFR: 41 .IO I43.5 I45.13) AA2.03 Reactor water cleanup blowdown rate Proposed Question: # 21 A plant startup is in progress per OP-65 Startup and Shutdown procedure.

The following conditions exist:

Power is in the IRM range with coolant temperature approaching 212 OF.

The ATC is in control of withdrawing control rods.

The SN02 is controlling reactor water level.

The SNO is preparing the secondary plaint for the introduction of steam.

The A CRD pump is running.

Annunciator 09-5-1-28, Rx Wtr Lvl Alarm Hi or Lo is received.

Water level on panel 09-5 is 193 inches Per OP-65, what actions are needed to restore reactor water level to the normal band?

A. Start a second CRD pump and maximize flow.

B. Throttle closed on RWCU blowdown flow.

C. Reduce CRD pump flow to minimum.

D. Throttle closed on FW Start-up Valve - 34FCV-137.

Proposed Answer: 6 Explanation (Optional): Stem provides low level alarm comes in at 196.5 inches, to correct the condition it is necessary to raise level. OP-65 provides two options at this point: RWCU blowdown or FW start-up valve (34FCV-137) operation, adding more water will affect core reactivity by adding positive reactivity.

A. Incorrect - Will cause level to rise- but will affect control rod drive speeds & will affect core reactivity by adding positive reactivity also not directed by OP-65.

B. Correct- will reduce inventory loss, once > 212 F steam loads will contribute to loss which can be compensated by educing blowdown flow- it is directed by OP-65.

C. Incorrect -Will cause level to lower- but will affect control rod drive speeds- level needs to be raised, NOT directed by OP-65 D. Incorrect - Will cause level to rise- is one of choices allowed by OP-65 for level control, but direction should be to throttle down.

NUREG-1021. Revision 9 1 of2

ES-401 Sample Written IExamination Form ES-401-5 Question Worksheet Technical Reference(s): OP-65 Rev 106 (Attach if not previously provided)

ARP 09-5-1-28 Rev 4 OP-28 Rev 71 Proposed references to be provided to applicants during examination: NONE Learning Objective: LP-OP65A EO-1. I 4.B (As available)

Question Source: Bank #

Modified Bank # (Note changes or attach parent)

New X Question History: Last NRC Exam Question Cognitive Level: Memory or Fundamental Knowledge Comprehension or Analysis X 10 CFR,Part 55 Content: 55.41 55.43 Comments: Changed distracter D to one of the possible choices per OP-65. The previous distracter proposed was increase CRD pump flow to maximum with one CRD pump, this change makes it so that CRD was not used in all three distracters.

NUREG-1021, Revision 9 2of2

ES-40 1 Sample Written Examination Form ES-40 1-5 Question Worksheet Examination Outline Cross-reference: Level RO SRO Tier # 1 Group # 2 WA # 295013 AK2.01 Importance Rating 3.6 3.7 295013 High Suppression Pool Temperature Knowledge of the interrelations between HIGH SUPPRESSION POOL TEMPERATURE and the following: (CFR: 41.7 / 45.8) AK2.01 Suppression pool cooling Proposed Question: # 22 A steam line break has occurred in the Primary Containment, several control rods failed to fully insert with the following:

e RPV level is 200 inches.

e RPV pressure is 420 psig.

e Drywell pressure is 4.5 psig.

e Drywell temperature is 180 OF.

e Torus pressure is 2.5 psig.

e Torus temperature is 106 O F .

Which one of the following residual heat removal system lineups is required?

A. RHR Loop A in suppression pool cooling, RHR Loop B in Drywell spray.

6. RHR Loop A & B in Torus spray.

C. RHR Loop A & B in suppression pool cooling.

D. RHR Loop A in Torus spray, RHR Loop lt3 in Drywell spray.

Proposed Answer: C Explanation (Optional): EOP-4 requires operating all available RHR pumps for torus cooling that are not needed to ensure adequate core cooling. Per the stem level is 200 inches = adequate core cooling via submergence. Torus spray is required to be initiated prior to 15 psig but is currently low, starting at a low pressure will challenge the containment by taking longer to secure sprays and pressure lowering while this is happening, with the lower pressure of RPV at 420 psig and a steam break pressure will continue to rise in the Drywell, spraying the Torus will not control the Torus pressure at this condition as it is being compressed by the elevated DW pressure from the break. Spraying the DW is not required till DW pressure is 15 psig, currently it is below 15 psig.

NUREG-1021, Revision 9 1 of2

ES-401 Sample Written Examinatio n Form ES-401-5 Question Worksheet A. Incorrect - Spraying the DW is not required till DW pressure is 15 psig, currently it is below 15 psig.

6. Incorrect - Torus spray is required to be initiated prior to 15 psig but is currently low, starting at a low pressure will challenge the containment, with the lower pressure of RPV at 420 psig and a steam break pressure will continue to rise in the Drywell, spraying the Torus will not control the Torus pressur'e at this condition as it is being compressed by the elevated DW pressure from the break.

C. Correct- EOP-4 requires operating all available RHR pumps for torus cooling that are not needed to ensure adequate core cooling. Per the stem level is 200 inches =

adequate core cooling via submergence.

D. Incorrect - Torus spray is required to be initiated prior to 15 psig but is currently low, starting at a low pressure will challenge the containment, with the lower pressure of RPV at 420 psig and a steam break pressure will continue to rise in the Drywell, spraying the Torus will not control the Torus pressure at this condition as it is being compressed by the elevated DW pressure from the break. Spraying the DW is not required till DW pressure is 15 psig, currently it is below 15 psig.

Technical Reference(s): EOP-4 Rev 7 (Attach if not previously provided)

Proposed references to be provided to applicants during examination: EOP-4 (w/o entry conditions)

Learning Objective: MIT-301.I 1E EO-4.03 (As available)

Question Source: Bank #

Modified Bank # 22265 (Note changes or attach parent)

New Question History: Last NRC Exam Question Cognitive Level: Memory or Fundamental Knowledge Comprehension or Analysis X 10 CFR Part 55 Content: 55.41 55.43 Comments: Changed Torus pressure to 5.0 psig as at 2.0 it is not possible unless a failure is assumed.

NUREG-1021, Revision 9 2of2

ES-40 1 Sample Written IExamination Form ES-401-5 Question Worksheet Examination Outline Cross-reference: Level RO SRO Tier # 1 Grciup # 2 KIA # 295017 AA2.04 Importance Rating 3.6 4.3*

295017 High Off-Site Release Rate AA2. Ability to dietermine and/or interpret the following as they apply to HIGH OFF-SITE RELEASE RATE: (CFR: 41 . I O / 43.5 / 45.13) AA2.04 tSource of off-site release Proposed Question: # 23 The plant has experienced a radiological accident condition.

The following are currently alarmed:

09-3-2-29 RX BLDG VENT RAD MON Ill 09-3-1-40 RX BLDG ARM RAD HI The Area of the radiation release is from the:

A. Post Accident Sample System exhaust B. RWR MG Room ventilation exhaust C. Standby Gas Treatment System exhaust D. Reactor Building Ventilation exhaust Proposed Answer: D Explanation (Optional): 09-3-1-40 RX BLDG ARM RAD HI provides direction to isolate control and relay room ventilation. 09-3-2-29 RX BLDG VENT RAD MON HI -This alarm is at 5 X I O 3 cpm which is below the alert level of 9.9E5 cpm. With the two alarms in the Rx Bldg the source is the Rx Bldg. NO entry into EOP-6 is not entered due to levels are below the Alert level. RX BLDG Exhaust is below the HI HI level so that SGT will not be running refer to ARP 09-3-2-40 RX BLDG VENT RAD MON HI HI RWR MG SET ventilation exhaust is not monitored and discharges to the roof. PASS ties to Turbine building Ventilation so none of the listed alarms would be consistent with a radiation problem in the PASS area.

A. Incorrect - PASS ties to Turbine building Ventilation so none of the listed alarms would be consistent with a radiation problem in the PASS area.

B. Incorrect - RWR MG SET ventilation exhaust is not monitored and discharges to the roof. NO entry into EOP-6 is not entered (dueto levels are below the Alert level.

C. Incorrect - RX BLDG Exhaust is below the HI HI level so that SGT will not be running refer to ARP 09-3-2-40 RX BLDG VENT RAD MON HI HI for initiation of RX VENT ISOL

& SGT Start. NO entry into EOP-6 is not entered due to levels are below the Alert level.

D. Correct - With the two alarms in the Rx Bldg the source is the Rx Bldg. Levels are below the alert level so EOP-6 is Not required.

NUREG-1021, Revision 9 1of2

ES-401 Sample Written Examination Form ES-401-5 Question Worksheet Technical Reference(s): ARP-09-3-2-29 Rev 6 (Attach if not previously provided)

ARP-09-3-1-40 Rev 8 IAP-2 Rev 27 Attrnt # IAP-2.1 EOP-6 Rev 7 Proposed references to be provided to applicants during examination: NONE Learning Objective: SDLP-17 EO-1.14 (As available)

Question Source: Bank #

Modified Bank # 5683 (Note changes or attach parent)

New Question History: Last NRC Exam Question Cognitive Level: Memory or Fundamental Knowledge X Comprehension or Analysis 10 CFR Part 55 Content: 55.41 55.43 Comments:

NUREG-I 021, Revision 9 2 of 2

ES-40 1 Sample Written Examination Form ES-401-5 Question Worksheet Examination Outline Cross-reference: Level RO SRO Tier # 1 Group # 2 KIA, # 295022 AKI .02 Importance Rating 3.6 3.7 295022 Loss of CRD Pumps AK1 . Knowledge of t h e operational implications of the following concepts as they apply to LOSS OF CRD PUMPS: (CFR: 41.8 to 41 .IO) AKI .02 Reactivity control Proposed Question: # 24 The plant is in a startup at 29% power with all systems normal.

Subsequently the following occurs:

0 at 0800 the ACRD pump trips 0 at 0805 annunciator 09-5-1-43 CRD ACCUM PRESS LO OR LVL HI is in alarm and two yellow ACCUM lights are lit on the full core display 0 at 0810 the B CRD pump is started Upon re-adjusting drive water pressure, the drive water differential pressure is 350 psid.

When the accumulator alarms are cleared IAW OP-25 Section G, the startup is resumed.

What is an operational implication of the drive water differential pressure remaining at 350 psid?

A. Double notching of ANY control rod may occur B. Double notching of ONLY the two control rods that had accumulator alarms may occur C. The coupling check for ANY rods will indicate an uncoupled rod D. The coupling check for ONLY the two control rods which had accumulator alarms will indicate an uncoupled rod Proposed Answer: A Explanation (Optional):

Technical Reference(s): OP-25 (Attach if not previously provided)

AOP-69 Rev 7 Proposed references to be provided to applicants during examination: NONE Learning Objective: LP-AOP EO-1.03 (As available)

NUREG-1021, Revision 9 1of2

ES-401 Sample Written Examination Form ES-401-5 Question Worksheet Question Source: Bank #

Modified Bank # (Note changes or attach parent)

New X Question History: Last NRC Exam Question Cognitive Level: Memory or Fundamental Knowledge Comprehension or Analysis X I O CFR Part 55 Content: 55.41 55.43 Comments: proposed choices did not meet the Tech spec actions for conditions provided in the stem.

NUREG-1021, Revision 9 2of2

ES-40 1 Sample Written IExamination Form ES-401-5 Question Worksheet Examination Outline Cross-reference: Level RO SRO Tielr # 1 Group # 2 KIA. # 295032 EK3.01 Importance Rating 3.5 3.8 295032 High Secondary Containment Area Temperature- EK3. Knowledge of the reasons for the following responses as they apply to HIGH SECONDARY CONTAINMENT AREA TEMPERATURE :

(CFR: 41.5 I 45.6) EK3.01 Emergencyhormal depressurization Proposed Question: ## 25 An unisolable leak from the RWCU system is in progress.

All attempts to isolate the leak have been unsuccessful, and the following plant conditions exist:

0 RPV pressure is 800 psig and lowering slowly.

0 RPV level is 150 inches and steady.

0 All injection systems are available with Condensate/ Feedwater in service.

0 All control rods are inserted into the core.

0 A RWCU Pump Room is 230 OF.

0 RWCU Heat Exchanger Room is 205 O F .

0 All other Reactor Building location temperatures are c 103 O F .

What is the required action & N J the reason for conducting a normal OR emergency depressurization (ED)?

A. ED required because one area is above max safe temperature indicating a potential threat to secondary containment integrity.

B. Normal cooldown using the turbine bypass valves is required because the main condenser is available.

C. ED required because more than one area is above the max safe temperature indicating a potential threat to equipment reliability.

D. Normal cooldown using SRVs is required because use of the SRVs will limit the spread of contamination.

Proposed Answer: C Explanation (Optional): A RWCU Pump Room 2130 OF is > max safe 225 OF and RWCU Heat Exchanger Room 205 O F is > max safe 203 OF. With a unisolable RWCU leak per EOP-5 a ED is required vice a normal cooldown.

NUREG-1021, Revision 9 1 of2

ES-401 Sample Written Examination Form ES-401-5 Question Worksheet A. Incorrect - ED is required, however reason is > 2 max safe temperatures per EOP-5.

B. Incorrect - With a unisolable RWCU leak & 2 max safe temperatures per EOP-5 a ED is required vice a normal cooldown.

C. Correct Response - ED is required, reason 2 or more areas above max safe is indication of wide spread problem which may pose a direct and immediate threat to the Rx Bldg integrity, equipment in the secondary containment and continued safe operation of the plant.

D. Incorrect - With a unisolable RWCU leak & 2 max safe temperatures per EOP-5 a ED is required vice a normal cooldown.

Technical Reference(s): EOP-5 Rev 7 (Attach if not previously provided)

EOP Bases Proposed references to be provided to applicanlts during examination: EOP-5 w/o entry conditions.

Learning Objective: MIT-301.I 1F EO-1.07 (As available)

Question Source: Bank #

Modified Bank # (Note changes or attach parent)

New X Quest ion History : Last NRC Exam Question Cognitive Level: Memory or Fundamental Knowledge Comprehension or Analysis X 10 CFR Part 55 Content: 55.41 55.43 Comments:

NUREG-1021, Revision 9 2of2

ES-401 Sample Written Examination Form ES-401-5 Question Worksheet Examination Outline Cross-reference: Level RO SRO Tier #

Group #

KIA # 295034 EA2.01 Importance Rating 295034 Ability to determine andlor interpret the following as they apply to SECONDARY CONTAINMENT VENTILATION HIGH RADIATION: (CFR:41. I O / 43.5 I 45.1 3) EA2.01 Ventilation radiation levels.

Proposed Question: # 26 During movement of spent fuel from the reactor, a fuel bundle is dropped and is damaged.

What auto-actuated design feature(s) ensure 10CFRI00 limits are not exceeded?

A. SBGT actuates on High Stack Radiation B. SBGT actuates on High Radiation in exhaust air from refuel floor.

C. Supply and exhaust Normal Reactor Building Ventilation fans trip on high Radiation in RB exhaust air plenum.

D. Normal Reactor Building Ventilation system isolates on HI Area Radiation Monitor on refuel floor.

Proposed Answer: B Explanation (Optional):

Technical Reference(s): (Attach if not previously provided)

Proposed references to be provided to applicants during examination:

Learning Objective: (As available)

Question Source: Bank #

Modified Bank # (Note changes or attach parent)

New X Question History: Last NRC Exam NUREG-1021, Revision 9 1 of2

ES-40 1 Sample Written Examination Form ES-40 1-5 Question Worksheet Question Cognitive Level: Memory or Fundarnental Knowledge X Comprehension or' Analysis 10 CFR Part 55 Content: 55.41 55.43 Comments:

NUREG-1021, Revision 9 2of2

ES-401 Sample Written Examination Form ES-401-5 Question Worksheet Examination Outline Cross-reference: Level RO SRO Tier # 1 Group ## 2 KIA # 295036 EKI .01 Importance Rating 2.9 295036 Knowledge of the operational implications of the following concepts as they apply to SECONDARY CONTAINMENT HIGH SUMPIAREA WATER LEVEL (CFR: 41.8 to 41 . I O ) EK 1.01 Radiation releases Proposed Question: # 27 The plant is shutdown at 60 psig.

  • Reactor water level is 220 inches and slowly decreasing
  • Drywell pressure is I .O psig and steady
  • Annunciator 09-75-2-6 RX BLDG VENT lSOL is in alarm

Given the above conditions, what is the most important parameterkondition that must be monitored?

A. Reactor Building temperature B. Reactor Building Equipment Drain Sump level C. Reactor pressure D. Reactor Building radiation levels Proposed Answer: D Explanation (Optional):

Technical Reference(s): (Attach if not previously provided)

- ~

Proposed references to be provided to applicant!; during examination:

NUREG-I 021, Revision 9 1of2

ES-401 Sample Written Examination Form ES-401-5 Question Worksheet Learning 0bjective: (As available)

Question Source: Bank ##

Modified Bank # (Note changes or attach parent)

New X Question History: Last NRC Exam Question Cognitive Level: Memory or Fundamental Knowledge Comprehension or Analysis X 10 CFR Part 55 Content: 55.41 55.43 Comments:

NUREG-1021, Revision 9 2of2

ES-401 Sample Written Examination Form ES-401-5 Question Worksheet Examination Outline Cross-reference: Level RO SRO Tieir # 2 Group # 1 KIA. # 203000 A I .09 Importance Rating 2.9 2.9 203000 RHWLPCI: Injection Mode (Plant Specific) - A I . Ability to predict and/or monitor changes in parameters associated with operating the RHWLPCI: INJECTION MODE (PLANT SPECIFIC) controls including: (CFR: 41.5 I 45.5) A I .09Component cooling water systems Proposed Question: # 28 When operating Loop A RHR in the LPCl mode, RHR heat exchanger A shell outlet temperature is maintained by throttling RHRSW DISCH VLV FROM HX A, 1OMOV-89A to establish  ?

A. > 68 O F , 2500 to 4000 gpm per pump B. 190 O F , 4000 to 6500 gpm per pump C. > 68 O F , 4000 to 6500 gpm per pump D. 190 O F , 2500 to 4000 gpm per pump Proposed Answer: A Explanation (Optional): Per OP-l3A; Step D.1.8 (a G.8.4) Maintain RHR HX A shell outlet temperature > 68 OF using one or a combination of the following methods:

0 Throttle RHRSW DISCH VLV FROM HX A 10MOV-89A to establish 2500 to 4000 gpm per pump.

NOTE THAT Per OP-I 3C; Attachment # 1, Step 5. Throttle RHRSW DlSCH VLV FROM HX A(B) IOMOV-89A(B) to establish 2500 to 4000 gpm per RHRSW pump.

Step 8. WHILE performing the following step, maintain torus water temperature (NOT RHR HX A shell outlet temperature) 2 TO 72 OF ON EPIC-A-3380 (SPWT).

9. Control containment heat removal rate using one or a combination of the following methods:

0 Throttle RHRSW DISCH VLV FROM HX A(B) IOMOV-89A(B).

NUREG-1021 Revision 9 1 of2

ES-401 Sample Written Examination Form ES-401-5 Question Worksheet A. Correct Response - Per OP-13A Step D. 1.8 (& G.8.4) Maintain RHR HX A shell outlet temperature > 68 OF using one or a combination of the following methods: Throttle 1OMOV-89A to establish 2500 to 4000 gpm per pump B. Incorrect - No high temperature limit for the HX shell outlet provided Per OP-I 3A. Vortex limit of 190 OF is based on Torus temperature, However, directions are to Throttle 10MOV-89A to establish 2500 to 4000 gpm per pump.

C. Incorrect - Per OP-13A Step D.1.8 (& G.8.4) Maintain RHR HX A shell outlet temperature

> 68 O F is correct, however, directions are to Throttle 1OMOV-89A to establish 2500 to 4000 gpm per pump.

D. Incorrect - No high temperature limit for the HX shell outlet provided Per OP-I 3A. Vortex limit of 190 OF is based on Torus temperature, However, directions are to Throttle 1OMOV-89A to establish 2500 to 4000 gpm per pump.

Technical Reference(s): OP-I 3A R 14 (Attach if not previously provided)

OP-13C R 9 Proposed references to be provided to applicants during examination: NONE Learning Objective: SDLP-10 EO-1.13.A (As available)

Question Source: Bank #

Modified Bank # (Note changes or attach parent)

New X Question History: Last NRC Exam Question Cognitive Level: Memory or Fundamental Knowledge X Comprehension or Analysis 10 CFR Part 55 Content: 55.41 55.43 Comments: Moved throttling RHRSW DlSCH VLV FROM HX A, 1OMOV-89A to establish to stem as it was part of all answers, NUREG-1021, Revision 9 2 of 2

ES-401 Sample Written Examination Form ES-401-5 Question Worksheet Examination Outline Cross-reference: Level RO SRO Tieir # 2 Group # 1 KIA, # 205000 A2.03 Importance Rating 3.2 3.2 205000 Shutdown Cooling System (RHR Shutdown Cooling Mode) - A2. Ability to (a) predict the impacts of the following on the SHUTDOWN COOLING SYSTEM (RHR SHUTDOWN COOLING MODE); and (b) based on those predictions, use procedures to correct, control, or mitigate the consequences of those abnormal conditions or operations: (CFR: 41.5 / 45.6)

A2.03 A.C. failure Proposed Question: # 29 The plant is in Mode 4 with the following conditions:

B RHR pump is running in shutdown cooling (SDC) mode.

0 Reactor coolant system (RCS) temperature is 180 OF and steady.

Subsequently, a sustained loss of Bus 10600 occurs.

SDC is lost due to (1) and Procedures (2) are used to mitigate the consequences.

fll 121 A. RHR pump loss only AOP-59 Loss of RPS Bus A Power AND AOP-18B B. RHR pump loss only AOP-60 Loss of RPS Bus E3 Power AND AOP-30 Loss of Shutdown Cooling C. RHR pump loss and SDC isolations AOP-60 Loss of RPS Bus B Power AOP-30 Loss of Shutdown Cooling D. RHR pump loss and SDC isolations AOP-59 Loss of RPS Bus A Power @ l AOP-188 Proposed Answer: C NUREG-1021, Revision 9 1 of3

ES-401 Sample Written Examination Form ES-401-5 Question Worksheet Explanation (Optional): Per Loss of 10600 Bus Power, AOP-19 AUTOMATIC ACTIONS Loss of RPS Bus B (AOP-60) results in the following actions: Primary Containment System B Group 2 Isolation (closure of Outboard isolation valves) and RHR pump B is a major load loss from 10600.

Per AOP-30, AUTOMATIC ACTIONS NOTE: Automatic actions occur if loss of shutdown cooling is due to a shutdown cooling isolation. Group I1 isolation of primary containment Per Loss of RPS B Power (AOP-60), AUTOMATIC ACTIONS Closure of valves listed on 0 SHUTDOWN CLG SUCT 1OMOV-17 (only in SDC mode) (outboard valve) BMCC-4 (DC powered valve, will respond to isolation signal and is part of the RHR Pump suction flow path interlock circuit) 0 SHUTDOWN CLG SUCT IOMOV-18 (only in SDC mode) (inboard valve) MCC-156 fed from L-I 5 (AC powered valve from A side 10500 L-gear, will respond to isolation signal and is part of the RHR Pump suction flow path interlock circuit)

A. Incorrect - (1) Answer does NOT address fact that SDC is lost due to isolation due to loss of RPS B. (2) Entry for Loss of RPS B (AOP-60) vice Loss of RPS A (AOP-59) and Entry for Loss of Switchgear L-26 (AOP-19B) vice Loss of Switchgear L-25 (AOP-18B) due to conditions in stem.

B. Incorrect (1) Answer does NOT address fact that SDC is lost due to isolation due to loss of RPS B. ( 2 ) Entry into Loss of RPS B (AOP-60) and Loss of SDC (AOP-30) are both correct procedures for the conditions in the stem.

C. Correct Response - (1) RHR Pump trips due to loss of power and receives a trip signal due to loss of suction flow path from the SDC isolations as a result of the Loss of RPS-B, (2) Sustained loss of 10600 results in loss of RPS B (AOP-60) which leads to isolation of SDC. ARHR is lost upon loss of 10500 requiring entry into Loss of SDC (AOP-30).

D. Incorrect - (1) RHR Pump trips due to loss of power and receives a trip signal due to loss of suction flow path from the SDC isolations as a result of the Loss of RPS-B. ( 2 ) Entry for Loss of RPS B (AOP-60) vice Loss of RPS A (AOP-59) and Entry for Loss of Switchgear L-26 (AOP-19B) vice Loss of Switchgear L.25 (AOP-188) due to conditions in stem.

Technical Reference(s): AOP-30 R 19 AOP-59 R 7 (Attach if not previously provided)

AOP-60 R 5 AOP-18B R 1 Proposed references to be provided to applicants during examination:

Learning 0bjective: LP-AOP EO-1.02 (As available)

Question Source: Bank #

Modified Bank # (Note changes or attach parent)

New X Question History: Last NRC Exam Question Cognitive Level: Memory or Fundamental Knowledge Comprehension or Analysis X NUREG-1021, Revision 9 2 of 3

ES-401 Sample Written Examination Form ES-401-5 Question Worksheet 10 CFR Part 55 Content: 55.41 55.43 Comments: Changed Stem to loss of 10600 Bus due to if the 10500 bus is loss this would result in a loss of power to the inboard isolation valve which is AC powered, the outboard isolation power is DC powered and would stroke close due the actuation of the isolation logic from the loss of RPS bus 6. The loss of 10600 bus will iilso result in the trip of RHR pump B. Distracters were changed to make them more plausible for the stem conditions.

NUREG-1021, Revision 9

ES-40 1 Sample Written Examination Form ES-40 1-5 Question Worksheet Examination Outline Cross-reference: Level RO SRO Tier # 2 Group # 1 KIP, # 206000 A3.08 Importance Rating 3.7 3.6 206000 High Pressure Coolant Injection System - A3. Ability to monitor automatic operations of the HIGH PRESSURE COOLANT INJECTION SYSTEM including: (CFR: 41.7 I45.7) A3.08 Condensate storage tank level: BWR-2,3,4 Proposed Question: # 30 The plant was at 100% power with Condensate Storage Tanks (CSTs) 12A and 12B with 100,000 gallons each.

A small break loss of coolant accident (SBLOG4) and loss of offsite power (LOOP) occurred.

All automatic actions functioned as designed.

Current plant conditions are:

0 Drywell pressure is 20 psig.

e RPV pressure is 1000 psig and slowly lowering.

e HPCl is being utilized for RPV level control at full rated flow.

e RPV level is +30 inches and stable.

0 'A' loop of RHR is in suppression pool cooling.

0 'B' loop of RHR is in drywell spray.

Assuming this condition continued for 1 hour1.157407e-5 days <br />2.777778e-4 hours <br />1.653439e-6 weeks <br />3.805e-7 months <br />, what would the MINIMUM level in the CSTs be?

A. 324 inches B. 150 inches C. 60 inches D. 0 inches Proposed Answer: C Explanation (Optional): Per OP-15, In standby, MPCl flow indicator controller is in AUTO with a setpoint of 4,250 gpm. HPCl pump suction is normally lined up to the CSTs. The lower 100,000 gallons of water in each CST is available only to HPCl & RCIC. The alternate HPCl suction supply is the torus. HPCl pump suction shifts from CSTs to torus when: Low CST level - 59.5" Per ARP-09-3-3-7 R 4, HPCl Lvl Lo CST trip is set at 60 'I.

Per ARP-09-6-2-10 R 5, CST Level Low: 238.19 ", CST Level High: 350.81 "

Suction valves from the torus and the CSTs are DC powered and would still transfer during LOOP.

NUREG-1021, Revision 9 1of2

ES-401 Sam ple Written Examination Form ES-401-5 Question Worksheet A. Incorrect - HPCl suction is normally the CSTs &would be taking suction from the CST till low CST level is achieved. HPCl pumlp suction shifts from the CSTs to the torus upon Low CST level - 60 inches, B. Incorrect - HPCl flowrate is 4250 gpm >: 60 minutes = 255,000 gallons pumped, the Two CSTs added together provide only 200,000 gallons per the stem resulting in a HPCl suction shift to the Torus at approximately 60 inches in the CSTs C. Correct Response - HPCl flowrate is 4250 gpm X 60 minutes = 255,000 gallons pumped, the Two CSTs added together provide only 200,000 gallons per the stem resulting in a HPCl suction shift to the Torus at approximately 60 inches in the CSTs.

D. Incorrect - HPCl flowrate is 4250 gpm X: 60 minutes = 255,000 gallons pumped, the Two CSTs added together provide only 200,000 gallons per the stem resulting in a HPCI suction shift to the Torus at approximately 60 inches in the CSTs.

Technical Reference(s): OP-I 5 R 54 (Attach if not previously provided)

ARP-09-3-3-7 R 4 Proposed references to be provided to applicants during examination: NONE Learning Objective: SDLP-23 EO-1.05.A.1 (As available)

Question Source: Bank #

Modified Bank # (Note changes or attach parent)

New X Question History: Last NRC Exam Question Cognitive Level: Memory or Fundamental Knowledge Comprehension or Analysis X 10 CFR Part 55 Content: 55.41 55.43 Comments:

NUREG-1021, Revision 9 2 of 2

ES-40 1 Sample Written Examination Form ES-401-5 Question Worksheet Examination Outline Cross-reference: Level RO SRO Tieir # 2 Group # 1 KIA # 209001 A4.02 Importance Rating 3.5 3.4 209001 Low Pressure Core Spray System -A4. Ability to manually operate and/or monitor in the control room: (CFR: 41.7 / 45.5 to 45.8) A4.02 Suction valves Proposed Question: # 31 The plant is in Mode 5 and the A core spray loop is being used to fill the reactor cavity from the CSTs per procedure OP-30A.

An I&C functional test error causes Channel A and Channel B drywell pressure instrument to go full scale.

The following alarms are received in the control room:

0 09-3-1-22 Core Spray Sys A Logic Activated.

0 09-3-2-22 Core Spray Sys B Logic Activated.

60 seconds from receiving the alarms, what is the position of the A and B core spray suction valves (14MOV-7A & B)?

14MOV-7A 14MOV-75 A. Closed Closed B. Closed Open C. Open Closed D. Open Open Proposed Answer: B Explanation (Optional): Per OP-30A as stated in the stem the A loop CS suction valvel4MOV-7A is shut with suction aligned to the CST vice the Torus. There are NO automatic functions that will reposition 14MOV-7A & 5,they are operated manually. The normal lineup for 14MOV-7A &

B is open thus 14MOV-7B is open.

NUREG-1021, Revision 9 1 of2

ES-40 1 Sample Written Examination Form ES-401-5 Question Worksheet A. Incorrect - Per OP-30A as stated in the stem the A loop CS suction vaIve14MOV-7A is shut with suction aligned to the CST vice the Torus. There are NO automatic functions that will reposition 14MOV-7A & B, they are operated manually. The normal lineup for 14MOV-7A & B is open thus 14MOV-7B is open B. Correct -Per OP-30A as stated in the stem the A loop CS suction valve14MOV-7A is shut with suction aligned to the CST vice the Torus. There are NO automatic functions that will reposition 14MOV-7A & B, they are operated manually. The normal lineup for 14MOV-7A & B is open thus 14MOV-7E3 is open C. Incorrect - Per OP-30A as stated in the stem the A loop CS suction valvel4MOV-7A is shut with suction aligned to the CST vice the Torus. There are NO automatic functions that will reposition 14MOV-7A & B, they are operated manually. The normal lineup for 14MOV-7A & B is open thus 14MOV-7B is open.

D. Incorrect -Per OP-30A as stated in the stem the A loop CS suction valvel4MOV-7A is shut with suction aligned to the CST vim the Torus. There are NO automatic functions that will reposition 14MOV-7A & B, they are operated manually. The normal lineup for 14MOV-7A 81B is open thus 14MOV-7E; is open.

Technical Reference(s): OP-30A Rev 11 (Attach if not previously provided)

OP-14 Rev 31 Proposed references to be provided to applicants during examination: None Learning Objective: SDLP-14 EO-1,14.A (As available)

Question Source: Bank #

Modified Bank # (Note changes or attach parent)

New X Question History: Last NRC Exam Question Cognitive Level: Memory or Fundamental Knowledge X Comprehension or Pmalysis 10 CFR Part 55 Content: 55.41 55.43 Comments:

NUREG-1021, Revision 9 2of2

ES-40 1 Sample Written Examination Form ES-401-5 Question Worksheet Examination Outline Cross-reference: Level RO SRO Tier # 2 Group # 1 KIA # 21 1000 G2.2.12 Importance Rating 3.0 3.4 21 1000 Standby Liquid Control System 2.2.12 Knowledge of surveillance procedures. (CFR: 41.10/45.13)

Proposed Question: # 32 The plant is operating at 100% power.

The SLC quarterly surveillance test STGHA is in progress with 55 gpm flow on the A system when an ATWS occurs.

What actions, if any, are required in the reactor building to allow control room operators to inject boron with the A pump?

A. Manually reposition valves in accordance with the restoration section of the ST

6. A SLC pump motor breaker must be racked in to allow pump starting from the control room.

C. Locally fire the A SLC squib valve to establish the flow path to the reactor.

D. No actions required. The A SLC pump remains lined up for injection of boron during this surveillance procedure.

Proposed Answer: A Explanation (Optional):

A. Correct - SLC tank suction is isolated, must be unisolated to line- up the flow path. ST-6HA Step 8.1.4 refers to section 9.1 for restoring the line-up for injection- open the tank suction valve.

8. Incorrect - Pump breaker is not racked out during ST-6HA.

C. Incorrect - Squib valves are fired by remote pump start.

D. Incorrect - SLC tank suction is isolated, rmust be unisolated to line- up the flow path.

Technical Reference(s): ST-OGHA Rev 2 (Attach if not previously provided)

Proposed references to be provided to applicants during examination: NONE Learning Objective: SDLP-11 EO-1.13.F (As available)

Question Source: Bank #

NUREG-1021, Revision 9 1of2

ES-40 1 Sample Written Examination Form ES-401-5 Question Worksheet Modified Bank # (Note changes or attach parent)

New X Question History: Last NRC Exam Question Cognitive Level: Memory or Fundamental Knowledge Comprehension or Analysis X 10 CFR Part 55 Content: 55.41 55.43 Comments:

NUREG-1021, Revision 9 2of2

ES-401 Sample Written Examination Form ES-401-5 Question Worksheet Examination Outline Cross-reference: Level RO SRO Tieir # 2 Group # I KIA # 212000 K1.04 Importance Rating 3.4 3.6 212000 Reactor Protection SystemKl . Knowledge of the physical connections andlor causeleffect relationships between REACTOR PROTECTION SYSTEM and the following: (CFR: 41.2 to 41.9 / 45.7 to 45.8) K1.04A.C. electrical distribution Proposed Question: # 33 The reactor is operating at 100% power with the electrical distribution system in the normal configuration when a loss of MCC 251 occurs.

What effect, if any, does this have on the scram1 solenoids 10 seconds after the loss of the MCC?

A. The A scram solenoids are deenergized.

B. The B scram solenoids are deenergizeid.

C. Both the A and B scram solenoids are deenergized.

D. Both the A and B scram solenoids rerriain energized.

Proposed Answer: A Explanation (Optional): Per ARP-09-5-1-05 a loss of the normal power MCC-251 will result in a RPS A trip (1/2 scram).

A. Correct- Per ARP-09-5-1-05 a loss of the normal power MCC-251 will result in a RPS A trip (112 scram).

B. Incorrect- RPS 6 is still powered from its MCC.

C. Incorrect- Per ARP-09-5-1-05 a loss of the normal power MCC-251 will result in a RPS A trip (1/2 scram). RPS B is still powered from its MCC.

D. Incorrect- Per ARP-09-5-1-05 a loss of the normal power MCC-251 will result in a RPS A trip (1/2 scram). RPS B is still powered from its MCC.

Technical Reference(s): ARP-09-5-1-05 Rev 7 (Attach if not previously provided)

Proposed references to be provided to applicants during examination: NONE Learning 0bjective: SDLP-05 EO-1.03 (As available)

Question Source: Bank #

NUREG-1021, Revision 9 1 of2

ES-401 Sample Written Examination Form ES-401-5 Question Worksheet Modified Bank # (Note changes or attach parent)

New X Question History: Last NRC Exam Question Cognitive Level: Memory or Fundamental Knowledge X Comprehension or Analysis 10 CFR Part 55 Content: 55.41 55.43 Comments :

NUREG-1021, Revision 9 2of2

ES-40 1 Sample Written Iixamination Form ES-401-5 Question Worksheet Examination Outline Cross-reference: Level RO SRO Tieir # 2 Grciup # 1 KIA. # 215003 K1.01 Implortance Rating 3.9 3.9 215003 Intermediate Range Monitor (IRM) System .. K1. Knowledge of the physical connections andlor cause effect relationships between INTERMEDIATE RANGE MONITOR (IRM) SYSTEM and the following: (CFR: 41.2 to 41.9 145.7 to 45.8) K1.01 R.PS Proposed Question: # 34 The following conditions exist during a plant startup:

0 Reactor Mode Switch is in the Startup position.

0 IRM G BYPASSED and its instrument drawer selector switch is in the STANDBY position.

0 All other IRMs are on Range 4.

Which ONE of the following conditions correctly describes AUTOMATIC plant response if IRM G is taken out of BYPASS?

A. RPS A & B Full Scram.

B. Rod Withdrawal Block and RPS A Half Scram.

C. Rod Withdrawal Block and RPS 6 Half Scram.

D. Rod Withdrawal Block and NO Half Scr(am.

Proposed Answer: B Explanation (Optional): IRM G is assigned to RFS A and a IRM INOP will occur in startup if the drawer is in standby and the Bypass switch is placed out of Bypass Per ARP 09-5-1-41 R 2 NEUTRON MON SYS Trip a. With mode switch NOT in RUN, the following will cause a trip of a RPS trip channel. 2. IRM INOP AUTOMATIC ACTIONS 1. Half Scram for either RPS trip channel. Companion Instruments APRM E - IRM G

1. RPS trip channel A relays 5A-K12A, C, E, G Per ARP 09-5-2-2 R 4, Rod Withdrawal Block, CAUSES Reactor Mode Switch in Startup/Hot Standby or Refuel: - iRM inoperative Per ARP-09-5-2-52 R 4 IRM TRIP SYS A INOP OR UPSCALE TRIP - DEVICE IRM A, C, E, or G AUTOMATIC ACTIONS - Mode switch NOT in RUN: Rod block AND RPS A half scram CAUSES - Instrument malfunction (inop):- IRM mode switch not in operate A. Incorrect - Per ARP-09-5-2-52 Rod Withdrawal Block & RPS A Half Scram B. Correct - Per ARP-09-5-2-52 Rod Withdrawal Block & RPS A Half Scram C. Incorrect - Per ARP-09-5-2-52 Rod Withdrawal Block & RPS A Half Scram D. Incorrect - Per ARP-09-5-2-52 Rod Withldrawal Block & RPS A Half Scram NUREG-1021. Revision 9 1 of2

ES-40 1 Sample Written Examination Form ES-401-5 Question Worksheet Technical Reference(s): ARP-09-5-2-52 R 4 (Attach if not previously provided)

ARP 09-5-1-41 R 2 ARP 09-5-2-2 R 4 Proposed references to be provided to applicarits during examination: NONE Learning Objective: SDLP-07B EO-1.09.B (As available)

Question Source: Bank #

Modified Bank # (Note changes or attach parent)

New X Question History: Last NRC Exam Question Cognitive Level: Memory or Fundamental Knowledge X Comprehension or Analysis 10 CFR Part 55 Content: 55.41 55.43 Comments: Changed the listed KA From 215003 K1.05 to 215003 K1.01 to reflect the sample plan.

NUREG-1021, Revision 9 2 of 2

ES-401 Sample Written Eixamination Form ES-401-5 Question Worksheet Examination Outline Cross-reference: Level RO SRO Tier # 2 Group # 1 WA # 215004 K3.02 Importance Rating 3.4 3.4 215004 Source Range Monitor (SRM) System K3. Knowledge of the effect that a loss or malfunction of the SOURCE RANGE MONITOR (SRM) SYSTEM will have on following: (CFR: 41.7 I45.4) K3.02 Reactor manual control: Plant-Specific Proposed Question: # 35 Reactor startup is in progress per OP-65. The reactor is NOT critical.

SRMs are all operable and read as follows:

Channel -

A -

B -C -

D Counts per second 2 X I O 3 3 X IO3 2x io3 5 X IO3 Predict the effect of a loss of SRM C High voltage Power supply, AND what would be the necessary action to continue the startup?

EFFECT NECESSARY ACTION A. Half Scram Suspend startup until repairs are complete.

B. Rod Block Suspend startup until repairs are complete.

C. Half Scram Bypass affected channel and continue startup.

D. Rod Block Bypass affected channel and continue startup.

Proposed Answer: D Explanation (Optional): SRMs provide no scrarri signal in normal configuration as stated in the stem. A SRM will go INOP on a loss of high voltage and will cause a rod block only for conditions in the stem.

A. Incorrect - SRMs provide no scram signal in normal configuration.

B. Incorrect - SRM will go INOP on a loss of high voltage & will cause a rod block, SRM can be bypassed per OP-16 at which tirne the startup may continue.

C. Incorrect - SRMs provide no scram signal in normal configuration.

D. Correct - SRMs provide no scram signal in normal configuration. SRM will go INOP on a loss of high voltage & will cause a rod block only, SRM can be bypassed per OP-16 at which time the startup may continue Technical Reference(s): OP-I 6 Rev 27 (Attach if not previously provided)

ARP 09-5-2-2 Rev 4 NUREG-1021, Revision 9 1 of2

ES-401 Sa mpI e Written Examination Form ES-401-5 Question Worksheet ARP 09-5-2-51 Rev 4 Proposed references to be provided to applicants during examination: NONE Learning Objective: SDLP-07B EO-1.14.C (As available)

Question Source: Bank #

Modified Bank # (Note changes or attach parent)

New X Question History: Last NRC Exam Question Cognitive Level: Memory or Fundamental Knowledge X Comprehension or Analysis 10 CFR Part 55 Content: 55.41 55.43 Cornments:

NUREG-1021, Revision 9 2of2

ES-401 Sample Written Examination Form ES-401-5 Question Worksheet Examination Outline Cross-reference: Level RO SRO Tier # 2 Group # 1 KIA # 215005 K4.07 Importance Rating 3.7 3.7 215005 Average Power Range Monitor/Local Power Range Monitor System - K4. Knowledge of AVERAGE POWER RANGE MONITOWLOCAL POWER RANGE MONITOR SYSTEM design feature(s) and/or interlocks which provide for the following: (CFR:41.7) K4.07 Flow biased trip setpoints Proposed Question: # 36 The following plant conditions exist:

0 Reactor power - 75%.

0 Total Recirculation drive loop flow - 70% (22.1 Mlbm/hr).

0 Recirculation loops in operation - Both.

Which ONE of the following is the APRM Upscale Thermal Power Trip Setpoint?

A. 120.0 %

B. 117.0 Yo C. 111.0 Yo D. 109.0 %

Proposed Answer: B NUREG-1021, Revision 9 1 Of3

ES-401 Sample Written Examination Form ES-401-5 Question Worksheet Explanation (Optional): Per COLR R 23 7.4.1.2. When operating in Mode 1, the APRM Neutron Flux-Hlgh (Flow Biased)

Allowable Value shall be for two loop operation:

Sr (% RTP) = 117.00% (Clamp) W > 68.7?10 With Core Flow 70% (wd > 68.7%) the APRM Upscale Thermal Power Trip is clamped at 117 % per the following ARP 09-5-2-54 136:

ANXIUNCIATOR ARP 0 9 2 - 5 4 tEGEND APRM T R I P SYS A INOP OR UPSCALE T R I ?

DEVICE APRM A , C, or E SETPQSNT

  • 15% (mode switch MOT in Run) b 120% (mode switch in Run) b Flaw Bias TCO per table below:

A. Incorrect - With Core Flow 70% the APRM Upscale Thermal Power Trip is clamped at 117 %

B. -

Correct With Core Flow 70% the APRhll Upscale Thermal Power Trip is clamped at 117 %

C. Incorrect - With Core Flow 70% the APR.M Upscale Thermal Power Trip is clamped at 117 %

D. Incorrect - With Core Flow 70% the APRM Upscale Thermal Power Trip is clamped at 117 %

Technical Reference(s): ARP 09-5-2-54 R 6 (Attach if not previously provided)

TS-3.3.1.1 & COLR R 23 Proposed references to be provided to applicants during examination: NONE Learning Objective: SDLP-07C EO-I .05 A.3.D (As available)

Question Source: Bank #

Modified Bank # 21838 (Note changes or attach parent)

New NUREG-I 021, Revision 9 2 of 3

ES-401 Sample Written Examination Form ES-401-5 Question Worksheet Question History: Last NRC Exam 1/1/01 Perry Question Cognitive Level: Memory or Fundamental Knowledge Comprehension or Analysis IO CFR Part 55 Content: 55.41 55.43 Comments: Changed proposed answers from / to:

A. From 114.7 % to 120 YO (reason for plausible = Neutron F I u x - High fixed)

B. 117.0 % kept same correct ( reason changed others as this was only one that stood out with a 4%)

C. From 116.8 % to 111.O % (this is the clamped rod block flow biased)

D. From 117.8 % to 109.0 % (this is 3 YO c the clamped rod block flow biased which I is the same % as (differencebetween choices a & b)

NUREG-1021, Revision 9 3 of 3

ES-40 1 Sample Written Examination Form ES-401-5 Question Worksheet Examination Outline Cross-reference: Level RO SRO Tier # 2 Group # 1 KIA # 217000 K5.06 Importance Rating 2.7* 2.7 217000 Reactor Core Isolation Cooling System (RCIC) - K5. Knowledge of the operational implications of the following concepts as they apply to REACTOR CORE ISOLATION COOLING SYSTEM (RCIC): (CFR:

41.5 / 45.3) K5.06 Turbine operation Proposed Question: # 37 RClC was operating following an initiation when an apparent spurious RCIC turbine trip is received.

The SNO attempts a RCIC turbine reset by having an NPO close and then open the RClC Turbine Trip Throttle Valve.

The NPO completes the task and the SNO has the following indications of RCIC:

RCIC TURB TRlPnHROT VLV - green light ON, red light OFF RCIC TURB GOV VLV - green light ON, red light ON 0 RCIC Steam Supply Inboard Isolation Valve 13MOV green light OFF, red light ON Which ONE of the following describes the current condition of RCIC?

A. RCIC is tripped requiring local mechanical linkage to be reset.

B. RCIC is reset and will start as soon as 13MOV-131, RClC Turbine Steam Inlet Isolation Valve, begins to open.

C. RClC is reset and will start after RClC Turbine Steam Inlet Isolation Valve, is full open.

D. RCIC is tripped requiring ISOL SIG A & 81 RESET pushbuttons to be depressed.

Proposed Answer: A NUREG-1021, Revision 9 1 of3

ES-401 Sample Written Examination Form ES-401-5 Question Worksheet Explanation (Optional): Per OP-I 9, When RClC trips, the following occurs:

0 RClC Turbine Tripnhrottle Valve 13HOV-1 closes 0 RClC Pump Minimum Flow Isolation Valve 13MOV-27 closes 0 RClC Pump Discharge to Reactor Inboard Isolation Valve 13MOV-21 closes When RClC isolates, the following will occur:

RClC Steam Supply Inboard Isolation Valve 13MOV-15 closes RClC Turbine Steam Supply Outboard Isolation Valve 13MOV-16 closes RClC turbine trips NOTE: The following Subsection is used to reset all RClC turbine trips including turbine overspeed and manual.

G.6 RClC Turbine Trignhrottle Valve 13HOV-1 Reset G.6.1 Manually operate 13HOV-1 hand wheel tlo closed posit ion.

G.6.2 IF turbine trip was from overspeed or local manual, THEN perform the following:

a. Pull spring loaded emergency connecting rod against spring force, moving emergency head lever away from emergency tappet and tappet nut.
b. Verify emergency tappet & tappet nut moves downward into "reset" position, engaging head lever & holding emergency connecting rod in position under spring tension.
c. IF emergency tappet & tappet nut did not move into the reset position per Step G.6.2.b, THEN perform the following:
1) Lift up on RClC manual trip lever while pulling spring loaded emergency connecting rod against spring force, moving emergency head lever away from emergency tappet & tappet nut.
2) Verify emergency tappet & tappet nut moves downward into "reset" position, engaging head lever & holding emergency connecting rod in position under spring tension.

G.6.3 Ensure trip hook engages latchup lever.

G.6.4 Manually open 13HOV-1 to full open.

G.6.5 Manually close one-quarter turn 13HOV-1 handwheel to allow for thermal expansion.

G.6.6 Verify annunciator 09-4-1-14 RClC TRIP is clear.

G.6.7 Verify TURB TRIP & THROTTLE VLV 13HOV-1 indicates open at panel 09-4.

From SDLP-13-Turbine Trip Throttle Valve, 13HOV-1

b. Interlocks - Any turbine trip or system isolation will energize the solenoid which releases a latch mechanism on the valve allowing a spring to close the valve. This valve must be reset manually at the turbine, per OP-19 instructions.

A. Correct - Per OP-I 9, When RClC trips, the RClC Turbine Tripnhrottle Valve 13HOV-1 closes & must be manually reset locally.

6. Incorrect - RClC is tripped & will NOT be at speed.

C. Incorrect - With RClC TURB TRIPTTHROT VLV- green light ON & red light off, this indicates that the HOV-1 is Not Open.

D. Incorrect - RClC is not isolated as indicated by 13MOV green light OFF, red light ON Technical Reference(s): OP-I 9 R 44 (Attach if not previously provided)

ARP 09-4-1-14 R 3 Proposed references to be provided to applicants during examination: NONE Learning Objective: SDLP-13 EO-1.14.C (As available)

NUREG-1021, Revision 9 2 of 3

ES-401 Sample Written Examination Form ES-401-5 Question Worksheet Question Source: Bank #

Modified Bank # X (Note changes or attach parent)

New Question History: Last NRC Exam 4/1/2000 Grand Gulf Question Cognitive Level: Memory or Fundamental Knowledge Comprehension or Analysis X 10 CFR Part 55 Content: 55.41 55.43 Comments: (NOTE- use BANK distracters if plausible, otherwise please provide ideas.)

Added: RClC Steam Supply Inboard Isolation Valve 13MOV green light OFF, red light ON to the stem to make distracter D wrong.

Added an apparent spurious trip so it does not appear the CRO is violating procedure by:

- not determining (or attempting) to determine ciause of the trip

- not waiting for the NPO to reset the local overspeed (if that was the cause).

NUREG-1021, Revision 9 3 of 3

ES-401 Sample Written Examination Form ES-401-5 Question Worksheet Examination Outline Cross-reference: Level RO SRO Tier # 2 Group # 1 WA # 218000 K6.01 Importance Rating 3.9 4.1 218000 Automatic Depressurization System - K6. Knowledge of the effect that a loss or malfunction of the following will have on the AUTOMATIC DEPRESSURIZATION SYSTEM: (CFR: 41.7 I45.7)

K6.01 RHWLPCI system pressure: Plant-Specific Proposed Question: # 38 The plant was operating at 100% power, with the B RHR pump out of service for maintenance. Subsequently a small break LOCA (SBLOCA) occurs.

The plant configuration is as follows:

0 Reactor vessel water level is + 55 inches 0 HPCl is injecting with discharge pressure at 1000 psig ADS NormaVOverride switches on Panel 09-4 are in the Normal position 0 Annunciator 09-4-1-28 [ADS Timers Actuated] lit 3 minutes ago 0 Core Spray and RHR pumps are running in minimum flow, although annunciator 09-4-1-26 [Core Spray or RHR running] is not lit Drywell pressure is 2.5 psig What is the status of all SRVs?

Valves A, B, CyD, E, G , H -

Valves F, J, K, L A. OPEN OPEN B. OPEN CLOSED C. CLOSED CLOSED D. CLOSED OPEN Proposed Answer: C Explanation (Optional): RPV pressure is below the SRV lift setpoint as determined by HPCl discharge at 1000 psig, no SRVS will be open. ADS valves will not open due to no input into the ADS logic from RHR or CS pump operation as noted by 09-4-1-26 [Core Spray or RHR running] is not lit. Valves A, 6,C, D, E, G, H are ADS valves. Valves F, J, K, L are non ADS valves.

NUREG-1021, Revision 9 1 of2

ES-401 Sample Written Examination Form ES-401-5 Question Worksheet A. Incorrect - RPV pressure is below the SRV lift setpoint as determined by HPCl discharge at 1000 psig, no SRVS will be open. ADS valves will not open due to no input into the ADS logic from RHR or CS pump operation as noted by 09-4-1-26 is not lit.

B. Incorrect - RPV pressure is below the SRV lift setpoint as determined by HPCl discharge at 1000 psig, no SRVS will be open. ADS valves will not open due to no input into the ADS logic from RHR or CS pump operation as noted by 09-4-1-26 is not lit.

C. Correct - RPV pressure is below the SRV lift setpoint as determined by HPCl discharge at 1000 psig, no SRVS will be open. ACE valves will not open due to no input into the ADS logic from RHR or CS pump operation as noted by 09-4-1-26 is not lit.

D. Incorrect - RPV pressure is below the SRV lift setpoint as determined by HPCl discharge at 1000 psig, no SRVS will be open. ADS valves will not open due to no input into the ADS logic from RHR or CS punip operation as noted by 09-4-1-26 is not lit.

Technical Reference(s): OP-68 Rev 18 (Attach if not previously provided)

ARP-09-4-1-28 Rev 5 ARP-09-4-1-17 Rev 5 Proposed references to be provided to applicants during examination: NONE Learning Objective: SDLP-02J EO-1.06.D (As available)

Question Source: Bank #

Modified Bank # (Note changes or attach parent)

New X Question History: Last NRC Exam Question Cognitive Level: Memory or Fundamental Knowledge Comprehension or Analysis X 10 CFR Part 55 Content: 55.41 55.43 Comments: Ops reviewer thought that it would ble better to state NO LP ECCS pumps are operating vice:

Core Spray and RHR pumps are running in minimum flow, although annunciator 09-4-1-26 [Core Spray or RHR running] is not lit NUREG-1021, Revision 9 2of2

ES-401 Sample Written Examination Form ES-401-5 Question Worksheet Examination Outline Cross-reference: Level RO SRO Tier # 2 Group # 1 K/A, # 223002 A I .02 Importance Rating 3.7 223002 Primary Containment Isolation System/Nuclear Steam Supply System Shut-off Ability to predict and/or monitor changes in parameters associated with operating the PRIMARY CONTAINMENT ISOLATION SYSTEM/NUCLEAR !STEAM SUPPLY SHUT-OFF controls including:(CFR:

41.5 I45.5)A I .02 Valve closures Proposed Question: # 39 The reactor was at 100% power, conducting LFRM calibration with TlPs per RAP-7.3.14.

Subsequently, a small break loss of coolant accident (SBLOCA) and loss of off-site power (LOOP) occurs.

  • Drywell pressure is 2.5 psig and increasing
  • Reactor water level decreased to 100 inches and is now stable.
  • All automatic actions have occurred.

Assuming NO operator action, what isolation valves will be the last to close?

A. Group I valves B. TIP ball valves C. Drywell vent and purge valves D. Reactor sample valves Proposed Answer: B Explanation (Optional):

During LPRM calibration with TlPs per RAP-7.3.74, TIPS will be extended into RV. Upon receipt of a PClS initiation, the TlPs will shift to fast speed, retract from the RV, and then the TIP ball valves will shut to isolate the system. This will take -45 seconds, while the ventilation isolation will occur within 10-15 seconds.

Technical Reference(s): RAP-7.3.14 (Attach if not previously provided)

I Proposed references to be provided to applicants during examination:

Learning Objective: (As available)

NUREG-1021, Revision 9 1of2

ES-401 Sample Written Examination Form ES-401-5 Question Worksheet Question Source: Bank #

Modified Bank # (Note changes or attach parent)

New X Question History: Last NRC Exam Question Cognitive Level: Memory or Fundamental Knowledge Comprehension or Analysis X 10 CFR Part 55 Content: 55.41 55.43 Comments:

NUREG-1021, Revision 9 2of2

ES-401 Sample Written Examination Form ES-401-5 Question Worksheet Examination Outline Cross-reference: Level RO SRO Tier # 2 Group # 1 WA # 239002 A2.02 Importance Rating 3.1 239002 Ability to (a) predict the impacts of the following on the RELIEFSEAFETY VALVES; and (b) based on those predictions, use procedures to correct, control, or mitigate the consequences of those abnormal conditions or operations: (CFR:41.5 I45.6) A2.02 Leaky SRV.

Proposed Question: # 40 While operating at 100% power, the following conditions exist:

Annunciator 09-4-1-16 SRV LEAKING is in alarm Annunciator 09-4-2-6 SRV SONIC MON ALRM HI is extinguished 02TR-166 indicates 295°F for SRV D and approximately 135°F for the remaining SRVs Panel 09-4 SRV solenoid energized indications are all extinguished RPV pressure is 1040 psig and stable Which ONE of the following identifies the status of the SRVs and the appropriate action(s) to be taken for this condition?

A. An SRV is leaking. Initiate a CR for Engineering to evaluate.

6. An SRV is open. initiate an immediate plant shutdown.

C. An SRV is open. initiate a plant shutdown within one hour.

D. An SRV is leaking. Initiate an immediate plant shutdown.

Proposed Answer: A Explanation (Optional):

Technical Reference(s): (Attach if not previously provided)

Proposed references to be provided to applicants during examination:

Learning Objective: (As available)

Question Source: Bank #

Modified Bank # (Note changes or attach parent)

NUREG-I 021, Revision 9 1of2

~

13-40 1 Sample Written Examination Form ES-401-5 Question Worksheet New X Question History: Last NRC Exam Question Cognitive Level: Memory or Fundamental Knowledge Comprehension or Analysis X 10 CFR Part 55 Content: 55.41 55.43 Comments:

NUREG-?02?,Revision 9 2 of 2

ES-401 Sample Written Examination Form ES-401-5 Question Worksheet Examination Outline Cross-reference: Level RO SRO Tier # 2 Group # 1 KIA # 259002 A3.04 Importance Rating 3.2 3.2 259002 Reactor Water Level Control System - A3. Ability to monitor automatic operations of the REACTOR WATER LEVEL CONTROL SYSTEM including: (CFR: 41.7 I45.7) A3.04 Changes in reactor feedwater flow Proposed Question: # 41 Plant startup is in progress per OP-65.

Reactor power is 20% and Feedwater is in a normal lineup per OP-2A.

Loss of DC Power System B occurs.

Describe the RPV water level response, if any, to this power supply loss.

What action, if any, is required to respond to the effect on RPV water level?

(1) Reactor level increases.

(2) Take manual control of FW flow, control level at 196.5 to 206.5 inches.

(1) Reactor level decreases.

(2) Take manual control of FW flow, control level at 196.5 to 206.5 inches, (1) Reactor level increases.

(2) Maintain FW flow in Automatic, level will stabilize at approximately 214 inches.

(1) Reactor level stays the same.

(2) No action required.

Proposed Answer: A Explanation (Optional): PER OP-2A -FWLC is normally selected to Water Column B which is affect by a loss of DC 6.Also with power 20% will be in single element level control till > 25 %

power per OP-2A. Per AOP-46 loss of DC B, PV level will rise fast due to 06LI-946 failing downscale which is controlling FWLC in conditions given in the stem. AOP-41 is entered for the level rise and provides directions to place FW in imanual and maintain 196.5 to 206.5.

NUREG-I 021, Revision 9 1 of2

ES-401 Sample Written Examination Form ES-401-5 Question Worksheet A. Correct - Per AOP-46 loss of DC 6, PV level will rise fast due to 06LI-94B failing downscale which is controlling FWLC in conditions given in the stem. AOP-41 is entered for the level rise and provides directions to place FW in manual and maintain 196.5 to 206.5.

B. Incorrect - Per AOP-46 loss of DC B, PV level will rise fast due to 06LI-94B failing downscale which is controlling FWLC in conditions given in the stem.

C. Incorrect - Per AOP-46 loss of DC 6, PV level will rise fast due to 06LI-94B failing downscale which is controlling FWLC in conditions given in the stem. AOP-41 is entered for the level rise and provides directions to place FW in manual and maintain 196.5 to 206.5.

D. Incorrect - Per AOP-46 loss of DC 6, PV level will rise fast due to 06Ll-948 failing downscale which is controlling FWLC in conditions given in the stem.

Technical Reference(s): AOP-46 Rev 13 (Attach if not previously provided)

AOP-41 Rev7 OP-2A Rev 59 Proposed references to be provided to applicants during examination: NONE Learning Objective: SDLP71B EO-1.09.A.10 (As available)

Question Source: Bank #

Modified Bank # (Note changes or attach parent)

New X Question History: Last NRC Exam Question Cognitive Level: Memory or Fundamental Knowledge Comprehension or Analysis X 10 CFR Part 55 Content: 55.41 55.43 Comments:

NUREG-1021. Revision 9 2of2

ES-401 Sample Written Examination Form ES-401-5 Question Worksheet Examination Outline Cross-reference: Level RO SRO Tier # 2 Group # 1 KIP, # 261000 A4.07 Importance Rating 3.1 3.2 261000 Standby Gas Treatment System - A4. Ability to manually operate andlor monitor in the control room: (CFR: 41.7 / 45.5 to 45.8) A4.07 System flow Proposed Question: # 42 The plant is in a startup at 70% power. Drywell pressure has risen to 2.0 psig and A SBGT is being used to vent the drywell per OP-37.

(1) In this mode of operation, what is the design flowrate of ASBGT?

(2) If the existing flowrate is too high, how should the flowrate be reduced if at all?

A. (1) 6000 scfm.

(2) Throttle SGT Fan A Suct lsol Valve,01-125SGT-ZA locally.

B. (1) 6000 scfm.

(2) No procedural direction for flowrate adjustment.

C. (1) 3000 scfm.

(2) Throttle SGT Fan A Suct lsol Valve,01-125SGT-2A locally.

D. (1) 3000 scfm.

(2) No procedural direction for flowrate adjustment.

Proposed Answer: B Explanation (Optional): PER DBD-027, flow rate for SGTS is 6000 scfm, also in OP-20 expected to be 5800 scfm. To adjust the flowrate if too high is done per ST-39D by adjusting SGT Fan A Suct lsol Valve,01-125SGT-2A locally (normally it is locked at 635 open) but this is for adjusting the flowrate and maintaining the reactor building d/p. For the conditions given in the step there is no procedure guidance to make the flow rate adjustment. The correct action would be to shift SGTS trains and initiate corrective action to adjust the A train flowrate per ST-39D. A review of procedures and the DBD did not provide a lower flowrate when venting the DW, the flowrate would be expected to be in the 5800 scfm band when venting through the 6 pipe.

A. Incorrect - For the conditions given in the step there is no procedure guidance to make the flow rate adjustment B. Correct - OP-20 flowrate is 6000 scfm, For the conditions given in the step there is no procedure guidance to make the flow rate adjustment.

C. Incorrect - OP-20 flowrate is 6000 scfm, For the conditions given in the step there is no procedure guidance to make the flow rate adjustment.

D. Incorrect - OP-20 flowrate is 6000 scfm, For the conditions given in the step there is no procedure guidance to make the flow rate! adjustment.

NUREG-I 021, Revision 9 1 of2

ES-401 Sample Written Examination Form ES-401-5 Question Worksheet Technical Reference(s): DBD-027 Rev 3 (Attach if not previously provided)

OP-20 Rev 35 OP-37 Rev 76 ST-39D Rev 22 Proposed references to be provided to applicants during examination: NONE Learning Objective: SDLP-01B EO-1.09.D (As available)

Question Source: Bank #

Modified Bank # (Note changes or attach parent)

New X Question History: Last NRC Exam Question Cognitive Level: Memory or Fundamiental Knowledge X Comprehension or Analysis 10 CFR Part 55 Content: 55.41 55.43 Comments: Changed stem for DW pressure to 2.0 psig below initiation at 2.2 psig.

Changed answer and distracter order from propose question due to information provided in the explanation above.

Ops Reviewer comment- Possibility of eliminating the second part of the question. Change flowrates as follows:

A. 8000scfm.

B. 6000scfm.

C. 4000scfm.

D. 2000scfm.

NUREG-1021, Revision 9 2 of 2

ES-401 Sample Written Examination Form ES-401-5 Question Worksheet Examination Outline Cross-reference: Level RO SRO Tier #

Group #

KIA, # 262001 G 2.4.48 Importance Rating 3.5 3.8 262001 A.C. Electrical Distribution Ability to interpret control room indications to verify the status and operation of system / and understand how operator actions and directives affect plant and system conditions. (CFR:43.5 / 45.12)

Proposed Question: # 43 A plant startup is in progress with power at 50%). You have been directed to transfer 10100 bus from reserve to normal service per Section D.20 of OP-46A.

=TheRPS bus is lined up to the normal source.

=Youobserve that Incoming and Running meter voltage difference is 300 volts.

Given the above conditions, what is the most likely consequence of closing NSS to Bus 10100 BKR 10102?

The transfer will cause.. ...

A. a half scram due to RPS EPA undervoltage trip.

6. a full scram due to RPS EPA undervoltage trip C. the Normal supply breaker to trip on bus overcurrent D. the Reserve supply breaker to trip on bus overvoltage Proposed Answer: C Explanation (Optional):

Technical Reference(s): (Attach if not previously provided)

Proposed references to be provided to applicants during examination:

Learning Objective: (As available)

Question Source: Bank #

NUREG-1021, Revision 9 1 of2

ES-40 1 Sample Written Examination Form ES-401-5 Question Worksheet Modified Bank # (Note changes or attach parent)

New X Question History: Last NRC Exam ~

Question Cognitive Level: Memory or Fundamental Knowledge X Comprehension or Analysis 10 CFR Part 55 Content: 55.41 55.43 Comments:

NUREG-1021, Revision 9 2of2

ES-401 Sample Written Examination Form ES-401-5 Question Worksheet Examination Outline Cross-reference: Level RO SRO Tier # 2 Group # 1 KIP, # 262002 K1.O1 Importance Rating 2.8 3.1 262002 Uninterruptable Power Supply (A.C.1D.C.) - K1. Knowledge of the physical connections and/or cause-effect relationships between UNINTERRUPTABLE POWER SUPPLY (A.C./D.C.) and the following: (CFR: 41.2 to 41.9 / 45.7 to 45.8) K1.01 Feedwater level control: Plant-Specific Proposed Question: # 44 The plant is at 100% power.

The following alarm; UPS Gen Volts Lo annunciates at panel 09-8 for 60 seconds.

Assuming all automatic actions function as designed, and with operator action, which ONE of the following will occur?

A. Loss of Feedwater control signal, full scram on low reactor vessel water level.

B. Loss of A, C, E APRMs, half scam C. Loss of B, D, F APRMs, half scam.

D. Loss of Feedwater control signal, full scram from turbine trip.

Proposed Answer: D Explanation (Optional): Per AOP-21 for a loss of UPS as indicated by the UPS Gen Volts Lo alarm which is a symptom of either a complete loss or momentary loss of UPS, (stem provides 60 seconds as cue to complete loss) RWR will runback, RFPS will lock up at pre power loss position, RPV level will rise quickly due to stmlfeed flow mismatch, with no operator action the main turbine will trip on high RPV level. UPS is riot a power supply to the APRMS - they are powered from RPS MG sets.

A. Incorrect - Level will cause a high RPV level trip of turbine and a scram.

B. Incorrect - UPS is not a power supply to the APRMS - they are powered from RPS MG sets.

C. Incorrect - UPS is not a power supply to the APRMS -they are powered from RPS MG sets.

D. Correct - UPS Gen Volts Lo alarm indicates a complete loss of UPS, RWR will runback, RFPS will lock up at pre power loss position, RPV level will rise quickly due to stm/feed flow mismatch, with no operator action thle main turbine will trip on high RPV level.

Technical Reference(s): AOP-21 R21 (Attach if not previously provided)

ARP-09-8-1-17 Rev 3 Proposed references to be provided to applicants during examination:

NUREG-1021, Revision 9 1 of2

~

ES-401 Sample Written Examination Form ES-401-5 Question Worksheet Learning Objective: SDLP-71F EO-1. I 5.A (As available)

Question Source: Bank #

Modified Bank ## (Note changes or attach parent)

New X Question History: Last NRC Exam Question Cognitive Level: Memory or Fundaniental Knowledge Comprehension or Analysis X 10 CFR Part 55 Content: 55.41 55.43 Comments:

NUREG-1021, Revision 9 2of2

ES-401 Sample Written Examination Form ES-401-5 Question Worksheet Examination Outline Cross-reference: Level RO SRO Tier # 2 Group # 1 WA, # 263000 K2.01 Importance Rating 3.1 3.4 263000 D.C. Electrical Distribution - K2. Knowledge of electrical power supplies to the following:

(CFR: 41.7) K2.01 Major D.C. loads Proposed Question: # 45 The plant is at 100% power.

All systems are operating normally when a loss of DC Power System A occurs.

Which ONE of the following valves loses electric stroke capability due to the loss of DC Power System A?

A. 13MOV RClC Pump Suct from Corid Stor lsol Valve.

6. 23MOV HPCI Steam Supply lnbd Is01 Valve.

C. 13MOV RClC Steam Supply lnbd Is01 Valve.

D. 23MOV HPCl Turb Exh Line Vac Breaker Valve.

Proposed Answer: A Explanation (Optional): 13MOV-18 is powered from 71BMCC-1 (DC system A) per attachment #

2 of AOP-45 on a loss of DC all RClC MOVS will lose power except 13MOV-15 which is powered from a AC source (71MCC-163), The I-1PCI valves listed are powered from a AC source.

A. Correct -13MOV-18 is powered from 71BMCC-1 (DC system A) per attachment # 2 of AOP-45 on a loss of DC all RClC MOVS will lose power except 13MOV-15.

6. Incorrect - AC powered from 71MCC-153.

C. Incorrect - AC powered from 71MCC-163.

D. Incorrect - AC powered from 71MCC-163.

Technical Reference(s): AOP-45 Rev 9 (Attach if not previously provided)

OP-15 Rev 54 OP-19 Rev 46 Proposed references to be provided to applicants during examination: NONE Learning Objective: SDLP-716 EO-1.09.A.2 (As available)

NUREG-1021, Revision 9 1 of2

ES-401 Sample Written Examination Form ES-401-5 Question Worksheet Question Source: Bank #

Modified Bank # (Note changes or attach parent)

New X Question History : Last NRC Exam Question Cognitive Level: Memory or Fundamental Knowledge X Comprehension or Analysis 10 CFR Part 55 Content: 55.41 55.43 Comments:

NUREG-1021, Revision 9 2 of 2

ES-401 Sample Written Examination Form ES-401-5 Question Worksheet Examination Outline Cross-reference: Level RO SRO Tier # 2 Group # 2 KIA # 264000 K3.02 Importance Rating 3.9 264000 Knowledge of the effect that a loss or malfunction of the EMERGENCY GENERATORS (OIESELlJET) will have on the following: (CFR41 ."? I 45.4) K3.02 A.C. electrical distribution.

Proposed Question: # 46 The plant is in Mode 2 when a Loss of Off-site Power (LOOP) occurs. The NPO dispatched to monitor Emergency Generator operation reports the following; EDG 'A' HIGH WATER TEMP annunciator is in alarm EDG 'A' jacket water temperature reads 210°F Which of the following describes the status of bus A 0500 and bus 10600?

Bus 10500 Bus 10600 A. De-energized De-energized B. Energized De-energized C. Energized Energized D. De-energized Energized Proposed Answer: C Explanation (Optional):

Technical Reference(s): (Attach if not previously provided)

Proposed references to be provided to applicants during examination:

Learning Objective: (As available)

Question Source: Bank #

Modified Bank # (Note changes or attach parent)

New X NUREG-1021, Revision 9 1 of2

ES-401 Sample Written Examination Form ES-401-5 Question Worksheet Question History: Last NRC Exam Question Cognitive Level: Memory or Fundamental Knowledge Comprehension or Analysis X 10 CFR Part 55 Content: 55.41 55.43 Comments:

NUREG-1021, Revision 9 2 of 2

ES-401 Sample Written Examination Form ES-401-5 Question Worksheet Examination Outline Cross-reference: Level RO SRO Tier # 2 Group # 1 KIA # 300000 K4.03 Importance Rating 2.8 300000 Knowledge of INSTRUMENT AIR SYSTEM design feature(s) andlor interlocks which provide for the following: (CFR: 41.7) K4.03 Securing of IAS upon loss of cooling water.

Proposed Question: # 47 The plant is operating at 100% power when the TBCLC system is lost.

Which of the following sensing points provides ,the FIRST signal to trip the Air compressors?

A. Low Compressor cooling water pressure B. High Compressor intercooler air temperature C. Low TBCLC flow D. High compressor bearing oil pressure Proposed Answer: 6 Explanation (Optional):

Technical Reference(s): ( A h c h if not previously provim Proposed references to be provided to applicants during examination:

Learning Objective: (As available)

Question Source: Bank #

Modified Bank # (Note changes or attach parent)

New X Question History: Last NRC Exam -

Question Cognitive Level: Memory or Fundamental Knowledge X Comprehension or Ainalysis NUREG-1021, Revision 9 1of2

ES-401 Sample Written Examination Form ES-401-5 Question Worksheet 10 CFR Part 55 Content: 55.41 55.43 Comments:

NUREG-1021. Revision 9 2of2

ES-401 Sample Written Examination Form ES-401-5 Question Worksheet Examination Outline Cross-reference: Level RO SRO Tieir # 2 Group # 2 KIA # 400000 G 2.4.1 1 Importance Rating 3.4 3.6 400000 Component Cooling Water Knowledge of abnormal condition procedures: (CFR: 41 . I O / 43.5 / 45.13).

Proposed Question: # 48 The reactor scrammed from 100% power apprclximately 10 minutes ago due to MSlV closure.

Plant conditions are as follows:

HPCl failed to start on Low-Low reactor level Reactor level is 100 inches and slowly decreasing All other ECCS equipment is operating as designed Drywell pressure is 2.3 psig and slowly rising Drywell temperature is 285°F and rising Annunciator 09-6-2-22 RBC MAKEUP TK LVL HI OR LO is in alarm Annunciator 09-4-2-12 DW FLOOR SUMP LEAKAGE is in alarm In response to the alarms, the crew isolates RBCLC flow to the drywell.

Per EOP-4, what actions will maximize drywell cooling?

A. Re-establish RBCLC flow to the drywell coolers.

6. Initiate Emergency Depressurization.

C. Establish drywell cooling from ESW IAW A,OP-11, Section F.2.4 D. Initiate Drywell Sprays as soon as conditions permit.

Proposed Answer: D Explanation (Optional):

Technical Reference(s ): (Attach if not previously provided)

Proposed references to be provided to applicants during examination:

Learning Objective: (As available)

NUREG-1021, Revision 9 1 of2

ES-401 Sample Written Examination Form ES-401-5 Question Worksheet Question Source: Bank #

Modified Bank # (Note changes or attach parent)

New X Question History: Last NRC Exam Question Cognitive Level: Memory or Fundamental Knowledge Comprehension or Analysis X 10 CFR Part 55 Content: 55.41 55.43 Comments:

NUREG-1021, Revision 9 2 of 2

ES-401 Sample Written Examination Form ES-401-5 Question Worksheet Examination Outline Cross-reference: Level RO SRO Tier # 2 Group # 1 KIA # 261000 A I .04 Importance Rating 3.0 3.3 261000 Standby Gas Treatment System - A I . Ability to predict and/or monitor changes in parameters associated with operating the STANDBY GAS TREATMENT SYSTEM controls including: (CFR: 41.5 / 45.5) A I .04 Secondary containment differential pressure Proposed Question: # 49 A large break LOCA and a Loss of Offsite Power (LOOP) has occurred.

0 DW pressure is 15 psig.

0 A & C EDGs failed to start.

0 SGT Disch Header To Stack Is01Valve 01-125SGT-1 did not change position.

0 All other plant equipment functioned as (designed.

What, if any, action is REQUIRED to maintain the reactor building at a negative pressure?

A. No action required.

8. Reposition SGT Disch Header To Stack Is01 Valve 01-125SGT-1 C. Stop Reactor Building Supply Fan FN-513.

D. Stop Below Elevation 369 Exhaust Fan FN-128.

Proposed Answer: A Explanation (Optional): Valve 01-125SGT-1 is a manually operated butterfly valve, which is normally open.

A. Correct - For RB isolated on high DW pressure, flow rate of SGT is 5600-5800 scfm.

System is normally aligned to provide this flow upon initiation.

B. Incorrect - SBGT valve 01-125SGT-1 is the SGT disch header to stack isolation valve and is normally open and remains open on initiation.

C. Incorrect - RB ventilation will be in recirc mode due to RB isolation. Stopping supply and exhaust fans will not maintain the RB negative.

D. Incorrect - RB ventilation will be in recirc mode due to RB isolation. Stopping supply and exhaust fans will not maintain the RB negative.

Technical Reference(s): OP-20 R 35 (Attach if not previously provided)

~ ~~

Proposed references to be provided to applicants during examination: NONE NUREG-1021, Revision 9 1 of2

ES-40I Sample Written Examination Form ES-401-5 Question Worksheet Learning Objective: SDLP-01B EO-1.03 (As available)

Question Source: Bank #

Modified Bank # (Note changes or attach parent)

New X Question History: Last NRC Exam Question Cognitive Level: Memory or Fundamental Knowledge Comprehension or Analysis X 10 CFR Part 55 Content: 55.41 55.43 Comments: Verify KA question had 261000 A I .05 but sample plan had 261000 A I .04 Added lineup name to SGT Disch Header To Stack Is01 Valve 01-125SGT-1 Included correct name for Below Elevation 369 Exhaust Fan FN-128 NUREG-1021, Revision 9 2of2

ES-40 1 Sample Written Examination Form ES-401-5 Question Worksheet Examination Outline Cross-reference: Level RO SRO Tier # 2 Group # 1 KIA # 262001 A2.04 Importance Rating 3.8 4.2 262001 A.C. Electrical Distribution - A2. Ability to (a) predict the impacts of the following on the A.C. ELECTRICAL DISTRIBUTION ; and (b) based on those predictions, use procedures to correct, control, or mitigate the consequences of those abnormal conditions or operations:

(CFR: 41.5 / 45.6) A2.04 Types of loads that, if deenergized, would degrade or hinder plant operation Proposed Question: # 50 The plant is at 100% power.

RX WTR LVL COLUMN SEL switch is in A-LEVEL while calibrating the B-Level instrument.

Subsequently, L26 is discovered to be over-heating and needs to be IMMEDIATELY deenergized.

The Crew must implement Procedure(s)  ?

A. AOP-59 Loss of RPS A Power and AOP-41 Feedwater Malfunction (Rising Feedwater Flow- High RPV Water Level)

B. AOP-59 Loss of RPS A Power.

C. AOP-60 Loss of RPS B Power D. AOP-60 Loss of RPS B Power and AOP-41 Feedwater Malfunction (Rising Feedwater Flow- High RPV Water Level).

Proposed Answer: C Explanation (Optional): Per AOP-19B R 1, Loss of Switchgear L26 Automatic Action is a loss of RPS Bus B and a PClS Group II Isolation Per AOP-46 R 13 DC Power System B (note will not lose the DC bus for a period of time) will lose the charger so this AOP will be entered but per Automatic Actions -Loss of DC Power System B If RX WTR LVL COLUMN SEL 06-SI switch is in A-LEVEL, RPV water level will not be affected.

A. Incorrect - Loss of L26 will impact RPS B not A, per stem loss of DC Bus B will not affect level if on water column A.

B. Incorrect - Loss of L26 will impact RPS B not A, C. Correct - Loss of Switchgear L26 Automatic Action is a loss of RPS Bus B (AOP-60) & a PClS Group II Isolation (AOP-15)

D. Incorrect - Per stem loss of DC Bus B will not affect level if on water column A. (note will not lose the DC bus for a period of time)

NUREG-1021, Revision 9 1 of2

ES-40 1 Sample Written Examination Form ES-401-5 Question Worksheet Technical Reference(s): AOP-199 R 1 AOF-46 R 13 (Attach if not previously provided)

AOP-60 R 5 AOP-59 R 7 AOP-15 R 26 Proposed references to be provided to applicants during examination: NONE Learning Objective: LP-AOP EO-1.06 (As available)

Question Source: Bank #

Modified Bank # (Note changes or attach parent)

New X Question History: Last NRC Exam Question Cognitive Level: Memory or Fundamental Knowledge Comprehension or Analysis X 10 CFR Part 55 Content: 55.41 55.43 Comments: Proposed Question was as follows and a ARP reference was not found, suggest the alternate question above.

The NY IS0 is in a heat emergency with area temperatures forecast to remain above 90 OF for the next few days.

The plant is at 100% power with all AC electrical distributions systems in a normal lineup.

Alarm OX-XX-X-X annunciates indicating a a tottat loss of cooling for T-4.

Per ARP-OX-XX-X-X, what action is required to (ensurecontinued 100% power generation for the next 24 hours2.777778e-4 days <br />0.00667 hours <br />3.968254e-5 weeks <br />9.132e-6 months <br />?

A. Manually start and load all EDGs.

B. Reduce main generator load to 2222 amps.

C. Manually transfer station loads to the RSSTs.

D. Start the standby generator bus duct cooling fan.

NUREG-1021, Revision 9 2 of 2

ES-40 1 Sample Written E.xaminat ion Form ES-401-5 Question Worksheet Examination Outline Cross-reference: Level RO SRO Tier # 2 Group # 2 KIA # 262002 K4.01 Importance Rating 3.1 3.4 262002 Uninterruptible Power Supply (A.C./D.C.) K4. Knowledge of UNINTERRUPTIBLE POWER SUPPLY (A.C./D.C.) design feature(s) and/or interlocks which provide for the following: (CFR: 41 -7)

K4.01 Transfer from preferred power to alternate power supplies Proposed Question: # 51 Which of the following sets of conditions will cause an automatic uninterrupted transfer of the UPS M-G set from the AC Motor Drive to the DC Motor Drive?

A. AC motor over-temperature, generator under-frequency B. M-G Set over-speed, generator under-frequency C. Generator ground , generator under-voltalge D. Generator over-current, generator over-load Proposed Answer: A Explanation (Optional): Refer to OP-46B Page 10 & 11 A. Correct - Any of the following conditions will cause an automatic uninterrupted transfer from normal AC motor drive to DC motor drive: Main feeder undervoltage (three phase), Main feeder undervoltage (single phase), AC motor overtemperature, Generator underfrequency

6. Incorrect - Any of the following conditions will cause an M-G set shutdown and an automatic interrupted transfer from MG set DC drive to alternate feeder: MG set overspeed, DC motor overload, DC motor undervoltage, Generator underfrequency (after 5 sec TD), Generator undervoltage C. Incorrect -Any of the following conditions will cause an M-G set shutdown and an automatic interrupted transfer from M-G set to alternate feeder: Generator undervoltage, Generator overload, Generator overcurrent, Generator ground D. Incorrect - Any of the following conditions will cause an M-G set shutdown and an automatic interrupted transfer from M-G set to alternate feeder: Generator undervoltage, Generator overload, Generator overcurrent, Genterator ground Technical Reference(s): OP-468 (Attach if not previously provided)

Proposed references to be provided to applicants during examination:

Learning Objective: SDLP-71F EO-1.05.C.1 (As available)

NUREG-1021, Revision 9 1of2

ES-401 Sample Written Examination Form ES-401-5 Question Worksheet Question Source: Bank #

Modified Bank # (Note changes or attach parent)

New X Q uest ion History : Last NRC Exam Question Cognitive LeveI: Memory or Fundamental Knowledge X Comprehension or Analysis 10 CFR Part 55 Content: 55.41 55.43 Comments:

NUREG-1021, Revision 9 2of2

ES-40 1 Sample Written E.xamination Form ES-401-5 Question Worksheet Examination Outline Cross-reference: Level RO SRO Tier # 2 Group # 1 KIA # 263000 A4.02 Importance Rating 3.2 3.1 263000 D.C. Electrical Distribution - A4. Ability to manually operate and/or monitor in the control room:

(CFR: 41.7 / 45.5 to 45.8) A4.02 Battery voltage indicator: Plant-Specific Proposed Question: # 52 The 125 VDC Station Battery Charger, 71BC-1A for 125 VDC Power System 'A' has been placed in a Normal Startup per OP-43A.

The Panel 09-8, 125 VDC Batt Bus Voltage meter is reading 135 VDC.

What adjustments, if any, will the operator make in this situation?

A. Adjust voltages using the FLOAT potentiometer on the battery charger to set voltage within the required range.

B. No adjustments needed because this voltage is acceptable.

C. Adjust voltages using the EQUALIZE potentiometer on the battery charger to set voltage within the required range.

D. No adjustments are needed provided the local battery meter, 71VM-BAT-IABC volts indicates between 136 and 137.5 VDC.

Proposed Answer: A Explanation (Optional): Per OP-43A, Step D.1.2 -Review Step D.1.7 & associated notes &

caution, & ensure a flat blade screwdriver is available to adjust float voltage potentiometer R3.

Step D.1.7 - IF charger output current remains 2 to 310 DC amperes on 71AM-BC-1A for more than one minute, THEN perform the following:

NOTE: Float voltage potentiometer R3 is located below FLOAT label on Float/Equalize toggle switch on front of 71BC-1A.

a. Adjust float voltage potentiometer R3 to establish between 290 & 310 DC amperes on 71AM-BC-1A.
b. As charger output current lowers, adjust float voltage potentiometer R3 to maintain the following:

131 to 133 DC volts on 71VM-BAT-IABC 0 Between 290 and 310 DC amperes on 71AM-BC-1A NUREG-1021, Revision 9 1 of2

ES-40 1 Sample Written Examination Form ES-401-5 Question Worksheet A. Correct - As charger output current lowers, adjust float voltage potentiometer R3 to maintain the following: 131 to 133 DC vcdts on 71VM-BAT-IABC B. Incorrect - 140 VDC too high, maintain 131 to 133 DC volts on 71VM-BAT-IABC C. Incorrect - Per stem a Normal startup indicates in Float, requires adjustment of Float pot to achieve desired voltages of 131 -133 VDC D. Incorrect - Voltages given are for equalize charge, stem Per stem a Normal startup indicates in Float, requires adjustment of Float pot to achieve desired voltages of 131 -

133 VDC Technical Reference(s): OP-43A R 22 (Attach if not previously provided)

Proposed references to be provided to applicants during examination: NONE Learning Objective: SDLP-71B EO-I .13..A (As available)

Question Source: Bank #

Modified Bank # 28794 (Note changes or attach parent)

New Question History: Last NRC Exam 11/29/04 Perry Question Cognitive Level: Memory or Fundamental Knowledge Comprehension or Analysis X 10 CFR Part 55 Content: 55.41 55.43 Comments: added to stem to clarify a normal startup vs. placing in equalize. Changed proposed distracter D to shown as there is no Div 3 Batt. And changed voltages to those for a equalize charge.

NUREG-1021, Revision 9 2of2

ES-401 Sample Written Examination Form ES-401-5 Question Worksheet Examination Outline Cross-reference: Level RO SRO Tier # 2 Group # 1 KIA # 205000 K5.03 Importance Rating 2.8 3.1 205000 Shutdown Cooling System (RHR Shutdown Cooling Mode) - K5. Knowledge of the operational implications of the following concepts as they apply to SHUTDOWN COOLING SYSTEM (RHR SHUTDOWN COOLING MODE): (CFR: 41.5 / 45.3) K5.03 Heat removal mechanisms Proposed Question: # 53 The plant is in shutdown cooling. reactor recirculation pumps are running.

Per OP-I 3D, which of the following is the MINIMUM water level which allows for adequate natural circulation flow within the reactor vessel to ensure accurate temperature indication?

Approximately inches.

A. 370 B. 130 C. 287 D. 235 Proposed Answer: D Explanation (Optional): Per OP-l3D, CAUTION .. If RPV water level is 234.5 inches with no forced core recirculation, reactor coolant temperature indications could be invalid due to insufficient natural circulation.

A. Incorrect - Top of Vessel Flange does not satisfy question.

B. Incorrect - feedwater penetration does nlot satisfy question.

C. Incorrect - MSL centerline does not satisfy question.

D. Correct - Per OP-l3D, CAUTION - If RPV water level is 234.5 inches with no forced core recirculation, reactor coolant temperature indications could be invalid due to insufficient natural circulation Technical Reference(s): OP-I 3D R 18 (Attach if not previously provided)

Proposed references to be provided to applicants during examination: NONE Learning Objective: SDLP-10 EO-1.13.D (As available)

NUREG-1021, Revision 9 1 of2

ES-401 Sample Written Examination Form ES-401-5 Question Woirksheet Question Source: Bank #

Modified Bank # X (Note changes or attach parent)

New Question History: Last NRC Exam Question Cognitive Level: Memory or Fundamental Knowledge X Comprehension or Analysis 10 CFR Part 55 Content: 55.41 55.43 Comments: Clarified the stem by asking which of the following is the MINIMUM.

Made changes to answer to ensure clear determination of level requirement being satisfied.

D. From 234 to 235 (level stated in OP-13D is 234.5)

NUREG-1021, Revision 9 2of2

ES-401 Sample Written E:xamination Form ES-401-5 Question Worksheet Examination Outline Cross-reference: Level RO SRO Tier # 2 Group # 2 KIA # 201003 A2.02 Importance Rating 3.7 3.8 201003 Control Rod and Drive Mechanism - A2. Ability to (a) predict the impacts of the following on the CONTROL ROD AND DRIVE MECHANISM; and (b) based on those predictions, use procedures to correct, control, or mitigate the consequences of those abnormal conditions or operations: (CFR: 41.5 /

45.6) A2.02 Uncoupled rod Proposed Question: # 54 During a plant startup, control rod 14-19 was withdrawn to position 48.

During the coupling check:

Position indication was lost.

0 Control rod overtravel alarmed.

Control rod drift alarmed.

This is an indication that control rod 14-19 [ 1)

The crew is required to enter procedure (2!) .

A. (1) is uncoupled (2) AOP-69, CONTROL ROD DRIVE PUMP TROUBLE B. (1) is uncoupled (2) AOP-25, UNCOUPLED CONTROL R.OD C. (1) is stuck (2) AOP-24, STUCK CONTROL ROD D. (1) is stuck (2) AOP-27, CONTROL ROD DRIFT Proposed Answer: B NUREG-1021, Revision 9 1 of2

ES-401 Sample Written Examination Form ES-401-5 Question Worksheet Explanation (Optional): Per AOP-25 B SYMPTOMS: 09-5-2-4 ROD OVERTRAVEL is received during a coupling check of control rod.

Per ARP 09-5-2-4 R 3, ROD OVERTRAVEL DEVICE Reed Switch S-50 SETPOINT 2 inches beyond full out position Per OP-26 R 22, Control Rod Drift A red drift light comes on & annunciator 09-5-2-3 ROD DRIFT alarms when either of the following conditions occur:

0 A control rod that is not selected moves off a latched even numbered position.

0 A control rod that is selected moves off an even numbered position or past an odd numbered position after rod sequence tirner has stopped.

Per AOP-24 R 8 Stuck Control Rod- A. ENTRY CONDITIONS Control rod cannot be moved with drive water dlp at 400 psig or less B. SYMPTOMS - Failure of RPlS to indicate rod motion,

-Failure of nuclear instrumentation to respond to1 rod, - Abnormally high or low drive water flow A. Incorrect - AOP-69 is not correct for a uncoupled rod as indication by conditions in stem.

B. Correct - Enter AOP-25 due to B SYMPTOMS: 09-5-2-4 Rod Overtravel is received during a coupling check of contirot rod. Per ARP 09-5-2-4 R 3, Rod Overtravel Device Reed Switch S-50 setpoint 2 inches beyond full out position C. Incorrect - AOP-24 conditions are not met for a stuck rod.

D. Incorrect - AOP-27 conditions are not met for a control rod drift.

Technical Reference(s): AOP-25 R 7 (Attach if not previously provided)

ARP 09-5-2-4 R 3 Proposed references to be provided to applicants during examination: NONE Learning Objective: LP-AOP EO-I.01 (As available)

Question Source: Bank #

Modified Bank # 23679 (Note changes or attach parent)

New Question History: Last NRC Exam 3/17/03 Fermi #2 Question Cognitive Level: Memory or Fundamental Knowledge Comprehension or Analysis 10 CFR Part 55 Content: 55.41 55.43 Comments:

NUREG-I 021, Revision 9 2of2

~~ ~ ~

ES-401 Sample Written Examination Form ES-401-5 Question Worksheet Examination Outline Cross-reference: Level RO SRO Tier # 2 Grolup # 2 KIA # 202001 G 2.1.7 Importance Rating 3.7 202001 Recirculation System Ability to evaluate plant performance and make operational judgments based on operating characteristics / reactor behavior / and instrument interpretation. (CFR: 43.5 / 45.12 /

45.1 3)

Proposed Question: # 55 The plant was operating at 100% power when the B RWR pump tripped. The CRS directed entry into AOP-8, Loss or Reduction of Reactor Coolant Flow.

The following plant conditions exist:

  • Reactor Power is approximately 65%
  • A RWR pump speed is 88% and unchanged
  • Annunciator 09-5-2-44 APRM UPSCALE is in alarm Per AOP-8, what operator action is required?

A. Lower the speed of the A RWR pump B. Raise the speed of the A RWR pump.

C. Insert control rods per RAP-7.3.16, Plant Power Changes.

D. Insert a manual scram and enter AOP-1, F;!eactor Scram.

Proposed Answer: C Explanation (Optional):

Technical Reference(s): ARP 09-5-2-44 (Attach if not previously provided)

AOP-8 Proposed references to be provided to applicants during examination:

Learning Objective: (As available)

Question Source: Bank #

Modified Bank # (Note changes or attach parent)

NUREG-1021, Revision 9 1 of2

ES-401 Sample Written Examination Form ES-401-5 Question Worksheet Examination Outline Cross-reference: Level RO SRO Tier :# 2 Group # 2 KIA 202001 G 2.1.7 Impclrtance Rating 3.7 202001 Recirculation System Ability to evaluate plant performance and make operational judgments based on operating characteristics / reactor behavior / and instrument interpretation. (CFR: 43.5 / 45.12 /

45.13)

Proposed Question: # 55 The plant was operating at 100% power when the 8 RWR pump tripped. The CRS directed entry into AOP-8, Loss or Reduction of Reactor Coolant Flow.

The following plant conditions exist:

  • Reactor Power is approximately 65%
  • A RWR pump speed is 88% and unchanged
  • Annunciator 09-5-2-44 APRM UPSCALE is in alarm Per AOP-8, what operator action is required?

A. Lower the speed of the A RWR pump B. Raise the speed of the A RWR pump.

C. insert control rods per RAP-7.3.16, Plant Power Changes.

D. Insert a manual scram and enter AOP-1, Reactor Scram.

Proposed Answer: C Explanation (Optional):

Technical Reference(s): ARP 09-5-2-44 (Attach if not previously provided)

AOP-8 Proposed references to be provided to applicant:; during examination:

Learning Objective: (As available)

Question Source: Bank #

Modified Bank # (Note changes or attach parent)

NUREG-1021, Revision 9 1 of2

ES-401 Sample Written Examination Form ES-401-5 Question Worksheet New X Question History: Last NRC Exam Question Cognitive Level: Memory or Fundamental Knowledge Comprehension or Analysis X 10 CFR Part 55 Content: 55.41 55.43 Comments:

NUREG-I 021, Revision 9 2 of 2

ES-401 Sample Written Examination Form ES-401-5 Question Worksheet Examination Outline Cross-reference: Level RO SRO Tier # 2 Group # 2 WA ;# 202002 K1.08 Importance Rating 3.1 3.2 202002 Recirculation Flow Control System - K1. Knowledge of the physical connections and/or cause effect relationships between RECIRCULATION FLOW CONTROL SYSTEM and the following: (CFR: 41.2 to 41.9 I 45.7 to 45.8) K1.08 Feedwater flow Proposed Question: # 56 The plant is at 90% power. Subsequently, the A Reactor Feed Pump trips.

What AUTOMATIC action, if any, will preclude a reactor water Low-Low level being reached?

A. No automatic action will preclude reaching reactor water Low-Low level.

B. Recirc flow runback to 30%.

C. Recirc flow runback to 44%.

D. Reactor scram.

Proposed Answer: C Explanation (Optional): Per AOP-42, AUTOMATIC Actions for loss of 1 RFP, - RFP trip annunciator alarms - RPV water level lowers - If RPV water level lowers to 196.5 inches, both RWR MG sets run back to 44% speed - Operating RFP should restore level Loss of both RFPs, or loss of two condensate or condensate booster pumps: - Pump trip annunciators alarm - RWR MG sets run back to :30% speed when total feedwater flow reaches 20% - - RPV water level lowers rapidly -196.5 inches annunciator alarms

- 177 inches: the following occur: Reactor scram A. Incorrect - Loss of 1 RFP, concurrent with RPV level of 196.5, Both RWR MG sets runback to 44% speed. Operating RFP should restore level to normal.

6. Incorrect - Loss of Both RFPs or 2 Condiensate/BoosterPumps- Both RWR MG sets runback to 30% speed when total FW flow reaches 20%.

C. Correct - Loss of 1 RFP, concurrent with RPV level of 196.5, Both RWR MG sets runback to 44% speed. Operating RFP should restore level to normal.

D. Incorrect - Loss of 1 RFP, concurrent with RPV level of 196.5, Both RWR MG sets runback to 44% speed. Operating RFP should restore level to normal.

Technical Reference(s): AOP-42 R12 (Attach if not previously provided)

Proposed references to be provided to applicants during examination: NONE NUREG-1021, Revision 9 1of2

ES-401 Sample Written Examination Form ES-401-5 Question Worksheet Learning Objective: LP-AOP EO-1.02 (As available)

Question Source: Bank #

Modified Bank # (Note changes or attach parent)

New X Question History: Last NRC Exam Question Cognitive Level: Memory or Fundamental Knowledge Comprehension or Analysis 10 CFR Part 55 Content: 55.41 55.43 Comments:

NUREG-1021, Revision 9 2of2

ES-401 Sample Written Examination Form ES-401-5 Question Worksheet Examination Outline Cross-reference: Level RO SRO Tier # 2 Group # 2 WA :# 216000 Importance Rating 2.8 216000 Knowledge of the effect that a loss or malfunction of the following will have on the NUCLEAR BOILER INSTRUMENTATION: (CFR: 41.7 / 45.7) K8.02 D.C. electrical distribution Proposed Question: # 57 The reactor is in Mode 4 with an outage of A Battery in progress. DC power system A is energized from Battery Charger A . All other DC and AC components are in the normal lineup.

Subsequently, a Loss of Off-Site Power (LOOP) occurs. The EDGs start and load onto their respective busses.

Assuming operator action, what reactor vessel water level instruments are available in the control room?

A. All reactor vessel water level instruments are available.

B. 06LI-94C is available.

C. 06LI-94B is available.

D. All reactor vessel water level instruments will be available after the EDGs re-energize their busses.

Proposed Answer: C Explanation (Optional):

Technical Reference(s): (Attach if not previously provided)

Proposed references to be provided to applicants during examination:

Learning Objective: (As available)

Question Source: Bank #

Modified Bank # (Note changes or attach parent)

New NUREG-1021, Revision 9 1of2

ES-40 1 Sample Written Examination Form ES-401-5 Question Worksheet Question History: Last NRC Exam Question Cognitive Level: Memory or Fundamental Knowledge Comprehension or Analysis 10 CFR Part 55 Content: 55.41 55.43 Comments:

NUREG-1021, Revision 9 2 of 2

ES-401 Sample Written Examination Form ES-401-5 Question Worksheet Examination Outline Cross-reference: Level RO SRO Tier # 2 Group ## 2 KIA if# 219000 K2.02 Importance Rating 3.1 219000 RHFULPCI: Torus/Suppression Pool Cooling Mode Knowledge of electrical power supplies to the following: (CFR:41.7) K2.02 Pumps Proposed Question: # 58 Plant conditions are as follows:

90% power.

0 Torus temperature is 90 O F and rising slowly.

0 4160Kv Bus 10600 is deenergized and remains deenergized Which one of the following describes the RHR loops and RHRSW pumps available for Torus Cooling per OP-I 3 8 and OP-I 3C based on the above conditions?

RHR Pumps RHR SW Pumps A. 10P-3A, IOP-3C 1OP-IA, 1OP-1C

9. 10P-3A, IOP-3B 10P-IA, IOP-IC C. 10P-3C, 10P-3D 1OP-I 6, 1OP-I D D. 10P-3B, IOP-3D 1OP-I 6,1OP-I D Proposed Answer: B Explanation (Optional):

A.

Technical Reference(s): OP-I 3 8 Rev 9 (Attach if not previously provided)

OP-13C Rev 9 Proposed references to be provided to applicants during examination: NONE Learning Objective: SDLP-10 EO-1.04 (As available)

Question Source: Bank #

Modified Bank # (Note changes or attach parent)

New X NUREG-1021, Revision 9 1 of2

~

ES-40 1 Sample Written Examination Form ES-401-5 Question Worksheet Question History: Last NRC Exam Question Cognitive Level: Memory or Fundamental Knowledge X Comprehension or Analysis 10 CFR Part 55 Content: 55.41 55.43 Comments: Set Stem temperature at 97 F to ensure EOP conditions.

NUREG-1021, Revision 9 2 of 2

ES-401 Sample Written Examination Form ES-40 1-5 Question Worksheet Examination Outline Cross-reference: Level RO SRO Tier # 2 Group # 2 KIA :# 230000 A4.06 Importance Rating 4.0 230000 RHWLPCI Torus/Suppression Pool Spray Mode Ability to manually operate and/or monitor in the control room: (CFR:41.7 / 45.5 to 45.8)

A4.06 Valve logic reset following automatic initiation of LPCVRHR in injection mode.

Proposed Question: # 59 The following plant conditions exist:

Drywell pressure is 10 psig and steady Drywell temperature is 150°F and steady RPV Fuel Zone level is -10 inches and steiady The operator has taken switch 10A-S17A (SPRAY CONTROL) to MANUAL Which ONE of the following describes torus spraiy response, if any, AND why?

A. Torus spray will not initiate due to drywell pressure.

B. Torus spray will not initiate due to RPV level.

C. Torus spray will initiate once the SPRAY CONTROL switch is allowed to spring return to NORMAL D. Torus spray will initiate as long as the SPRAY CONTROL switch is held in MANUAL.

Proposed Answer: B Explanation (Optional):

If Fuel Zone level indicates e0 inches, the DW arid Torus spray valve override of fuel zone level keylock switch 1OA-SI 8A(B) must be taken to manual override position to allow opening of DW and/or torus spray valves

' Technical Reference(s): (Attach if not previously provided)

Proposed references to be provided to applicants during examination:

NUREG-1021, Revision 9 1 of2

ES-401 Sample Written Examination Form ES-401-5 Question Worksheet Learning Objective: (As available)

Question Source: Bank #

Modified Bank # (Note changes or attach parent)

New X Question History: Last NRC Exam Question Cognitive Level: Memory or Fundamental Knowledge X Comprehension or Analysis 10 CFR Part 55 Content: 55.41 55.43 Comments:

NUREG-1021, Revision 9 2 of 2

ES-40 1 Sample Written Examination Form ES-401-5 Question Worksheet Examination Outline Cross-reference: Level RO SRO Tier # 2 Group # 2 KIA :# 234000 K1.04 Importance Rating 3.3 234000 Knowledge if the physical connections andlor cause effect relationships between FUEL HANDLING EQUIPMENT and the following: (CFR:41.2 to 41.9 / 45.7 to 45.8) K1.04 Reactor manual control system: Plant specific Proposed Question: # 60 Given the following conditions:

  • ALL hoists full up
  1. ALL hoists unloaded
  • Mode switch in Refuel
  • All rods fully inserted For these conditions, which ONE of the followincJcombinations will result in a rod withdrawal block?

A. Frame mounted hoist not full up AND Refuel Bridge near or over the core

8. Placing Mode switch to STARTUP / AND Refuel bridge over the Spent Fuel HOTSTANDBY Pool C. Fuel grapple not full up AND Refuel Bridge near or over the core D. Fuel grapple and frame mounted AND Refuel bridge over the Spent Fuel hoists not full up Pool Proposed Answer: C Explanation (Optional):

Technical Reference(s): (Attach if not previously provided)

Proposed references to be provided to applicants during examination:

Learning Objective: (As available)

NUREG-1021, Revision 9 1 of2

ES-40 1 Sample Written Examination Form ES-401-5 Question Worksheet Question Source: Bank #

Modified Bank # (Note changes or attach parent)

New X Question History: Last NRC Exam Question Cognitive Level: Memory or Fundamental Knowledge X Comprehension or Analysis 10 CFR Part 55 Content: 55.41 55.43 Comments:

NUREG-1021, Revision 9 2of2

ES-40 1 Sample Written Examination Form ES-401-5 Question Worksheet Examination Outline Cross-reference: Level RO SRO Tier # 2 Group # 2 WA # 290002 K3.03 Importance Rating 3.3 3.4 290002 Knowledge of the effect that a loss or malfunction of the REACTOR VESSEL INTERNALS will have on the following: (CFR: 41.7 / 45.4) K3.03 Reactor power Proposed Question: # 61 You are the ATC operator. The plant is at 100% power when you observe the following:

  • Reactor power lowers
  • Main generator output lowers
  • Main steam flow lowers
  • Indicated core flow lowers C o r e plate differential pressure lowers
  • Recirculation Loop A flow rises What is the cause of these indications?

A. There has been a malfunction in EHC B. An SRV is stuck open C. A reactor recirculation pump controller has failed downscale D. One or more jet pumps in the A loop has malfunctioned Proposed Answer: D Explanation (Optional):

TechnicaI Reference(s): A 0 P-29 (Attach if not previously provided)

Proposed references to be provided to applicants during examination:

Learning Objective: (As available)

Question Source: Bank #

Modified Bank ## (Note changes or attach parent)

New X NUREG-1021, Revision 9 1 of2

ES-401 Sample Written Examination Form ES-401-5 Question Worksheet Question History: Last NRC Exam Question Cognitive Level: Memory or Fundamental Knowledge Comprehension or Analysis X 10 CFR Part 55 Content: 55.41 55.43 Comments:

NUREG-1021, Revision 9 2of2

~~

ES-401 Sample Written Examination Form ES-401-5 Question Worksheet Examination Outline Cross-reference: Level RO SRO Tier # 2 Group # 2 KIA # 241000 Importance Rating 3.8 241 000 Reactornurbine Pressure Regulating System Knowledge of REACTORITURBINE PRESSURE REGULATING SYSTEM design feature(s) and/or interlocks which provide for the following: (CFR: 41.7)K4.01 Reactor pressure control Proposed Question: # 62 The plant was at 100% power when a low vacuum turbine trip occurred. All immediate actions of AOP-1 are complete.

Plant parameters are as follows:

.Two circulating water pumps are running

  • RPV level 200 inches and stable

.RPV pressure 950 psig and stable

-RPV level control is with RClC Condenser vacuum is 9 inches Hg and steady The CRS has directed reactor pressure be controlled with the turbine bypass valves.

Which of the following describes the actions required to accomplish the CRSs direction?

A. Reset the turbine trip and lower the EHC pressure setpoint B. Reset the turbine trip and raise the EHC pressure setpoint C. Raise the bypass opening jack setpoint D. Lower the bypass opening jack setpoint Proposed Answer: C Explanation (Optional):

Tech nicaI Reference(s): (Attach if not previously provided)

Proposed references to be provided to applicants; during examination:

NUREG-1021, Revision 9 1 of2

ES-401 Sample Written Examination Form ES-401-5 Question Worksheet Learning Objective: (As available)

Question Source: Bank #

Modified Bank # (Note changes or attach parent)

New Question History: Last NRC Exam Question Cognitive Level: Memory or Fundamental Knowledge Comprehension or Analysis 10 CFR Part 55 Content: 55.41 55.43 Comments:

NUREG-1021, Revision 9 2 of 2

ES-40 1 Sample Written Examination Form ES-401-5 Question Worksheet Examination Outline Cross-reference: Level RO SRO Tier # 2 Group # 2 WA :# 295034 EA2.01 Importance Rating 2.8 259001 Ability to predict andlor monitor changes in parameters associated with operating the REACTOR FEEDWATER SYSTEM controls including: (CFR:41.5 / 45.5) A I .05 RFP turbine control vale position:

Turbine-driven only.

Proposed Question: # 63 The plant is in cold shutdown. Mechanical maintenance has reported that the ARFPT has a problem. The High Pressure (HP) Steam Control Valve is closed and separated from its linkage. Mechanically, the arm is intact but the lip steam control valve is closed via the secondary piston.

If a reactor startup were to commence, the A RFPT (1) be available at 500 psig reactor pressure and (2) be available above 50% power.

(1) (2)

A. would would NOT

3. would would C. would NOT would NOT D. would NOT would Proposed Answer: D Explanation (Optional):

Techn icaI Reference(s): (Attach if not previously provided)

Proposed references to be provided to applicant!; during examination:

Learning Objective: (As available)

Question Source: Bank #

Modified Bank # (Note changes or attach parent)

New X NUREG-1021, Revision 9 1 of2

ES-401 Sample Written Examination Form ES-401-5 Question Worksheet Question History: Last NRC Exam Question Cognitive Level: Memory or Fundamental Knowledge X Comprehension or Analysis 10 CFR Part 55 Content: 55.41 55.43 Comments:

NUREG-I 021, Revision 9 2of2

ES-401 Sample Written Examination Form ES-401-5 Question Worksheet Examination Outline Cross-reference: Level RO SRO Tier # 2 Group # 2 K/A # 271000 A3.05 Importance Rating 2.9 271000 Ability to monitor automatic operations of the OFF GAS SYSTEM including:(CFR:41.7 I45.7)

.A3.05 System indicating lights and alarms.

Proposed Question: # 64 The plant is at 100% power.

A malfunction in the Hydrogen Water chemistry icontroller causes a full flow of H2 into the feed water.

The following alarms are received in the Control Room:

009-6-4-35 H2 ADDITION SYS TROUBLE 009-3-3-1 MAIN STM RAD MON HI

  • 09-3-2-27 OFF GAS RAD MON HI You dispatch an operator to 89 HAP and he reports annunciator 89HAP-1-3 OFF GAS EXCESS 0 2 LOW alarm is lit.

Describe the response of the Off Gas system:

A. The recombiner will continue to operate.

B. The recombiner will trip.

C. The SJAE Off Gas Trip Suction air operated valves 38AOV-113 A and B will close.

D. The Off Gas Outlet Isolation valve 01-107A,OV-100 will close.

Proposed Answer: A Explanation (Optional):

Technical Reference(s): AOP-24A (Attach if not previously provided)

Proposed references to be provided to applicants; during examination:

Learning Objective: (As available)

NUREG-1021, Revision 9 1 of2

~~ ~

ES-401 Sample Written Examination Form ES-401-5 Question Worksheet Question Source: Bank #

Modified Bank # (Note changes or attach parent)

New X Question History: Last NRC Exam Question Cognitive Level: Memory or Fundamental Knowledge Comprehension or Analysis X 10 CFR Part 55 Content: 55.41 55.43 Comments:

NUREG-1021. Revision 9 2 of 2

ES-401 Sample Written Examination Form ES-401-5 Question Worksheet Examination Outline Cross-reference: Level RO SRO Tier # 2 Group # 2 KIA # 286000 A4.01 Impclrtance Rating 3.3 3.2 286000 Fire Protection System Ability to manually operate andlor monitor in the control room: (CFR:41.7 I 45.5 to 45.8) A4.01 System alarms and indicating lights Proposed Question: # 65 Given the following conditions:

  • The Fire Protection water system has been actuated

.Fire Protection water system pressure ha:; lowered to 85 psig

  • All Fire Protection pumps have functioned as designed

.No operator action has been taken to start Fire Protection pumps Which of the following describes the status of the Fire Protection System water pumps?

A. All three Fire Pumps are running and will shutdown when system pressure exceeds the reset point.

B. All three Fire Pumps are running and will shutdown when system pressure exceeds the reset point.

C. Only the electric motor driven pump (76P-,2)is running and it will shutdown when system pressure exceeds the reset point.

D. Only the diesel driven fire pumps (76P-1 and 76P-4) are running and they will NOT shutdown when system pressure exceeds the reset point.

Proposed Answer: B Explanation (Optional):

TechnicaI Reference(s): (Attach if not previously provided)

Proposed references to be provided to applicants during examination:

Learning Objective: (As available)

Question Source: Bank #

NUREG-1021, Revision 9 1 of2

ES-40 1 Sample Written Examination Form ES-401-5 Question Worksheet Modified Bank # (Note changes or attach parent)

New X Question History: Last NRC Exam Question Cognitive Level: Memory or Fundamiental Knowledge X Comprehension or Analysis 10 CFR Part 55 Content: 55.41 55.43 Comments:

NUREG-1021, Revision 9 2of2

ES-401 Sample Written Examination Form ES-401-5 Question Worksheet Examination Outline Cross-reference: Level RO SRO Tier # 3 Group # 2 K/A I# G 2.2.1 1 Importance Rating 3.0 G 2.1 .I1 Knowledge of less than one hour technical specification action statements for systems.

(CFR:43.2 / 45.1 3)

Proposed Question: # 66 The plant is at 100% power with all systems operating normally. Which one of the following conditions requires Technical Specification action in less than one hour?

A. MSlV 29AOV-80A is declared inoperable.

6. Inoperable control room boundary.

C. Both A and 6 batteries voltage is 2.1 3 volts per connected cell.

D. A and 6 RHR pumps are declared inoperable.

Proposed Answer: B Explanation (Optional):

Techn icaI Reference(s): (Attach if not previously provided)

Proposed references to be provided to applicant:; during examination: TS Table 3.8.6-1 Learning Objective: (As available)

Question Source: Bank #

Modified Bank # (Note changes or attach parent)

New X Question History: Last NRC Exam Question Cognitive Level: Memory or Fundamental Knowledge X Comprehension or Analysis NUREG-1021, Revision 9 1 of2

ES-40 1 Sample Written Examination Form ES-401-5 Question Worksheet 10 CFR Part 55 Content: 55.41 55.43 Comments:

NUREG-I 021, Revision 9 2of2

ES-401 Sample Written Examination Form ES-401-5 Question Worksheet Examination Outline Cross-reference: Level RO SRO Tier # G Group # 2 WA # ~~

G 2.1.33 Importance Rating 3.4 G 2.1.33Ability to recognize indications for system operating parameters which are entry-level conditions for technical specifications. (CFR:43.2/ 43.3/ 45.3)

Proposed Question: # 67 The following plant conditions exist:

  • Plant is at 100% power
  • Scaffolding is being erected in the HPCl room for planned maintenance
  • Fuel sipping is in progress in the fuel pool
  • A liquid waste discharge, per OF49 is in progress
  • It is mid-February and lake temperature is 34°F Which of the following are entry level conditions specified in the Technical Specifications?

A. Annunciator 09-6-1-26 INTAKE STRUCTURE HTRS 6N66 NOT ON is in alarm

9. Annunciator 09-1-19 ARM DOWNSCALE is in alarm C. Air compressor A trips.

D. Diesel fire pump 76P-4 is tagged out for maintenance.

Proposed Answer: A Explanation (Optional):

Technical Reference(s): (Attach if not previously provided)

Proposed references to be provided to applicants during examination:

Learning Objective: (As available)

Question Source: Bank #

Modified Bank # (Note changes or attach parent)

New X NUREG-1021, Revision 9 1 of2

ES-401 Sample Written Examination Form ES-401-5 Question Worksheet Question History: Last NRC Exam Question Cognitive Level: Memory or Fundamlental Knowledge X Comprehension or Analysis 10 CFR Part 55 Content: 55.41 55.43 Comments:

NUREG-1021, Revision 9 2of2

ES-401 Sample Written Examination Form ES-401-5 Question Worksheet Examination Outline Cross-reference: Level RO SRO Tier # 3 Group # 1 WA it# G2.1.18 Importance Rating 2.9 3.0 2.1 . I 8 Ability to make accurate / clear and concise logs / records / status boards / and reports.

(CFR: 45.12 / 45.13)

Proposed Question: # 68 Which ONE of the following describes the correct use of narrative logs as described in EN-OP-107, eSOMS Narrative Log Module Users Manual?

A. Only the Shift Manager can make changes to an approved eSOMs Narrative Log.

8. All operators on the shift may edit their entries on a turned over log.

C. Only the operator who made the entry may edit it prior to the log being turned over.

D. The Shift Manager or the individual who made the entry can make changes to an approved eSOMs Narrative Log.

Proposed Answer: C Explanation (Optional):

A. Incorrect - No changes are permitted by anyone to an approved log.

B. Incorrect - A turned over log can not be changed.

C. Correct - Per EN-OP-107, step 5.15 D. Incorrect - No changes are permitted by anyone to an approved log.

Technical Reference(s): EN-OP-107 R2 (Attach if not previously provided)

Proposed references to be provided to applicants during examination: NONE Learning Objective: LP-AP EO-46.04 (As available)

Question Source: Bank # PNPS Modified Bank # (Note changes or attach parent)

New Question History: Last NRC Exam Question Cognitive Level: Memory or Fundamental Knowledge X NUREG-1021, Revision 9 1 of2

ES-401 Sample Written Examination Form ES-401-5 Question Worksheet Comprehension or Analysis 10 CFR Part 55 Content: 55.41 10 55.43 Comments:

Changed the question from:

Which ONE of the following describes the correct use of narrative logs as described in procedure ODSO-04?

(NOTE use bank choices if plausible, other wise please provide ideas)

A. Only the Shift Manager can make changes to an approved eSOMs Narrative Log.

B. All operators on the shift may edit their entries on a turned over log.

C. The offgoing shift will complete the plant parameters for the next shift.

D. The Shift Manager or the individual who made the entry can make changes to an approved eSOMs Narrative Log.

NUREG-1021, Revision 9 2 of 2

ES-40 1 Sample Written Examination Form ES-401-5 Question Worksheet Examination Outline Cross-reference: Level RO SRO Tier # 3 Group # 2 KIA $# G2.2.30 Importance Rating 3.5 3.3 2.2.30 Knowledge of RO duties in the control room during fuel handling such as alarms from fuel handling area / communication with fuel storage facility / systems operated from the control room in support of fueling operations / and supporting instrumentation. (CFR: 45.12)

Proposed Question: # 69 Per RAP-7.1.04BI Refueling Procedure, which one of the following is a responsibility of a Reactor Operator during core alterations?

A. Verify completion of daily refueling checks.

B. Monitor the Source Range Monitors for rising counts.

C. Ensure the Refueling Checklist is completed daily D. Complete the ICA Transfer Forms and transmit to the Reactor Engineer Proposed Answer: B Explanation (Optional):

A. Incorrect - Responsibility of Refuel Bridge SRO as defined in RAP-7.1.04B Step 6.7 B. Correct - RO responsibility as defined in RAP-7.1.048 Step 5.2 C. Incorrect - Responsibility of Refuel Bridge SRO as defined in RAP-7.1.04B Step 6.7 D. Incorrect - Responsibility of Refuel Bridge SRO as defined in RAP-7.1.04B Step 6.7 Technical Reference(s): RAP-7.1.04B R22 (Attach if not previously provided)

RAP-7.1.04C R5 Proposed references to be provided to applicants during examination: NONE Learning Objective: SDLP-08A EO-I .13.(3 (As available)

Question Source: Bank #

Modified Bank # X (Note changes or attach parent)

New NUREG-I 021, Revision 9 1 of2

ES-401 Sample Written Examination Form ES-401-5 Question Worksheet Quest ion History: Last NRC Exam 2002 Clinton Question Cognitive Level: Memory or Fundamental Knowledge X Comprehension or Analysis 10 CFR Part 55 Content: 55.41 55.43 Comments:

NUREG-I 021, Revision 9 2 of 2

ES-40 1 Sample Written Examination Form ES-40 1-5 Question Worksheet Examination Outline Cross-reference: Level RO SRO Tier # G Group #

WA ## G 2.2.22 Importance Rating 3.4 G. 2.2.22 Knowledge of limiting conditions for operations and safety limits (CFR: 43.2 / 45.2)

Proposed Question: # 70 Determine which of the following combinations of reactor power and reactor pressure constitute a Safety Limit violation.

Reactor power Reactor pressure A. 15% RTP 750 psig

8. 24% RTP 770 p i g C. 28% RTP 775 psig D. 32% RTP 810 psig Proposed Answer: C Explanation (Optional):

Technical Reference(s): (Attach if not previously provided)

Proposed references to be provided to applicants during examination:

Learning Objective: (As available)

Question Source: Bank #

Modified Bank # (Note changes or attach parent)

New X Question History: Last NRC Exam Question Cognitive Level: Memory or Fundamental Knowledge X Comprehension or Analysis NUREG-1021, Revision 9 1of2

ES-401 Sample Written Examination Form ES-401-5 Question Worksheet 10 CFR Part 55 Content: 55.41 55.43 Comments:

NUREG-1021, Revision 9 2of2

ES-401 Sample Written Examination Form ES-401-5 Question Worksheet Examination Outline Cross-reference: Level RO SRO Tier # 3 Group # 3 WA # 2.3.1 Importance Rating 2.6 3.0 2.3.1 Knowledge of 10 CFR: 20 and related facility radiation control requirements.

(CFR: 41.12 / 43.4. 45.9 / 45.10)

Proposed Question: # 71 A pump room has general area dose rates ranging from 10 mrem/hr to 115 mrem/hr.

Average floor contamination levels are 2000 dpm/lOO cm2 gamma.

Which ONE of the following lists the minimum required area posting for the pump room per EN-RP-108, Radiation Protection Posting?

A. Radiation Area and Contamination Area.

B. Radiation Area and High Contamination Area.

C. High Radiation Area and Contamination Area.

D. High Radiation Area and High Contamination Area.

Proposed Answer: C Explanation (Optional):

A. Incorrect - The area requires a High Radiation Area sign even if doses drop at times to 10 mr/hr.

B. Incorrect - A High Contamination Area requires > I 00,000 dpm/lOO cm2 gamma C. Correct - >IO0 mrem/hr makes it a HRA. A contamination level > 1000 dpm/100 cm2 gamma makes it a contamination area.

D. Incorrect - A High Contamination Areal requires >100,000 dpm/100 cm2 gamma Technical Reference(s): EN-RP-108 (Attach if not previously provided)

Proposed references to be provided to applicants during examination: NONE Learning Objective: LP-AP EO-31.02 (As available)

Question Source: Bank #

NUREG-1021, Revision 9 1of2

ES-40 1 Sample Written Examination Form ES-401-5 Question Worksheet Modified Bank # X (Note changes or attach parent)

New Question History: Last NRC Exam 2005 Limerick 1 Q # 72 Question Cognitive Level: Memory or Fundamental Knowledge X Comprehension or Analysis 10 CFR Part 55 Content: 55.41 55.43 Comments: Check WA in sample plan- thought it was 2.2.22 Knowledge of limiting conditions for operations and safety limits. (CFR: 43.2 / 45.2)

Comments: The definition of RCA has changed (it no longer has a dose level requirement).

Increased the dose levels so the area is now a high radiation area. Changed distracters to make them plausible and balanced.

NUREG-1021, Revision 9 2of2

ES-401 Sample Written Examination Form ES-401-5 Question Worksheet Examination Outline Cross-reference: Level RO SRO Tier # G Group # 3 WA iV G 2.3.1 1 Importance Rating 2.7 G 2.3.1 1 Ability to control radiation releases (CFR: 45.9 / 45.10)

Proposed Question: # 72 During a planned discharge of the A Waste Tank, a valve positioning error results in simultaneously discharging the 6 Waste Tank. The specific activity of the 6 Waste Tank is ten times (1OX) higher than the A Waste Tank.

What design feature(s) and/or administrative controls ensure 10CFR20 limits are not exceeded?

A. Per OP-49, three circulating water pumps are required to be running for all liquid radioactive waste discharges.

B. The discharge flow control valve will reduce the flow if activity is > 10-7 pCi/ml.

C. Per OP-49, the tempering gate is always set at 0% for all liquid radioactive waste discharges.

D. Radiation monitor 17RM-350 Hi-Hi setpoint will close valve 20AOV-230 and 20AOV-231 and isolate the discharge.

Proposed Answer: D Explanation (Optional):

Technical Reference(s ): (Attach if not previously provided)

Proposed references to be provided to applicants during examination:

Learning 0bjective: (As available)

Question Source: Bank #

Modified Bank # (Note changes or attach parent)

New X Question History: Last NRC Exam NUREG-1021, Revision 9 1of2

ES-401 Sample Written Examination Form ES-401-5 Question Worksheet Question Cognitive Level: Memory or Fundamental Knowledge X Comprehension or Analysis 10 CFR Part 55 Content: 55.41 55.43 Comments:

NUREG-1021, Revision 9 2 of 2

ES-40 1 Sample Written Examination Form ES-401-5 Question Worksheet Examination Outline Cross-reference: Level RO SRO Tier :# 3 Group # 3 WA # G2.3.10 Importance Rating 2.9 3.3 2.3.10 Ability to perform procedures to reduce excessive levels of radiation and guard against personnel exposure. (CFR: 43.4 / 45. IO)

Proposed Question: # 73 An ALARA work analysis must be performed for a job requiring entry to a High Radiation Area.

0 The worker who will perform the job has received 1650 mr TEDE this year.

0 The worker has no undocumented quarters this year.

The work is in a 400 mr/hr field and will take one hour.

0 Shielding can be placed by the worker to reduce the dose rate by 50%, however, 100 mr total would be received to instalVremove the shielding.

0 A special tool is available which will allow the job to be performed farther from the radiation source in an area that is 200 mdhr without shielding.

0 Using the special tool will extend the job to 1.5 hours5.787037e-5 days <br />0.00139 hours <br />8.267196e-6 weeks <br />1.9025e-6 months <br />.

Which ONE of the following describes:

(1) The method that would result in the lowest worker dose exposure, AND (2) Whether the workers routine annual administrative TEDE limit would be exceeded?

A (1) Use BOTH the shielding and the special tool.

(2) The TEDE limit would NOT be exceeded.

B. (1) Use NO shielding, but use the special tool.

(2) The TEDE limit would NOT be exceeded.

C. (1) Use NO shielding, but use the special tool.

(2) The TEDE limit would BE exceeded.

D. (1) Use BOTH the shielding and the special tool.

(2) The TEDE limit would BE exceeded.

Proposed Answer: A Explanation (Optional):

NUREG-1021, Revision 9 1 of2

ES-401 Sample Written Examinat ion Form ES-401-5 Question Worksheet A. Correct -This would result in a 250 mr dose: 100 mr to installhemove shielding + 150 mr (100 mr X 1.5) by using the special tciol.

B. Incorrect - This would result in a 300 mr dose (200 mr x 1.5)

C. Incorrect - Routine annual admin limit is 2000 mr and would not be exceeded.

D. Incorrect - Routine annual admin limit is 2000 mr and would not be exceeded.

Technical Reference(s): EN-RP-201 R2 (Attach if not previously provided)

Proposed references to be provided to applicants during examination: NONE Learning Objective: LP-AP EO-28.03 (As available)

Question Source: Bank # X Modified Bank # (Note changes or attach parent)

New Question History: Last NRC Exam Question Cognitive Level: Memory or Fundamental Knowledge X Comprehension or Analysis 10 CFR Part 55 Content: 55.41 55.43 Comments: Changed the wording of doseage received when installinghemoving the shielding NUREG-1021, Revision 9 2of2

ES-40 1 Sample Written Examination Form ES-401-5 Question Worksheet Examination Outline Cross-reference: Level RO SRO Tier # 3 Group # 4 KIA :# G2.4.29 Importance Rating 2.6 4.0 2.4.29 Knowledge of the emergency plan. (CFR: 43.5 / 45.1 1)

Proposed Question: # 74 A Site Area Emergency has been declared.

Fifty-five minutes later, all required emergency response facilities are manned and the EOF is operational.

Per the JAF Emergency Plan, which ONE of the following individuals is responsible for the management of onsite response?

A. Shift Manager as the Emergency Director

6. Site Duty Manager as the Emergency Director C. Technical Support Center Manager D. Emergency Plant Manager Proposed Answer: D Explanation (Optional):

A. Incorrect - The SM, acting as the ED, did have this responsibility but is no longer the PED if the EOF is operational.

6. Incorrect - The Site Duty Manager makes notifications to corporate and does not act as the ED.

C. Incorrect - The TSC Manager can act as the ED but, with the EOF operational, the ED duties are assumed at the EOF.

D. Correct - The Emergency Plant Manager can direct onsite plant operations.

Technical Reference(s): EAP-14.2 R24 (Attach if not previously provided)

E-plan Section 5 R4 Proposed references to be provided to applicants during examination: NONE Learning Objective: EP-12.4.2 EO-3.01 (As available)

Question Source: Bank #

Modified Bank # X (Note changes or attach parent)

NUREG-1021, Revision 9 1 of2

ES-40 1 Sample Written Examination Form ES-401-5 Question Worksheet New Question History: Last NRC Exam W 2005 - Q # 75 Question Cognitive Level: Memory or Fundamental Knowledge X Comprehension or Analysis 10 CFR Part 55 Content: 55.41 55.43 Comments: Changed the original question to reflect JAF: Changed the word activated to operational. This word denotes that the ED authority has been transferred to the EOF.

Changed titles to reflect JAF E-plan titles.

A Site Area Emergency has been declared.

Fifty-five minutes later, all required emergency response facilities are manned and the EOF is activated.

Per procedure XXXXYZZ, which ONE of the following individuals is responsible for coordinating the overall emergency response?

A. Shift Manager as the Plant Emergency Director (PED)

B. Duty call Officer as the Plant Emergency Director (PED)

C. Technical Support Center (TSC) Coordinator D. Site Recovery Manager (SRM)

NUREG-I 021, Revision 9 2of2

ES-401 Sample Written Examination Form ES-401-5 Question Worksheet Examination Outline Cross-reference: Level RO SRO Tier # 3 Group # 4 KIA ;# G2.4.18 Importance Rating 2.7 3.6 2.4,18 Knowledge of the specific bases for EOPs. (CFR: 41.10 / 45.13)

Proposed Question: # 75 EOP-3 has the operator verify recirculation runback to minimum prior to tripping the recirculation pumps.

Per EOP-3 basis, the reason for this is to prevent:

A. MSlV closure.

6. Main Turbine trip.

C. Condensate Pump trip.

D. A rapid pressure increase which may lift a relief valve.

Proposed Answer: 6 Explanation (Optional): In order to effect a rapid power reduction, if the main turbine is on line, the RWR pumps are ran back to minimum speed. This increases the void production in the core which increases the slowing down length of the rieutrons increasing the chances of resonance capture, thereby adding negative reactivity to shutdown the reactor. If RWR pumps are tripped from too high a speed, a resulting level swell could result in a trip of the Main turbine affecting the main condenser heat sink ability to remove all the heat generated from the Rx, would be limited to just the main turbine bypass valve capacity as the major heat rejection method which could ultimately result in challenges to the containment from heat addition from SRVs..

A. Incorrect - The MSlVs close on a low level, not a high level.

B. Correct - The runback of the recirc pumps minimizes the vessel level swell after the trip.

The swell could result in the loss of the turbine/generator system.

C. Incorrect - The feedwater pumps trip on ia high reactor level, not the condensate pumps.

D. Incorrect - The runback inserts negative reactivity, which lowers power and will cause a decrease in pressure.

Technical Reference(s): BWROG EPG/SAG Guidelines (Attach if not previously provided)

Proposed references to be provided to applicants during examination: NONE Learning Objective: MIT-301.I1D EO-1.07 (As available)

NUREG-1021, Revision 9 1 of2

ES-401 Sample Written Examination Form ES-401-5 Question Worksheet Question Source: Bank #

Modified Bank # (Note changes or attach parent)

New X Question History: Last NRC Exam Question Cognitive Level: Memory or Fundamental Knowledge Comprehension or Analysis 10 CFR Part 55 Content: 55.41 55.43 Comments: Changed stem word you to the operator. Changed choice C to make it incorrect.

NUREG-1021. Revision 9 2of2

ES-401 Sample Written Examination Form ES-401-5 Question Worksheet Examination Outline Cross-reference: Level RO SRO Tier :# 1 Group # 1 KIA #! 295034 G 2.2.13 Impalrtance Rating 3.8 295004 Partial or Complete Loss of DC Power Knowledge of tagging and clearance procedures. (CF:R:41 . I O / 45.13)

Proposed Question: # 76 The plant is at 100% power.

A ground of 75 VDC is present on 71DC-A2. During implementation of AOP-22, DC Power System Ground Isolation, circuit 9 of 71 DC-A2 iis identified as the source of the ground. A tagging order has been prepared to OPEN circuit 9 and as FSS you have been given the tagging order.

1.) Should you approve the tagging order?

2 . ) What is your basis for approvalldisapproval.

1.) 2.)

A. Yes Equipment affected will only result in entering LCO B. No Equipment affected will cause EDG start C. Yes Equipment affected is not required for the existing mode.

D. No Equipment affected will cause a full scram.

Proposed Answer: A Explanation (Optional):

Technica I Reference(s): (Attac,, if not previousj provica Proposed references to be provided to applicants during examination: List of circuits powered from 71DC-A2 Learning Objective: (As available)

Question Source: Bank #

Modified Bank # (Note changes or attach parent)

NUREG-1021, Revision 9 1 of2

ES-401 Sample Written Examination Form ES-401-5 Question Worksheet New X Question History: Last NRC Exam Question Cognitive Level: Memory or Fundam~entalKnowledge Comprehension or Analysis X 10 CFR Part 55 Content: 55.41 55.43 (2)(3)

Comments:

NUREG-1021, Revision 9 2of2

ES-401 Sample Written Examination Form ES-401-5 Question Worksheet Examination Outline Cross-reference: Level RO SRO Tier # 1 Group # 1 WA i# 295006 G 2.4.27 Importance Rating 3.5 295006 SCRAM G 2.4.27 Knowledge of fire in the plant procedure. (CFR:41. I O / 43.5 / 45.13)

Proposed Question: # 77 The plant is at 100% power. The electrical distribution system is in the normal lineup.

An I&C surveillance being performed on the E APRM has generated a ?hscram.

0 Fire in the East Electric Bay causes de-energization of the B RPS MG Set Supply Breaker.

Control rod 26-27 remains at position 48.

0 All other equipment functions as designed As CRS what action(s) do you direct the crew to take for these conditions?

A. Bypass the E APRM, reset the scram B. Place the B RPS on alternate power supply, reset the scram C. Execute AOP-1.

D. Evacuate the Control Room and continue actions at the Remote Shutdown Panel Proposed Answer: C Explanation (Optional):

Tech nicaI Reference(s): (Attach if not previously provided)

Proposed references to be provided to applicants during examination:

Learning Objective: (As available)

Question Source: Bank #

Modified Bank # (Note changes or attach parent)

New X NUREG-1021, Revision 9 1 of2

ES-40 1 Sample Written Examination Form ES-401-5 Question Worksheet Question History: Last NRC Exam Question Cognitive Level: Memory or Fundamental Knowledge Comprehension or Analysis X 10 CFR Part 55 Content: 55.41 55.43 Comments:

NUREG-1021, Revision 9 2of2

Examination Outline Cross-reference: Level RO SRO Tier # 1 Group # 1 WA ;B 295016 EA2.01 Importance Rating 3.6 295016 Control Room Abandonment G. 2.4.30 Knowledge of which events related to system operationslstatus should be reported to outside agencies. (CFR: 43.5 / 45.1 1)

Proposed Question: # 78 The plant is at 100% power when the following occurs:

  • An NPO reports a significant fire in the Relay Room.
  • The reactor is automatically scrammed.
  • ID RHR pump has lost control room indication.
  • B RHR pump has spuriously started.

Per EAP-1.1, 1.) Which of the following agencies must be notified FIRST? AND 2.) What is the time requirement for the reiport?

1.I 2.)

A. New York State One hour B. New York State Fifteen minutes C. NRC One Hour D. NRC Fifteen minutes Proposed Answer: B Explanation (Optional):

Technical Reference(s): AOP-43 (Attach if not previously provided)

Proposed references to be provided to applicants during examination:

Learning Objective: (As available)

Question Source: Bank #

NUREG-1021, Revision 9 1 of2

ES-401 Sample Written Examination Form ES-401-5 Question Worksheet Modified Bank # (Note changes or attach parent)

New Question History: Last NRC Exam Question Cognitive Level: Memory or Fundamlental Knowledge Comprehension or Analysis 10 CFR Part 55 Content: 55.41 55.43 (1)(2)

Comments:

NUREG-1021, Revision 9 2of2

ES-40 1 Sample Written Examination Form ES-401-5 Question Worksheet Examination Outline Cross-reference: Level RO SRO Tier # 1 Group # 1 WA # 295021 AA2.07

~~

Importance Rating 3.1 295034 Ability to determine andlor interpret the following as they apply to LOSS OF SHUTDOWN COOLING: (CFR:41 . I O / 43.5 / 45.13) AA2.07 Reactor recirculationflow.

Proposed Question: # 79 The plant is in Mode 3. The following plant condiitions exist:

  • RPV pressure is 40 psig

-RPV water level is 200 inches

  • A RWR pump is operating
  • RWCU is in normal operation A loss of bus 10300 occurs, due to a bus lockout.

All automatic actions function as designed.

1.) What parameter do you direct the crew to control to ensure adequate temperature monitoring of the reactor core?

2.) What procedure is utilized for control of the selected parameters?

1.> 2.)

A. Shutdown cooling flow > 6000 gpm OP-I 3D

8. RPV water level is >200 inches AOP-30 C. Shutdown cooling flow is > 4000 gpm OP-I 3D D. RPV water level is > 240 inches AOP-30 Proposed Answer: D Explanation (Optional):

TechnicaI Reference(s): (Attach if not previously provided)

Proposed references to be provided to applicants during examination:

NUREG-1021, Revision 9 1 of2

ES-40 1 Sample Written Examination Form ES-401-5 Question Worksheet Examination Outline Cross-reference: Level RO SRO Tier # 1 Group # 1 WA ## 295021 AA2.07 Importance Rating 3.1 295034 Ability to determine and/or interpret the following as they apply to LOSS OF SHUTDOWN COOLING: (CFR:41. I O / 43.5 / 45.13) AA2.07 Reactor recirculation flow.

Proposed Question: # 79 The plant is in Mode 3. The following plant condntions exist:

  • RPV pressure is 40 psig
  • RPV water level is 200 inches
  • A RWR pump is operating
  • RWCU is in normal operation A loss of bus 10300 occurs, due to a bus lockout.

All automatic actions function as designed.

1.) What parameter do you direct the crew to control to ensure adequate temperature monitoring of the reactor core?

2.) What procedure is utilized for control of the selected parameters?

1.) 2.)

A. Shutdown cooling flow > 6000 gpm OP-13D

8. RPV water level is >ZOO inches AOP-30 C. Shutdown cooling flow is > 4000 gpm OP-I 3D D. RPV water level is > 240 inches AOP-30 Proposed Answer: D Explanation (Optional):

Technical Reference(s): (Attach if not previously provided)

Proposed references to be provided to applicants, during examination:

NUREG-1021, Revision 9 1 of2

ES-401 Sample Written Examination Form ES-401-5 Question Worksheet Learning Objective: (As available)

Question Source: Bank #

Modified Bank # (Note changes or attach parent)

New X Question History: Last NRC Exam Question Cognitive Level: Memory or Fundamental Knowledge Comprehension or Analysis X 10 CFR Part 55 Content: 55.41 55.43 (5)

Comments:

NUREG-1021, Revision 9 2 of 2

ES-40 1 Sample Written Examination Form ES-401-5 Question Worksheet Examination Outline Cross-reference: LeveI RO SRO Tier :# 1 Groiiip # 1 KIA l'f 295024 G2.2.17 Importance Rating 3.4 295024 High Drywell Pressure Knowledge of the process for managing maintenance! activities during power operations. (CFR: 43.5 /

45.1 3)

Proposed Question: # 80 The plant is at 100% power. Several I&C jobs are currently active. One crew of I&C personnel is all that is available to perform work.

Given the I&C resource limitations, which of the following job activities must receive the highest work priority?

A. Drywell high pressure input to RPS requires repair.

B. An ST on low level ECCS initiation drops dead in three days.

C. 'A' Service Air compressor failure has occurred due to a failed control board.

D. Calibration of 'A' Waste Sample tank level ,transmitter is due today.

Proposed Answer: A Explanation (Optional TechnicaI Reference(s): (Attach if not previously provided)

Proposed references to be provided to applicants during examination:

Learning Objective: (As available)

Question Source: Bank #

Modified Bank # (Note changes or attach parent)

New X Question History: Last NRC Exam NUREG-1021, Revision 9 1 of2

ES-40 1 Sample Written Examination Form ES-401-5 Question Worksheet Question Cognitive Level: Memory or Fundamental Knowledge Comprehension or Analysis X~

10 CFR Part 55 Content: 55.41 55.43 (3)

Comments:

NUREG-1021, Revision 9 2of2

ES-401 Sample Written Examination Form ES-401-5 Question Worksheet Examination Outline Cross-reference: Level RO SRO Tier i# 1 Group # 1 KIA ## 295028 EA2.05 lmpartance Rating 3.8 295028 Ability to determine and/or interpret the following as they apply to HIGH DRYWELL TEMPERATURE: (CFR:41. I O / 43.5 / 45.1 3) EA2.05 Toruslsuppression chamber pressure: Plant Specific Proposed Question: # 81 An ATWS and a small break loss of coolant accident (SBLOCA) have occurred. The crew first addressed the ATWS, and now all rods are in. Current plant conditions are:

  • Torus pressure 27 psig
  • Torus level 14 ft
  • Drywell temperature 220°F
  • Drywell pressure 30 psig
  • RPV pressure 700 psig and slowly lowerinlg
  • RPV level 100 inches and slowly rising With these conditions, the CRS must direct (1) . As spray(s) reduces drywell temperature, the torus pressure will (2)-- than drywell pressure.

(1) (2)

A. Drywell spray only eventualIy be higher B, Torus and drywell spray eventuall,y be higher C. Drywell spray only remain lower D. Torus and drywell spray remain lower Proposed Answer: B Explanation (Optional):

Technical Reference(s): EOP-4 (Attach if not previously provided)

EOP-11 Proposed references to be provided to applicants during examination:

Learning Objective: (As available)

NUREG-1021, Revision 9 1 of2

ES-401 Sample Written Examination Form ES-401-5 Question Worksheet Question Source: Bank #

Modified Bank # (Note changes or attach parent)

New X Question History: Last NRC Exam Question Cognitive Level: Memory or Fundamental Knowledge Comprehension or Analysis X 10 CFR Part 55 Content: 55.41 55.43 (5)

Comments:

NUREG-1021, Revision 9 2of2

ES-40 1 Sample Written Examination Form ES-401-5 Question Worksheet Examination Outline Cross-reference: Level RO SRO Tier # 1 Group # 1 WA # 295030 EA2.04 Importance Rating 3.7 295030 Ability to determine and/or interpret the following as they apply to LOW SUPPRESSION POOL WATER LEVEL: (CFR:41. I O 1 43.5 1 45.1 3) EA2.04 C)rywell/suppression chamber differential pressure:

Mark 1&11.

Proposed Question: # 82 The following plant conditions exist:

-RPV level is -10 inches and steady

-RPV Pressure is 850 psig and steady

-Torus level is 9.5 ft and slowly decreasing

-Four control rods are at position 04

-Reactor Power is on range 4 of IRMs

-Torus temperature is 155°F and slowly rising

  • RCIC is injecting

-SLC is injecting with SLC tank level 70% and lowering

-All low pressure ECCS systems are running Terminate and prevent IAW EP-5 has beeri completed for HP and LP systems Which ONE of the following is the:

1.) Required action 2.) Its basis?

1.)REQUIRED ACTION 2.) BASIS A. Emergency Depressurize the RPV by maintain the pressure suppression opening 7 ADS valves feature of the torus.

6. Emergency Depressurize the RPV by adequate core cooling has been lost.

opening 7 ADS valves.

C. Anticipate Emergency Depressurization of: maintain the pressure suppression the RPV by opening Bypass Valves feature of the torus.

D. Anticipate Emergency Depressurization of adequate core cooling has been lost.

the RPV by opening Bypass valves.

Proposed Answer: A NUREG-1021, Revision 9 1 of2

ES-401 Sample Written Examination Form ES-401-5 Question Worksheet Explanation (Optional):

Technical Reference(s): EOP-2, EOP-3, (Attach if not previously provided)

EOP-4, EOP-11 Proposed references to be provided to applicants during examination:

Learning Objective: (As available)

Question Source: Bank #

Modified Bank # (Note changes or attach parent)

New X Question History: Last NRC Exam Question Cognitive Level: Memory or Fundamental Knowledge Comprehension or Analysis X 10 CFR Part 55 Content: 55.41 55.43 (5)

Comments:

NUREG-1021, Revision 9 2of2

ES-401 Sample Written Examination Form ES-401-5 Question Worksheet Examination Outline Cross-reference: LeveI RO SRO Tier :# 1 Group # 2 WA # 295002 G2.4.11 Impclrtance Rating 3.4 3.6 295002 Loss of Main Condenser Vacuum - 2.4.1 1 Knowledge of abnormal condition procedures. (CFR: 41.10 / 43.5 I45.13)

Proposed Question: # 83 Main turbine load is 213 MWe with the 09-5-1-52 TCV FAST CLOSURE & TSV TRIP BYPASSED annunciator in alarm.

Main Condenser vacuum is 24.5 inches Hg and steady.

1.) What direction do you provide to the crew to implement immediately?

2.) What procedure provides the applicable direction?

14 2.)

A. Place spare steam jet air ejectors in service OP-24C, Condenser Air Removal

6. Start condenser air removal pumps. OP-24C, Condenser Air Removal C. Trip the Reactor Feed Pump Turbines. AOP-41, Feedwater Malfunction (Rising FW Flow- High RPV water Level)

D. Trip the Main Turbine. AOP-31, Loss of Condenser Vacuum Proposed Answer: D Explanation (Optional):

A. Incorrect - Subsequent action in AOP-31

.B. Incorrect - Subsequent action in AOP-31 C. Incorrect -RFPTs trip on low vacuum but are not required to be manually tripped.

D. Correct - Vacuum is low enough (<25) and turbine load is low enough (~255MWe)to require a manual turbine trip.

Technical Reference(s): AOP-31 R17 (Attach if not previously provided)

Proposed references to be provided to applicants during examination: NONE Learning 0bjective: LP-AOP EO-1.03.A (As available)

Question Source: Bank #

NUREG-1021. Revision 9 1 of2

~ ~~ ~~

ES-40 1 Sample Written Examination Form ES-401-5 Question Worksheet Modified Bank # (Note changes or attach parent)

New X Question History: Last NRC Exam Question Cognitive Level: Memory or Fundamental Knowledge X Comprehension or Analysis 10 CFR Part 55 Content: 55.41 55.43 (5)

Comments: The turbine trip requirement is an override that is in both the immediate and subsequent actions sections.

Added immediate action to stem because choices A and C would be correct (they are required subsequent actions).

Changed choice C from scramming and tripping the main turbine to tripping the RFPTs. A scram is not a wrong choice even if not directly called for in AOP-31.

NUREG-1021, Revision 9 2of2

ES-401 Sample Written Examination Form ES-401-5 Question Worksheet Examination Outline Cross-reference: Level RO SRO Tier # 1 Group # 2 WA # 295009 M 2 . 0 2 Importance Rating 3.7 295009 Ability to determine and/or interpret the following as they apply to LOW REACTOR WATER LEVEL: (CFR: 41 .I 0 / 43.5 / 45.1 3) AA2.02 Steam flow/feed flow mismatch.

Proposed Question: # 84 The reactor is at 100% power when a feedwater controller failure results in feedwater flow at 0%

with steam flow at 100°/~.

1.) Assuming operator action, but all automatic actions function, what will be the lowest RPV water level observed in the first two minutes?

2.) What procedure provides the appropriate subsequent operator action?

14 2.)

A. 189 inches AOP-42, Feedwater Malfunction (Lowering FW Flow)

B. 177 inches AOP-1, Reactor Scram C. 90 inches EOP-2, RPV Control D. 0 inches EOP-2, RPV Control Proposed Answer: C Explanation (Optional):

Technical Reference(s): (Attach if not previously provided)

Proposed references to be provided to applicants during examination:

Learning Objective: (As available)

Question Source: Bank #

NUREG-1021, Revision 9 1 of2 I

ES-401 Sample Written Examination Form ES-401-5 Question Worksheet Modified Bank ## (Note changes or attach parent)

New X Question History: Last NRC Exam Question Cognitive Level: Memory or Fundamental Knowledge Comprehension or Analysis X 10 CFR Part 55 Content: 55.41 55.43 (5)

Comments:

NUREG-1021, Revision 9 2 of 2

ES-401 Sample Written Examination Form ES-401-5 Question Worksheet Examination Outline Cross-reference: Level RO SRO Tier #f 1 Group # 2 WA ## 295035 EA2.01

~

Importance Rating 39:

295035 Ability to determine and/or interpret the following as they apply to SECONDARY CONTAINMENT HIGH DIFFERENTIAL PRESSURE: (CFR: 41.8 TO 41 . I O ) EA2.01 Secondary containment pressure:

Plant-Specific.

Proposed Question: ## 85 Reactor Building (RB) ventilation is in normal operation, when a trip of all supply fans occurs.

NO automatic actions occur.

1.) What action do you direct the crew to take?

2.) What is the purpose of this action?

1.) 2.)

A. Start SBGT Increase building AP B. TripSBGT Increase building AP C. Trip all RB exhaust fans Decrease building AP D. Isolate RB ventilation Decrease building AP Proposed Answer: D Explanation (Optional):

Technical Reference(s): (Attach if not previously provided)

Proposed references to be provided to applicants during examination:

Learning 0bjective: (As available)

Question Source: Bank #

Modified Bank # (Note changes or attach parent)

New x NUREG-1021, Revision 9 1 of2

ES-401 Sample Written Examination Form ES-401-5 Question Worksheet Question History: Last NRC Exam Question Cognitive Level: Memory or Fundamental Knowledge Comprehension or Analysis X 10 CFR Part 55 Content: 55.41 55.43 (5)

Comments:

NUREG-1021, Revision 9 2of2

ES-40 1 Sample Written Examination Form ES-401-5 Question Worksheet Examination Outline Cross-reference: Level RO SRO Tier 17 2 Group # 1 KIA #I 203000 A2.09 Importance Rating 3.4 203000 Ability to (a) predict the impacts of the following on the RHR/LPCI:INJECTIONMODE (PLANT SPECIFIC); and (b) based on those predictions, use procedures to correct, control, or mitigate the consequences of those abnormal conditions or operations: (CFR:41.5 / 45.6) A2.09 inadequate system flow Proposed Question: # 86 A loss of coolant accident (LOCA) and Loss of Offsite Power (LOOP) has occurred with the existing plant conditions as follows:

  • B and D EDGs failed to start
  • Reactor pressure 50 psig and steady
  • RPV level + 1 inch and slowly decreasing
  • Drywell pressure 25 psig and slowly decreasing
  • B RHR pump tripped on overcurrent
  • A LPCl loop injection flow 5900 gpm
  • A DrywelVTorus Spray flow 3000 gpm The crew has implemented EOP-2 and EOP-4 to control RPV level and containment pressure, respectively.

1.) What action is required to address both EOPs, in the correct priority? AND 2.) What is the basis for thidthese actions?

1.) 2.)

A. Increase LPCl flow by terminating Submergence adequate core cooling DrywelVTorus Spray flow takes precedence over containment sprays B. Increase Drywell Spray flow Containment takes precedence over submergence adequate core cooling C. Increase both LPCl and Drywell Spray High Drywell pressure (>20 psig) allows increased RHR flow D. Decrease both LPCl and Drywell Spray RHR has exceeded its design flow NUREG-I 021, Revision 9 1 of2

EST40 1 Sample Written Examination Form ES-401-5 Question Worksheet Proposed Answer: A Explanation (Optional):

Technical Reference(s): (Attach if not previously provided)

Proposed references to be provided to applicants during examination:

Learning Objective: (As available)

Question Source: Bank #

Modified Bank # (Note changes or attach parent)

New X Question History: Last NRC Exam Question Cognitive Level: Memory or Fundamental Knowledge Comprehension or Analysis X 10 CFR Part 55 Content: 55.41 55.43 (5)

Comments:

NUREG-1021, Revision 9 2 of 2

ES-401 Sample Written Examination Form ES-401-5 Question Worksheet Examination Outline Cross-reference: Level RO SRO Tier i$ 2 Group ## 1 KIA ## 209001 A2.03 lmpartance Rating 3.6 209001 Ability to (a) predict the impacts of the followiing on the LOW PRESSURE CORE SPRAY SYSTEM; and (b) based on those predictions, use procedures to correct, control, or mitigate the consequences of those abnormal conditions or operations: (CFR:41.5 I 45.6) A2.03 A.C. failures Proposed Question: # 87 The plant has experienced a loss of coolant accident(LOCA) with a 100% guillotine break on the B recirc loop discharge line, on the RPV side of the 02MOV-53 B, Recirc Pump B Disch Is01 Valve. The following plant conditions exist:

  • A loop RHR is injecting at 22,000 gpm
  • B loop RHR is spraying the drywell
  • RPV level is 0 inches and lowering Subsequently, a loss of L15 bus occurs.

Which of the following EOP directions will turn, or limit, the lowering RPV level? (Assume ONLY control room actions taken).

A. Maximize A loop RHR loop LPCl injection B. Realign and maximize B loop RHR LPCl injection C. Maximize A Core Spray injection D. Maximize B Core Spray injection Proposed Answer: D Explanation (Optional):

Technical Reference(s): (Attach if not previously provided)

Proposed references to be provided to applicants during examination:

Learning Objective: (As available)

NUREG-1021, Revision 9 1 of2

ES-401 Sample Written Examination Form ES-401-5 Question Worksheet Question Source: Bank #

Modified Bank # (Note changes or attach parent)

New X Question History: Last NRC Exam Question Cognitive Level: Memory or Fundamental Knowledge Comprehension or Analysis X 10 CFR Part 55 Content: 55.41 55.43 (5)

Comments:

NUREG-1021, Revision 9 2of2

ES-401 Sample Written Examination Form ES-401-5 Question Worksheet Examination Outline Cross-reference: Level RO SRO Tier i# 2 Group ## 1 KIA Jf 272000 G2.4.21 I mpalrtance Rating 3.7 4.3 21 5004 Source Range Monitor (SRM) System - 2.4.21 Knowledge of the parameters and logic used to assess the status of safety functions including: 1. Reactivity control 2. Core cooling and heat removal 3. Reactor coolant system integrity 4. Containment conditions 5. Radioactivity release control. (CFR: 43.5 / 45.12)

Proposed Question: # 88 While operating at 100% power, a station blackout occurred. Approximately 30 minutes later plant parameters are as follows:

All control rods except rod 26-27 have been determined to be full in 0 RPV pressure is 1000 psig and steady 0 Drywell pressure is 1.7 psig and steady 0 RClC is operating at 100 gpm 0 RPV level is indeterminate 0 All SRMs are indicating between 1,000 and 100,000 cps and lowering APRMs are downscale 1.) Which of the following best describes core status? The reactor...

2.) What procedure provides the appropriate direction 1.I 2.)

A. is NOT shutdown and adequate core cooliing is EOP-3, Failure to Scram NOT assured.

6. is shutdown and adequate core cooling is NOT EOP-7, RPV Flooding assured.

C. is shutdown and adequate core cooling is AOP-1, Reactor Scram assured.

D. is NOT shutdown and adequate core cooling is EOP-3, Failure to Scram assured.

Proposed Answer: 6 Explanation (Optional):

NUREG-1021, Revision 9 1 of2

ES-40 1 Sample Written Examination Form ES-401-5 Question Worksheet A. Incorrect - The Rx is shutdown with downscale APRMs and decreasing SRM count rate.

There is no ACC if level can not be determined (must be > or = TAF to insure ACC)

B. Correct - The Rx is shutdown with downscale APRMs and decreasing SRM count rate.

There is no ACC if level can not be determined (must be > or = TAF to insure ACC)

C. Incorrect - The Rx is shutdown with downscale APRMs and decreasing SRM count rate.

There is no ACC if level can not be determined (must be > or = TAF to insure ACC)

D. Incorrect - The Rx is shutdown with downscale APRMs and decreasing SRM count rate.

There is no ACC if level can not be determined (must be > or = TAF to insure ACC)

Technical Reference(s): EP-1 R9 (Attach if not previously provided)

Proposed references to be provided to applicants during examination: NONE Learning Objective: MIT-301.I1D EO- 1.03 (As available)

Question Source: Bank #

Modified Bank # (Note changes or attach parent)

New X Question History: Last NRC Exam Question Cognitive Level: Memory or Fundamental Knowledge Comprehension or Analysis X 10 CFR Part 55 Content: 55.41 55.43 5 Comments: Changed stem word increasing to decreasing.

NUREG-1021, Revision 9 2 of 2

E$-401 Sample Written Examination Form ES-401-5 Question Worltsheet Examination Outline Cross-reference: LeveI RO SRO Tier # 2 Group # 1 WA 4t 218000 A2.04 Importance Rating 4.1 4.2*

218000 Automatic Depressurization System - A 2 Ability to (a) predict the impacts of the following on the AUTOMATIC DEPRESSURIZATION SYSTEM; and (b) based on those predictions, use procedures to correct, control, or mitigate the consequences of those abnormal conditions or operations: (CFR: 41.5 / 45.6) A2.04 ADS failure to initiate Proposed Question: # 89 The plant has scrammed from 100% power. Plaint parameters at T=O were as follows:

0 HPCl has isolated from high area temperature 0 Reactor water level is -6 inches and lowering slowly Drywell pressure is 14 psig and steady 0 All Core Spray and RHR pumps are running on minimum flow 0 Reactor pressure is 650 psig and lowering slowly 0 Annunciator 09-4-1-28, ADS TIMERS ACTUATED, alarm is lit Plant parameters at T=5 minutes are as follows:

0 HPCl remains isolated 0 Reactor water level is -10 inches and continues to lower slowly 0 Drywell pressure is 14 psig and steady All Core Spray and RHR pumps are running on minimum flow 0 Reactor pressure is 615 psig and lowering slowly 0 Annunciator 09-4-1-28, ADS TIMERS ACTUATED, alarm is lit Per EN-OP-115, Conduct of Operations, what immediate action is required to insure Adequate Core Cooling? (Assume previous procedures entered NOR any operator actions taken).

A. Open all Turbine Bypass Valves.

B. Open all ADS valves C. Override ADS and attempt to restart HPCl D. Override ADS and establish normal RPV coo'ldown at <IO0 deg. F/hr Proposed Answer: B Explanation (Optional):

NUREG-1021, Revision 9 1 of2

ES-401 Sample Written Examination Form ES-401-5 Question Worksheet A. Incorrect - Using the bypass valves is an action that would need to be directed from the EOPs

6. Correct - reactor pressure indicates that ADS has failed to automatically open and is needed to insure ACC (inject with LP ECCS).

C. Incorrect - Overriding ADS is an action that would need to be directed from the EOPs D. Incorrect - Overriding ADS is an action that would need to be directed from the EOPs Technical Reference(s): EN-OP-115 (Attach if not previously provided)

Proposed references to be provided to applicant!; during examination: NONE Learning Objective: LP-AP EO-46.03 (As available)

Question Source: Bank #

Modified Bank # (Note changes or attach parent)

New X Question History: Last NRC Exam Question Cognitive Level: Memory or Fundamental Knowledge Comprehension or Analysis 10 CFR Part 55 Content: 55.41 55.43 (5)

Comments: ADS should have opened all ADS valves. EN-OP-115 directs licensed operators to initiate operation of ECCS when operating parameters exceed ECCS actuation setpoints and automatic action does not occur.

Changed the wording of the last sentence in the stem. Otherwise, the other choices might be valid. EN-OP-115 directs that operators adhere io all procedures and the assumption could be made that the operator was at some point in the EOPs where these actions are directed. Also added to insure Adequate Core Cooling. The clonduct of OPS procedure (EN-OP-115) is very large and complex. In an exam challenge, an operator probably could find many sections that he could construe to show his answer was right. With level less that TAF and no steam cooling (as shown by fairly constant vessel pressure) there could be no other right answer than to open the ADS valves.

Changed the word inhibit to override to match wording in the JAF EOPs.

NUREG-1021. Revision 9 2of2

ES-401 Sample Written Examination Form ES-401-5 Question Worksheet Examination Outline Cross-reference: Level RO SRO Tier $# 2 Group # 1 WA # 262001 G2.1.6

~~ ~

Importance Rating 2.1 4.3 262001 A.C. Electrical Distribution - 2.1.6 Ability to supervise and assume a management role during plant transients and upset conditions. (CFR: 43.5 145.12 145.13)

Proposed Question: # 90 While operating at power, a loss of off-site power (LOOP) occurs.

The following conditions exist:

  • The LOOP is expected to continue for several hours.
  • The Shift Manager was touring the RCA, is now contaminated and will take at least 20 minutes to return to the control room.

@TheSTA is in the control room.

As the CRS, what are your responsibilities, 1AW EAP-1.I, Off-site Notifications?

A. Classify the event; assign off-site notifications to the STA.

6. Assign EAL determination and offsite notifications to the STA.

C. Assume Emergency Director responsibilities and assign the control room communication aide to notify NY State that an Notification of Unusual Event (NUE) exists.

D. Assume Emergency Director responsibilities. Make required notifications once the SM has returned to the control room and concurred with the emergency classification.

Proposed Answer: C Explanation (Optional): Per EN-OP-115, CRS Responsibilities [8] CRS during a transient should not become in a single activity that distracts from the rest of operations in the control room. (e)

Assumes the roles and responsibilities of the SM if the SM becomes incapacitated. Immediately Notify Operations Management. [I I]STA (d) shall not be assigned tasks that detract from the STA role during abnormal or emergency conditions.

NUREG-1021, Revision 9 1 of2

ES-401 Sample Written Examination Form ES-401-5 Question Worksheet A. Incorrect - Per EAP-I., assigns control room communication aide to make notifications to ensure CRS is not distracted from event response.

B. Incorrect -Per EN-OP-115, Would result in distracting the STA from primary duties to monitor plant response if STA made the inotifications. Communications Aide (NPO) is designated for this activity, STA does provide backup on EAL determination making recommendations.

C. Correct - Per EN-OP-115, CRS assumes SM role in event SM is not available.

Communications Aide (NPO) is designated to make offsite notifications per EAP-1.1.

D. Incorrect - Communications Aide (NPO) is designated to make offsite notifications per EAP-1.1 and time to make notifications would be exceeded along with distracting CRS in event response.

Technical Reference(s): EN-OP-115 R 5, AP-12.11 R7 (Attach if not previously provided)

EAP-1. I R 61, IAP-21 R 27 Proposed references to be provided to applicants during examination: IAP-2 R 27 Learning Objective: (As available)

Question Source: Bank #

Modified Bank # (Note changes or attach parent)

New X Question History: Last NRC Exam Question Cognitive Level: Memory or Fundamental Knowledge Comprehension or Analysis 10 CFR Part 55 Content: 55.41 55.43 (11 Comments: Changed the SM in the stem to will take > 15 minutes to return as normally can return to control room within 15 minutes from site locations. Unable to provide a location > 15 minutes.

Changed to; The Shift Manager was touring the RCA as is contaminated and will take greater than 15 minutes to return to the control room Added that event occurs on a backshift - 01 :[DO am Sunday NUREG-1021, Revision 9 2of2

ES-401 Sample Written Examination Form ES-401-5 Question Worksheet Examination Outline Cross-reference: Level RO SRO Tier # 2 Groulp # 2 KIA #i G 2.4.50 Impoiqance Rating 3.3 G 2.4.50 Ability to verify system alarm setpoints and operate controls identified in the alarm response manual.

Proposed Question: # 91 The plant is at 100% power with all systems nornial.

The following alarm is received: 09-5-1-38 RX PRESS ALARM HI.

NO automatic actions have occurred.

1.) What is the alarm setpoint?

2.) What is the required action?

1.) 2-)

A. 1051 psig Reduce reactor pressure using Bypass Valves

6. 1080 psig Reduce EHC Pressure setpoint C. 1051 psig Reduce EHC Pressure setpoint D. 1080 psig Insert manual scram Proposed Answer: C Explanation (Optional):

Technical Reference(s): (Attach if not previously provided)

Proposed references to be provided to applicants, during examination:

Learning Objective: (As available)

Question Source: Bank #

Modified Bank # (Note changes or attach parent)

New Question History: Last NRC Exam NUREG-1021, Revision 9 1of2

ES-401 Sample Written Examination Form ES-401-5 Question Worksheet Question Cognitive Level: Memory or Fundamental Knowledge Comprehension or Analysis 10 CFR Part 55 Content: 55.41 55.43 Comments:

NUREG-1021, Revision 9 2of2

~~ ~

ES-401 Sample Written Examination Form ES-401-5 Question Worksheet Examination Outline Cross-reference: Level RO SRO Tier ffi 2 Group # 2 KIA ## 219000A2.03 Importance Rating 3.2 219000 Ability to (a) predict the impacts of the following on the RHFULPCI:TORUS/SUPPRESSION POOL COOLING MODE; and (b) based on those predictions, use procedures to correct, control, or mitigate the consequences of those abnormal conditions or operations: (CFR:41.5 / 45.6) A2.03 Valve closures.

Proposed Question: # 92 Following an inadvertent MSlV closure from 100% power, the plant experienced an ATWS and Loss of Offsite Power (LOOP). The following plant parameters exist:

Reactor power is 65%

Drywell pressure is 1.5 psig Suppression Pool temperature is 112OF B RHR loop is aligned for suppression pool cooling All SRVs are cycling RPV level is 100 inches and being deliberately lowered per EOP-3 The DW and TORUS SPRAY VLV OVEERRIDE OF FUEL ZONE LVL 10A-S18A&B switch has been taken to MANUAL OVERRIDE As RPV level is lowered per EOP-3:

1.) Describe the effect on suppression pool cooling 2.) Describe the actions required to reduce suppression pool temperature 1 0 ) 2.)

A. MSlV closure will seal in at 59.5 inches, Install MSlV jumpers per EP-2 to maintain which will eliminate the main condenser MSlVs open.

as heat sink B. Torus cooling valves will isolate at 59.5 Reopen torus cooling valves RHR TEST CLG inches, which will eliminate heat & SPRAY, 1OMOV-39B and TORUS CLG removal to RHRSW 1OMOV-34B C. RHR HX BYPASS VALVES 10 MOV- Close 1OMOV-66 A&B, RHR Hx BYPASS 66A&B will open at 59.5 inches, which VALVES will eliminate heat removal to RHRSW D. Torus cooling valves will isolate at 59.5 Place SPRAY CNTRL 10A-S17A&B to inches, which will eliminate heat MANUAL, spring return to normal, then removal to RHRSW reopen torus cooling valves NUREG-I 021 , Revision 9 1 of2

ES-401 Sample Written Examination Form ES-401-5 Question Worksheet Proposed Answer: D Explanation (Optional):

Tech nicaI Reference(s): (Attach if not previously provided)

Proposed references to be provided to applicants during examination:

Learning Objective: (As available)

Question Source: Bank #

Modified Bank # (Note changes or attach parent)

New X Question History: Last NRC Exam Question Cognitive Level: Memory or Fundamental Knowledge Comprehension or Analysis X 10 CFR Part 55 Content: 55.41 55.43 (5)

Comments:

NUREG-1021, Revision 9 2of2

ES-401 Sample Written Examination Form ES-401-5 Question Worksheet Examination Outline Cross-reference: Level RO SRO Tier # 2 Group # 2 KIA tf 234000 A2.01 Importance Rating 3.7 234000 Ability to (a) predict the impacts of the following on the FUEL HANDLING EQUIPMENT; and (b) based on those predictions, use procedures to correct, control or mitigate the consequences of those abnormal conditions or operations: (CFR:41.5 I 45.6) A2.01 Interlock failure Proposed Question: # 93 Refueling is in progress and bundle 37-42 is scheduled to be moved from the core to the fuel pool. The following sequence occurs:

.Upon reaching Grapple Down and Slack Cable, the grapple switch is actuated.

  • The Grapple Closed light is illuminated.
  • Upon lifting bundle 37-42, the hoist loading is 900 Ibs, but the Hoist Loaded light remains out.

1.) Describe the effect, if any, of refueling interlocks on further lifting of the bundle 2.) Describe the action, per RAP-7.1.04B, the Refueling Bridge operator must take 1.) 2.)

A. No interlocks are affected Return the bundle to full down B. Full up stop is defeated Use up jog switch to obtain grapple normal up light C. Movement to the pool inhibited Jumper refuel interlock limit switch #2 D. Unlatching is inhibited Place bundle in fuel pool and initiate WR Proposed Answer: A Explanation (Optional):

Tech nicaI Reference(s ): (Attach if not previously provided)

Proposed references to be provided to applicants during examination:

NUREG-1021, Revision 9 1 of2

ES-401 Sample Written Examination Form ES-401-5 Question Worksheet Learning Objective: (As available)

Question Source: Bank #

Modified Bank # (Note changes or attach parent)

New X Question History: Last NRC Exam Question Cognitive Level: Memory or Fundamental Knowledge Comprehension or Analysis X 10 CFR Part 55 Content: 55.41 55.43 (7)

Comments:

NUREG-1021, Revision 9 2of2

ES-401 Sample Written Examination Form ES-401-5 Question Worlksheet Examination Outline Cross-reference: Level RO SRO Tier iY 3 Group # 1 WA ## G2.1.12 SRO 55.43(b)(2)

Importance Rating 2.9 4.0 2.1. I 2 Ability to apply technical specifications for a system. (CFR: 43.2 / 43.5 / 45.3)

Proposed Question: ## 94 The plant is at 100% power.

The SLC Pump Operability and Discharge Check Valve surveillance has just been completed.

Your review of the test results reveals the following:

A Pump discharge pressure; 1300 psig, flow 52 gpm BPump discharge pressure; 1335 psig, flow 51 gpm Additionally, the RO has just reported the followiiig SLC parameters:

SLC Tank Volume 4250 gallons SLC Tank concentration 10% wt%

SLC Tank Temperature 51 OF What direction does Tech Specs give for these conditions, if any?

A. No action is required. The LCO for SLC is satisfied.

B. Immediately attempt to correct the deficiency, If not corrected within 8 hours9.259259e-5 days <br />0.00222 hours <br />1.322751e-5 weeks <br />3.044e-6 months <br />, be in Mode 3 within 12 hours1.388889e-4 days <br />0.00333 hours <br />1.984127e-5 weeks <br />4.566e-6 months <br /> of required action and completion time not met.

C. Immediately attempt to correct the deficiency, If not corrected within 8 hours9.259259e-5 days <br />0.00222 hours <br />1.322751e-5 weeks <br />3.044e-6 months <br />, be in Mode 3 within 12 hours1.388889e-4 days <br />0.00333 hours <br />1.984127e-5 weeks <br />4.566e-6 months <br /> of discovery.

D. Commence an orderly reactor shutdown such that the reactor is in cold shutdown within 24 hours2.777778e-4 days <br />0.00667 hours <br />3.968254e-5 weeks <br />9.132e-6 months <br />.

Proposed Answer: B Explanation (Optional): See attached TS 3.1.7 A. Incorrect -Concentration vs. solution temperature does not meet figure 3.1.7-2 requirement, therefore 3.1.7 is not met B. Correct - Per LCO 3.1.7 Action C. Incorrect - Concentration vs. solution terriperature does not meet figure 3.1.7-2 requirement, therefore 3.1.7 is not met D. Incorrect - Concentration vs. solution temperature does not meet figure 3.1.7-2 requirement, therefore 3.1.7 is not met NUREG-1021, Revision 9

ES-40 1 Sample Written Examination Form ES-401-5 Question Worksheet Technical Reference(s): TS LCO 3.1.7 (Attach if not previously provided)

Proposed references to be provided to applicants during examination: TS LCO 3.1.7 Learning Objective: SDLP-11 EO-1.18 (As available)

Question Source: Bank #

Modified Bank # (Note changes or attach parent)

New X Question History: Last NRC Exam Question Cognitive Level: Memory or Fundamental Knowledge Comprehension or A.nalysis X 10 CFR Part 55 Content: 55.41 55.43 (2)

Comments:

NUREG-1021, Revision 9 2Of4

ES-401 Sample Written Examination Form ES-401-5 Quest ion W orl<sheet 3.1 REACTIVITY CONTROL SYSTEMS 3.1.7 Standby Liquid Control (SLC) System LCO 3.1.7 Two SLC subsystems shall be OPERABLE.

APPLICABILITY: MODES I and 2.

CON0 ITION REQUIRED ACTION COMPLETION TIME A. One SLC subsystem A.l Restore SLC subsystem 7 days i noperab l e. t o OPERABLE status.

  • 1 I I I B. Two SLC subsystems 6.1 Restore one SLC a hours inoperabl e. subsystem t o OPERABLE status.

I C. Required Action and c.1 I

Be in MODE 3. 12 hours1.388889e-4 days <br />0.00333 hours <br />1.984127e-5 weeks <br />4.566e-6 months <br /> associated Cmpl etion Time not met.

I I NUREG-1021, Revision 9

ES-40 1 Sample Written Examination Form ES-401-5 Question Worksheet I00 90 80 70 60 6 8 10 12 14 te 10 20 Concentration (Weight Percent EnvEched Sodium Fentabomta)

Figure 3.1.7-2 (page 1 of 1)

Sodium Pentaborate Solution Temperature Yersus Concentratf on Requi rements NUREG-I 021, Revision 9 4 of 4

ES-401 Sample Written Examination Form ES-401-5 Question Worksheet Examination Outline Cross-reference: Level RO SRO Tier # 3 Group # 1 WA #: G2.1.10 I mpoiqance Rating 2.7 3.9 2.1. I O Knowledge of conditions and limitations in the facility license. (CFR: 43.1 I45.13)

Proposed Question: # 95 With the plant at 100% power, you discover the last performance of ST-GHA, Standby Liquid Control A Side Quarterly Operability Test, was completed one hundred- five (105) days ago.

Which ONE of the following is the implication of this discovery?

A. The surveillance interval of TS SR3.0.2 is not exceeded and the LCO is still met. TS SR 3.0.3 does not apply.

B. The surveillance interval of TS SR3.0.2 is exceeded and the applicable TS actions must be entered. TS SR 3.0.3 does not apply.

C. TS SR 3.0.3 applies for a missed ST. The LCO must be declared not met and the applicable TS actions entered.

D. TS SR 3.0.3 applies for a missed ST. Declaring the LCO statement not met can be delayed for up to 24 hours2.777778e-4 days <br />0.00667 hours <br />3.968254e-5 weeks <br />9.132e-6 months <br /> to perform the ST.

Proposed Answer: A NUREG-I 021, Revision 9 1 of6

ES-401 Sample Written Examination Form ES-401-5 Question Worksheet Explanation (Optional): Per ST-6HA R3, it is required to be performed on a frequency of 92 days.

LCO 3.0.2 Upon discovery of a failure to meet an LCO, the Required Actions of the associated Conditions shall be met, except as provided in LCO 3.0.5 and LCO 3.0.6. If the LCO is met or is no longer applicable prior to expiration of the specified Completion Time(s), completion of the Required Action(s) is not required, unless otherwise stated.

3.0 SURVEILLANCE REQUIREMENT (SR) APPLICABILITY SR 3.0.1 SRs shall be met during the MODES or other specified conditions in the Applicability for individual LCOs, unless otherwise stated in the SR. Failure to meet a Surveillance, whether such failure is experienced during the performance of the Surveillance or between performances of the Surveillance, shall be failure to meet the LCO. Failure to perform a Surveillance within the specified Frequency shall be failure to meet the LCO except as provided in SR 3.0.3. Surveillances do not have to be performed on inoperable equipment or variables outside specified limits.

LCO 3.0.3 When an LCO is not met and the associated ACTIONS are not met, an associated ACTION is not provided, or if directed by the associated ACTIONS, the plant shall be placed in a MODE or other specified condition in which the LCO is not applicable. Action shall be initiated within 1 hour1.157407e-5 days <br />2.777778e-4 hours <br />1.653439e-6 weeks <br />3.805e-7 months <br /> to place the plant, as applicable, in:

a. MODE 2 within 7 hours8.101852e-5 days <br />0.00194 hours <br />1.157407e-5 weeks <br />2.6635e-6 months <br />;
b. MODE 3 within 13 hours1.50463e-4 days <br />0.00361 hours <br />2.149471e-5 weeks <br />4.9465e-6 months <br />; and
c. MODE 4 within 37 hours4.282407e-4 days <br />0.0103 hours <br />6.117725e-5 weeks <br />1.40785e-5 months <br />.

Exceptions to this Specification are stated in the individual Specifications. Where corrective measures are completed that permit operation in1 accordance with the LCO or ACTIONS, completion of the actions required by LCO 3.0.3 is not required. LCO 3.0.3 is only applicable in MODES 1,2, and 3.

3.0 SURVEILLANCE REQUIREMENT (SR) APF'LICABILITY SR 3.0.3 If it is discovered that a Surveillance was not performed within its specified Frequency, then compliance with the requirement to declare the LCO not met may be delayed, from the time of discovery, up to 24 hours2.777778e-4 days <br />0.00667 hours <br />3.968254e-5 weeks <br />9.132e-6 months <br /> or up to the limit of the specified Frequency, whichever is greater. This delay period is permitted to allow performance of the Surveillance. A risk evaluation shall be performed for any Surveillance delayed greater than 24 hours2.777778e-4 days <br />0.00667 hours <br />3.968254e-5 weeks <br />9.132e-6 months <br /> and the risk impact shall be managed. If the Surveillance is riot performed within the delay period, the LCO must immediately be declared not met, and the applicable Condition(s) must be entered.

When the Surveillance is performed within the delay period and the Surveillance is not met, the LCO must immediately be declared not met, and the applicable Condition(s) must be entered.

A. Correct - Quarterly is 92 days X 1.25 = 'I 15 days which is c 105 days, the SR is into the 1.25 grace period which has not been exceeded.

6. Incorrect - Quarterly is 92 days X 1.25 =: 115 days which is < 105 days, the SR is into the 1.25 grace period which has not been exceeded.

C. Incorrect - TS SR 3.03 does not apply for declaring the LCO not met.

D. Incorrect - TS SR 3.03 does not apply for declaring the LCO not met. Provided the ST is completed within the 115 day period and the risk management has been performed.

Technical Reference(s): TS SR 3.02 TS SR :3.03 (Attach if not previously provided)

ST-6HA R 3 Proposed references to be provided to applicants during examination: TS SR 3.0 Series NUREG-1021, Revision 9 2of6

ES-401 Sample Written Examination Form ES-401-5 Question Worksheet Learning Objective: SDLP-11 EO-1.18 (As available)

Question Source: Bank #

Modified Bank # (Note changes or attach parent)

New X Question History: Last NRC Exam Question Cognitive Level: Memory or Fundamental Knowledge Comprehension or Analysis X 10 CFR Part 55 Content: 55.41 55.43 (2)

Comments: VERIFY WA Believe it was supposed to be G2.1.30 Ability to locate and operate components / including local controls. (CFR: 41 .'7 I 45.7)

NUREG-1021, Revision 9 3of6

ES-401 Sample Written Examination Form ES-401-5 Question Worksheet 1.0 USE AND APPLICATION 1.4 f fequency PURPOSE The purpose o f t h i s section i s t o define the proper use and application o f Frequency requirements.

~ ~~

DESCRIPTION Each Surveillance Requirement (SR) has a specified Frequency i n which the Surveillance must be met i n order t o meet the associated Limiting Condition for Operation (LCO). An understanding o f the correct ap l i c a t i o n of the specified 7

Frequency i s necessary for comp iance with the SR.

The "specified Fre uericy" i s referred t o throughout t h i s 8

section and each o the Specifications of Section 3.0, Surveillance Requirement (SR) Applicability. The "specified Frequency" consists ob' the requirements o f the Frequency column of each SR. as well as certain Notes i n the Surveillance column t h a t modify performance requirements.

Sometimes s i a l situations dictate when the requirements Y

o f a Survei lance are t o be met. They are "otherwise stated" conditions allowed by SR 3.0.1. They may be stated as clarifying Notes i n the Surveillance, as part of the Surveillance. or both.

Situations where a Surveillance could be required N e . . i t s Frequency could expire). but where i t i s not possible or not desired that it be perfo u n t i l sometime after the associated LCO i s within i t s Applicability. represent tential SR 3.0.4 conflicts. To avoid these conflicts, the 8 (i.e.. the Surveillance or the Frequency) i s stated such that it i s only "required" when i t can be and should be performed. With an SF! satisfied, SR 3.0.4 imposes no restriction.

The use of "met" or "performed" i n these instances conveys specific meanings. A Surveillance i s "met" only when the acceptance c r i t e r i a are s a t i s f i e d . Known failure o f the requi rements o f a Survei 1 1ance, even without a Survei 1I ance specificafly being "performed. constitutes a Surveillance not "met." "Performance" refers only t o the requirement t o speci f i c a f l y determine! the a b i l i t y t o meet the acceptance (cont inued)

NUREG-1021, Revision 9 4of6

ES-401 Sample Written Examination Form ES-401-5 Question Worksheet 1.4 Frequency DESCRIPTION c r i t e r i a . Some Surveillances contain notes that modify the (continued) Frequency o f performance o f the conditions Wring whlch the acceptance c r i t e r i a mist be satisfied. For these Surveillances, the MOO entry restrictions o f SR 3.0.4 may not apply. Such a Surveillance i s not required t o be performed prior t o enterjng a MODE or other specifted condition i n the Applicability o f the associated LCO i f any o f the following three conditions are satisfied:

a. The Surveillance i s not required t o be met i n the NODE or other specified condition t o be entered; or
b. The Surveillance i s required t o be met i n the MODE or other specified condition t o be entered, but has k e n performed within the specified Frequency (i ,e. i t i s current) and I s known not t o be failed; or
c. The Surveillance i s required t o be met. but not rformed. i n the MODE or other specified condition t o

!iE entered, and i s known not t o be f a i l e d .

Examples 1.4-3.1 . 4 - 4 . 1.4-5, and 1.4-6 discuss these special situations.

EXAMPLES The following examples i l l u s t r a t e the varlous ways t h a t Frequencies are specified. In these examples. the Applicability o f the LCO (LCO not shown) i s MODES 1. 2, and 3.

(continued)

NUREG-1021, Revision 9 5of6

ES-401 Sample Written Examination Form ES-401-5 Question Worksheet SURVEILLANCE REQUIREMENTS (continued)

SVRVEI LLANCE 1 FREQUENCY SR 3.1.7.6 Verif each SLC subsystem manual valve i n 31 days 7

the f ow path t h a t is not locked, sealed, or otherwise secured i n position is i n the correct position, or can be atigned t o the correct position.

SR 3.1.7.7 Verify each pump develops a flow rate In accordance 2: 50 pm a t a discharge pressure w i t h the 2  !

127 psig. Inservi ce Test i ng Program NUREG-1021. Revision 9 6of6

~~ ~ ~

ES-40 1 Sample Written Examination Form ES-401-5 Question Worksheet Examination Outline Cross-reference: Level RO SRO Tier Xt 3 Group ## 2 WA #i G2.2.8 Impoirtance Rating 1.8 3.3 2.2.8 Knowledge of the process for determining if the proposed change I test / or experiment involves an unreviewed safety question. (CFR: 43.3 / 45.13)

Proposed Question: ## 96 Per EN-LI-101, which of the following are considered unreviewed Safety Questions?

1. Emergency actions that depart from TS are needed to protect the public health and safety.
2. The possibility of an accident exists that has not been evaluated by the FSAR.
3. The PClS isolation time for the MSlVs is being increased by 50%.
4. Change the design basis limit for a fission product barrier as described in the FSAR.
5. An emergency event that can not be classified by the Emergency plan.

B. 3, 4, 5 C. 2, 3, 4 D. 2, 3, 5 Proposed Answer: C NUREG-1021, Revision 9 1 of6

Explanation (Optional): See attached 50.59. Per EN-LI-101 R3 General Guidance:

The 50.59 Evaluation criteria must be answered uniquely based on the question being asked.

Repeating responses without consideration of the question generally indicates that the Preparer is not performing a complete Evaluation; therefore, it may be returned for revision. The importance of the documentation is emphasized by the fact that experience and engineering knowledge (other than analytical models and experimental data) are often relied upon when performing the 50.59 Evaluation. Since an imporlant goal of the 50.59 Evaluation is completeness, the items considered by the Preparer must be clearly stated. The 50.59 Evaluation should, as a minimum, consider the fcillowing: a) What systems and components are affected by the change and what are the functions of those systems affected? b) What parameters of the accident analysis are affected by the change? c) What design basis accidents were reviewed for any effect? Questions such as the following may be helpful in focusing the 50.59 Evaluation and its review. a) What is the intent of the Change? b) What is (are) the safety function(s) added, altered, deleted or preserved by the proposed Change? c) What interactions may occur between the Change and other systernskomponents? d) Are there seemingly positive aspects of the Change which could, in unusual circumstances, adversely affect safety? ATTACHMENT 9.1 -50.59 EVALUATION FORMDoes the proposed Change: 1) Result in more than a minimal increase in the frequency of occurrence of an accident previously evaluated in the UFSAR? 2) Result in more than a minimal increase in the likelihood of occurrence of a malfunction of a structure, system, or component important to safety previously evaluated in the UFSAR? 3) Result in more than a minimal increase in the consequences of an accident previously evaluated in the UFSAR? 4)

Result in more than a minimal increase in the consequences of a malfunction of a structure, system, or component important to safety previously evaluated in the UFSAR? 5) Create a possibility for an accident of different type than any previously evaluated in the UFSAR? 6) Create a possibility for a malfunction of a structure, system, or component important to safety with a different result than any previously evaluated in the UFSAR? 7) Result in a design basis limit for a fission product barrier as described in the UFSAR being exceeded or altered? 8) Result in a departure from a method of evaluation described in the UFSAR used in establishing the design bases or in the safety analyses?

A. Incorrect - Emergency actions that depart from TS are needed to protect the public health & safety have been evaluated & do not meet this criteria.

B. Incorrect - E-Plan events outside EALs are covered by EAL 9.1 Judgment of individual with Command Authority.

C. Correct -Meets EN-LI-101 Attachment 9.1 criteria.

D. Incorrect - E-Plan events outside EALs are covered by EAL 9.1 Judgment of individual with Command Authority.

Technical Reference(s): EN-LI-101 R3 10CFR 50.59 (Attach if not previously provided)

Proposed references to be provided to applicants during examination: NONE Learning Objective: LP-AP EO-4.04 (As available)

Question Source: Bank #

Modified Bank # (Note changes or attach parent)

New X Question History: Last NRC Exam NUREG-I 021, Revision 9 2 of 6

~~

ES-401 Sample Written Examination Form ES-401-5 Question Worksheet Question Cognitive Level: Memory or Fundamental Knowledge X Comprehension or Analysis 10 CFR Part 55 Content: 55.41 55.43 (3)

Comments: Changed stem choices 3 and 4.

The previous choice 4 referred to a change that reduced margin to safety as defined in TS. This isnt part of the 50.59 evaluation form (so not clearly a wrong choice). Instead, substituted with a fission process barrier change with is number iin the 50.59 evaluation form.

On choice 3, changed it so it doesnt reference the FSAR in the choice and instead. The change is for an FSAR system (MISVs-PCIS) that changes the assumptions in the accident analysis (MSIV closing times for an accident). This would be number 3 in the 50.59 evaluation form.

NUREG-1021, Revision 9 3of6

ES-40 1 Sample Written Examination Form ES-401-5 Question Worksheet 5 50.59 Changes, tests and experiments.

(a) Definitions for the purposes of this section:

(1) Change means a modification or addition to, or removal from, the facility or procedures that affects a design function, method of performing or controlling the function, or an evaluation that demonstrates that intended funlctions will be accomplished.

(2) Departure from a method of evaluation described in the FSAR (as updated) used in establishing the design bases or in the safety analyses means:

(i) Changing any of the elements of the method described in the FSAR (as updated) unless the results of the analysis are conservative or essentially the same; or (ii) Changing from a method described in the FSAR to another method unless that method has been approved by NRC for the intended applici3t'Ion.

(3) Facility as described in the final safety analysis report (as updated) means:

(i) The structures, systems, and components (SSC) that are described in the final safety analysis report (FSAR) (as updated),

(ii) The design and performance requirements for such SSCs described in the FSAR (as updated), and (iii) The evaluations or methods of evaluation iricluded in the FSAR (as updated) for such SSCs which demonstrate that their intended function(s) will be accomplished.

(4) Final Safety Analysis Report (as updated) means the Final Safety Analysis Report (or Final Hazards Summary Report) submitted in accordance with Sec. 50.34, as amended and supplemented, and as updated per the requirernents of Sec. 50.71(e) or Sec. 50.71(f), as applicable.

(5) Procedures as described in the final safety analysis report (as updated) means those procedures that contain information described in the FSAR (as updated) such as how structures, systems, and components are operated and controlled (including assumed operator actions and response times).

(6) Tests or experiments not described in the final safety analysis report (as updated) means any activity where any structure, system, or cornponent is utilized or controlled in a manner which is either:

(i) Outside the reference bounds of the design bases as described in the final safety analysis report (as updated) or (ii) Inconsistent with the analyses or descriptions in the final safety analysis report (as updated).

NUREG-1021, Revision 9 4of6

ES-401 Sample Written Examination Form ES-401-5 Q uest ion Worksheet (b) This section applies to each holder of an operating license issued under this part or a combined license issued under part 52 of this chapter, including the holder of a license authorizing operation of a nuclear power reactor that has submitted the certification of permanent cessation of operations required under § 50.82(a)( 1) or 5 50.110 or a reactor licensee whose license has been amended to allow possession of nuclear fuel but not operation of the facility.

(c)(l) A licensee may make changes in the facility as described in the final safety analysis report (as updated), make changes in the procedures as described in the final safety analysis report (as updated), and conduct tests or experiments not described in the final safety analysis report (as updated) without obtaining a license amendment pursuant to Sec. 50.90 only if:

(i) A change to the technical specifications incorporated in the license is not required, and (ii) The change, test, or experiment does not meet any of the criteria in paragraph (c)(2) of this section.

(2) A licensee shall obtain a license amendment pursuant to Sec. 50.90 prior to implementing a proposed change, test, or experiment if the change, test, or experiment would:

(i) Result in more than a minimal increase in the frequency of occurrence of an accident previously evaluated in the final safety analysis report (as updated);

(ii) Result in more than a minimal increase in the likelihood of occurrence of a malfunction of a structure, system, or component (SSC) important to safety previously evaluated in the final safety analysis report (as updated);

(iii) Result in more than a minimal increase in the consequences of an accident previously evaluated in the final safety analysis report (as updated);

(iv) Result in more than a minimal increase in the consequences of a malfunction of an SSC important to safety previously evaluated in the final safety analysis report (as updated);

(v) Create a possibility for an accident of a different type than any previously evaluated in the final safety analysis report (as updated);

(vi) Create a possibility for a malfunction of an SSC important to safety with a different result than any previously evaluated in the final safety analysis report (as updated);

(vii) Result in a design basis limit for a fission product barrier as described in the FSAR (as updated) being exceeded or altered; or (viii) Result in a departure from a method of evaluation described in the FSAR (as updated) used in establishing the design bases or in the safety analyses.

(3) In implementing this paragraph, the FSAR (as updated) is considered to include FSAR changes resulting from evaluations performed pursuant to this section and analyses performed pursuant to Sec. 50.90 since submittal of the last update of the final safety analysis report pursuant to Sec. 50.71 of this part.

NUREG-1021, Revision 9 5of6

ES-40 1 Sample Written Examination Form ES-401-5 Question Worksheet (4) The provisions in this section do not apply to changes to the facility or procedures when the applicable regulations establish more specific criteria for accomplishing such changes.

(d)(l) The licensee shall maintain records of changes in the facility, of changes in procedures, and of tests and experiments made pursuant to paragraph (c) of this section. These records must include a written evaluation which provides the bases for the determination that the change, test, or experiment does not require a license amendment pursuant to paragraph (c)(2) of this section.

(2) The licensee shall submit, as specified in § 50.4 or 5 52.3 of this chapter, as applicable, a report containing a brief description of any changes, tests, and experiments, including a summary of the evaluation of each. A report must be submitted at intervals not to exceed 24 months. For combined licenses, the report must be submitted at intervals not to exceed 6 months during the period from the date of appli'cationfor a combined license to the date the Commission makes its findings under 10 CFR 52.103(g).

(3) The records of changes in the facility must be maintained until the termination of an operating license issued under this part, a combined license issued under part 52 of this chapter, or the termination of a license issued under 10 CFR part 54, whichever is later.

Records of changes in procedures and records of tests and experiments must be maintained for a period of 5 years.

[64 FR 53613, Oct. 4, 1999, as amended at 66 FIR 64738, Dec. 14,2001; 72 FR 49500, Aug.

28, 20071 NUREG-I 021, Revision 9 6of6

ES-401 Sample Written Examination Form ES-401-5 Question Worksheet Examination Outline Cross-reference: Level RO SRO Tier #: 3 Group # 2 KIA # ~~~

G2.2.24 Importance Rating 2.6 3.8 2.2.24 Ability to analyze the affect of maintenance activities on LCO status.(CFR: 43.2 / 45.13)

Proposed Question: # 97 The plant is at 100% power.

Electrical Maintenance has submitted a work package to overhaul the 1OMOV-l5A, RHR Pump A Shutdown Cooling Suction Isolation, motor actuator. The work package requires the 1OMOV-13A, RHR Pump A Torus Suction Isolation, to be closed to allow manual operation of the 1OMOV-15A valve limit switches.

What Limiting Condition(s) of Operation, if any, will be in effect?

A. No LCO required.

B. LCO 3.5.1 C. LCO 3.4.7, LCO 3.5.1, LCO 3.6.1.3 D. LCO 3.5.1, LCO 3.6.1.9, LCO 3.6.2.3 Proposed Answer: B Explanation (Optional): To open the 15A valve, the 13A valve must be closed, taking the RHR pump out of service.

A. Incorrect - RHR pump A is inoperable with MOV 13A closed.

B. Correct - LCO 3.5.1 C. Incorrect - SDC not required in MODE 1, MOV-13A is not a PCIV.

D. Incorrect - Suppression Pool Cooling (Containment Spray) subsystem requires only one pump per subsystem.

Technical Reference(s): LCO 3.5.1 LCO 3.4.7 (Attach if not previously provided)

LCO 3.6.1.3 LCO 3.6.1.9, LCO 3.6.2.3 Proposed references to be provided to applicants during examination: LCO 3.5.1, 3.4.7, 3.6.1.3, 3.6.1.9, 3.6.2.3 Learning Objective: (As available)

Question Source: Bank #

NUREG-1021, Revision 9 1 of2

ES-40 1 Sample Written Examination Form ES-401-5 Question Worksheet Modified Bank # X (Note changes or attach parent)

New Question History: Last NRC Exam Question Cognitive Level: Memory or Fundamental Knowledge Comprehension or Analysis X 10 CFR Part 55 Content: 55.41 55.43 (2)

NUREG-1021, Revision 9 2of2

ES-401 Sample Written Examination Form ES-401-5 Question Worksheet Examination Outline Cross-reference: Level RO SRO Tier # 3 Group # 3 KIA #t G 2.3.3 Importance Rating 2.9 2.3.3 Knowledge of SRO responsibilities for auxiliary systems that are outside the control room (e.g.

/waste disposal and handling systems). (CFR: 43.4 / 45.10)

Proposed Question: # 98 The plant is starting up with the reactor at 50% power.

A liquid radwaste release per OP-49 is planned on your shift. The following is noted on the liquid radwaste release permit provided to you for review and approval:

Required dilution factor 100 DF Time in recirc 0900 4/14/08 Time of sample 1200 4114/08 Specific activity 3.8x10-4pCi/ml Number of circulating pumps required 3 Discharge rate 20 gpm Per procedure SP-01.05, Waste Water Sampling and Analysis, 1.) Determine whether you may approve the permit , AND 2.) The basis for your decision.

1.) ApprovelDisapprove 2.) Basis A. Disapprove May not run 3 circulating pumps at this power level B. Approve Acceptable to run all circulating pumps at this power level C. Disapprove Inadequate recirculation time D. Approve Activity is below IOCFRIOO limits Proposed Answer: 6 Explanation (Optional):

Tech nicaI Reference(s ): (Attach if not previously provided)

NUREG-1021, Revision 9 1 of 2

ES-401 Sample Written Examination Form ES-401-5 Question Worksheet Proposed references to be provided to applicants during examination:

Learning Objective: (As available)

Question Source: Bank #

Modified Bank # (Note changes or attach parent)

New X Quest ion History: Last NRC Exam Question Cognitive Level: Memory or Fundamental Knowledge Comprehension or Analysis X 10 CFR Part 55 Content: 55.41 55.43 (4)

Comments:

NUREG-1021, Revision 9 2 of 2:

-~

ES-401 Sample Written Examination Form ES-401-5 Question Worksheet Examination Outline Cross-reference: Level RO SRO Tier kt 3 Group # 3 WA #: G.2.3.1 Importance Rating 2.6 3.0 G.2.3.1 Knowledge of 10 CFR: 20 and related facility radiation control requirements. (CFR: 41 . I 2 / 43.4 45.9 /45. IO)

Proposed Question: # 99 A Planned Special Exposure (PSE) is required.

APSEis-(I) .

When authorizing a PSE, 10 CFR 20 (2) are taken into account.

(1) (2)

A. Used whenever immediate measures are Annual TEDE limits of 10 Rem and required to save lives or equipment. Lifetime TEDE limits of 50 Rem

6. An infrequent exposure to radiation separate Annual TEDE limits of 5 Rem and from and in addition to annual limits. Lifetime TEDE limits of 25 Rem C. Used whenever immediate measures are Annual TEDE limits of 5 Rem and required to save lives or equipment. Lifetime TEDE limits of 25 Rem D. An infrequent exposure to radiation separate Annual TEDE limits of 10 Rem and from and in addition to annual limits. Lifetime TEDE limits of 50 Rem Proposed Answer: B Explanation (Optional):

A. -

B. -

C. -

D.

Technical Reference(s): EN-RP-207, Planned Special (Attach if not previously provided)

Exposure Proposed references to be provided to applicants during examination:

NUREG-1021, Revision 9 1of2

ES-401 Sample Written Examination Form ES-401-5 Question Worksheet Learning Objective: (As available)

Question Source: Bank #

Modified Bank ## (Note changes or attach parent)

New Question History: Last NRC Exam Quest ion Cognitive Level: Memory or Fundamental Knowledge Comprehension or Analysis 10 CFR Part 55 Content: 55.41 55.43 Comments:

NUREG-1021, Revision 9 2 of 2

ES-40 1 Sample Written Examination Form ES-401-5 Question Worksheet Examination Outline Cross-reference: Level RO SRO Tier # 3 Group # 4 KIA #: G 2.4.3 Importance Rating 3.8 G 2.4.3 Ability to identify post-accident instrumentation (CFR:41.6 / 45.4)

Proposed Question: # 100 With the Unit at 100% power, I&C reports that A radiation monitor 17RM-431 circuit board associated with TURB BLDG EXH RAD MON INOP OR HI alarm, has failed. This is the same failure which occurred to the 9 channel two months ago. The B channel repair is waiting on a circuit board to arrive from the vendor.

What action(s) and time limits, if any, are required to be taken for the second failure?

A. No additional actions are required since this is not a Tech Spec required system.

9. Obtain grab samples and restore required channel to operable status within 30 days, or provide explanation in next Radiation Release Report why inoperability was not corrected within 30 days.

C. Obtain grab samples and restore required channel to operable status within 30 days or initiate a condition report immediately.

D. Verify that the plant is not in an unanalyzed condition AND within 12 hours1.388889e-4 days <br />0.00333 hours <br />1.984127e-5 weeks <br />4.566e-6 months <br /> obtain the Operations Manager approval of the compensatory actions.

Proposed Answer: B Explanation (Optional):

Technical Reference(s): (Attach if not previously provided)

Proposed references to be provided to applican,ts during examination:

Learning Objective: (As available)

Question Source: Bank #

Modified Bank # X (Note changes or attach parent)

New NUREG-I 021, Revision 9 1 of2

ES-401 Sample Written Examination Form ES-401-5 Question Worksheet Question History: Last NRC Exam Question Cognitive Level: Memory or Fundamental Knowledge Comprehension or Analysis X 10 CFR Part 55 Content: 55.41 55.43 (2)

Comments:

NUREG-1021, Revision 9 2 of 2!